You are on page 1of 51

ASH NEWS AND REPORTS

Ask the Hematologist


COMPENDIUM 2010-2015
This collection contains updated clinical information on malignant
and nonmalignant hematology as profiled in The Hematologist.

Editor: Jason Gotlib, MD, MS

Ask the Hematologist Compendium


INTRODUCTION

A core feature of the ASH publication The Hematologist: ASH News


and Reports is the department known as Ask the Hematologist
(ATH). The aim of ATH is to provide clinicians with expert opinion
about challenging diagnostic and management problems that span the
areas of malignant and nonmalignant hematology. In some instances,
the question posed in the ATH column is premised on a very specific
patient scenario; in others, the question is intended to elicit a broader
review of practice-changing diagnostic algorithms or therapeutic
advances in hematology. ATH articles commonly touch on disease
pathophysiology that helps anchor clinical explanations to the biologic
underpinnings of disease. We have found that this translational mix
of information has appeal not only to clinicians, but also to physiciancollection
updated clinical information on malignant
scientistscontains
and basic researchers.

This
and nonmalignant hematology as profiled in The Hematologist.
In this compendium, we have compiled ATH articles spanning the years
2010 to 2015. With the exception of some more recent works, we
have asked authors to provide state-of-the-art updates to the content
where applicable; to that end, authors either have rewritten their ATH
articles or have provided an update/commentary that introduces their
original piece. We hope that readers find the ATH compendium to
be an easily digestible learning companion that serves as a portable
feast for the practicing hematologist.

Jason Gotlib, MD, MS


Editor-in-Chief

irector of Communications
D
Jenifer Hamilton, CAE

Managing Editor
Juana Llorens, MS

Editorial Associate
Laura Santini

Graphic Designer
grayHouse design

Table of Contents
M A L I G N A N T H E M AT O L O GY :
73-Year-Old Male with Stage 0 Chronic Lymphocytic Leukemia with Skin Involvement and No Evidence of Mantle Cell Lymphoma................................

Approach to Patient with a Bladder Tumor with Severe Hematuria and Hemoglobin Level of Eight with Associated Dyspnea.............................................

Recommendation of CHOP-Rituximab/More Intensive Regimen Such as Dose-Adjusted EPOCH-Rituximab for Cyclin D1 Expression in DLBCL..........

FDA Approval of the JAK Inhibitor Ruxolitinib and the Management of Patients with Myelofibrosis................................................................................................

Evaluation of Young Adult with Bone Marrow Failure...............................................................................................................................................................................

10

Standard of Care for Treating Older Patients with Chronic Lymphocytic Leukemia........................................................................................................................

12

Optimal, Upfront Management of Mantle Cell Lymphoma.......................................................................................................................................................................

14

Approach to the Diagnosis and Management of Mastocytosis..............................................................................................................................................................

16

Assessment and Management of Patients with Chemotherapy-Induced Peripheral Neuropathy................................................................................................

18

Approach to the Evaluation of Patients with an IgM Monoclonal Protein............................................................................................................................................

20

Treatment of Elderly Patients with Acute Myeloid Leukemia...................................................................................................................................................................

22

Treatment of Relapsed/Refractory Hairy Cell Leukemia...........................................................................................................................................................................

24

Diagnosis of Immune Pathophysiology in Patients with Low-Risk Myelodysplastic Syndromes...................................................................................................

25

Hematopathologic Overview of Lymphocytosis.........................................................................................................................................................................................

27

N O N M A L I G N A N T H E M AT O L O GY :
Specific Thrombophilia Work-Up Approach................................................................................................................................................................................................

30

Recommendations for Thrombophilia Testing for Women before Oral Contraceptives are Prescribed....................................................................................

31

Treatment of Pregnant Female with Mesenteric Thrombosis and History of Spontaneous Fetal Losses..................................................................................

32

Factor Replacement for Surgery in 48-Year-Old Male with Factor XI Deficiency.............................................................................................................................

33

Immune Thrombocytopenia with Pulmonary Embolism and Deep-Vein Thrombosis: Recommendations for Bone Marrow Aspirate and Biopsy.........

34

Heparin-Induced Thrombocytopenia Being Treated with Argatroban with Persistent Thrombocytopenia................................................................................

35

Approach to the Diagnosis and Management of the Anemia of Chronic Inflammation...................................................................................................................

37

Approach to the Diagnosis and Management of Transfusion-Related Acute Lung Injury...............................................................................................................

39

Criteria for Selecting Patients with Sickle Cell Anemia for Allogeneic Hematopoietic Stem Cell Transplantation..................................................................

41

Diagnosis and Management of Platelet Alloimmunization........................................................................................................................................................................

42

Recommendation of Light Transmission Aggregometry as Part of the Evaluation of a Patient with a Suspected Bleeding Disorder...............................

44

Anti-Phospholipid Antibodies and Pregnancy.............................................................................................................................................................................................

46

Treatment Approaches to Castleman Disease with the Advent of Anti-Interleukin-6 Therapy......................................................................................................

48

Treatment with Direct Oral Anticoagulants..................................................................................................................................................................................................

50

Ask the Hematologist


M A L I G N A N T H E M AT O L O GY
This collection contains updated clinical information on malignant
and nonmalignant hematology as profiled in The Hematologist.

73-Year-Old Male with Stage 0 Chronic Lymphocytic Leukemia with Skin


Involvement and No Evidence of Mantle Cell Lymphoma March/April 2010
The author has indicated that there has been no update regarding the content of this article since the original publication date in 2010.

SUSAN OBRIEN, MD
Associate Director for Clinical Science, Chao Family Comprehensive Cancer Center; Medical
Director, Sue & Ralph Stern Center for Clinical Trials and Research; Professor, Division of
Hematology/Oncology, School of Medicine, University of California, Orange, CA

(Editors Note: The original question was submitted to Dr. OBrien through ASHs
Consult a Colleague, a service for ASH members that helps facilitate the exchange
of information between hematologists and their peers. She expanded her answer for print.)

The Question
I have been following a 73-year-old male with Stage 0 chronic lymphocytic leukemia
(CLL) for two years. He has a normal hemoglobin and platelet count with a white blood
cell count of around 15,000 and cytogenetics showing 13q deletion and trisomy 12. He
recently had two small asymptomatic skin lesions removed from his arm and back. These
consisted of a clonal population of B cells that expressed Bcl-2 diffusely and were CD20positive, but CD5- and CD43-negative. This was similar to his original flow cytometry from
peripheral blood. This case was interpreted by a referral lab as B-cell lymphoma. The
patient is completely asymptomatic with no evidence of liver or spleen enlargement or
adenopathy on CT scans. Im not sure how to put this all together; is this atypical CLL with
skin involvement? There is no evidence of mantle cell lymphoma.

My Response
This case presents two unusual aspects of CLL. The first is that this patient has small skin
lesions that appear to be deposits of CLL. The second unusual feature is that this patients
cells are CD5-negative. Looking at the whole picture, I would agree that this is most likely
atypical CLL with skin involvement.
I will discuss the phenotype first. CLL is a B-cell malignancy that also expresses CD5. The
other well-known lymphoid malignancy that expresses CD5 in addition to B-cell markers is
mantle cell lymphoma, which can present in a leukemic phase. Typically, the morphology
of mantle cell is significantly different from CLL, although in less classical cases, there may
be some blurring of the morphology. CD23 is a useful marker to differentiate them, as this
antigen is present on CLL cells and not present on mantle cells. However, in this case we have
a patient who appears to have CLL but is CD5-negative.

From a practical point of view, the good news is that the approach to MZL is similar to that
of CLL namely watch-and-wait in early-stage asymptomatic patients. One difference,
however, is in the approach to treatment. MZL cells have very strong expression of CD20
(as opposed to CLL cells), and these patients have dramatic responses to single-agent
rituximab. In patients who need treatment, I generally use 375 mg/m2 rituximab weekly for
eight weeks and have had patients in complete remission for years after this single agent,
whereas the efficacy of single-agent rituximab in CLL is much weaker. The reason I would
diagnose CLL in this case is the presence of a 13q deletion and trisomy 12. These are wellknown abnormalities seen in CLL, and, in fact, the presence of trisomy 12 is commonly
associated with atypical features.
Skin involvement in CLL is quite rare. Probably the largest series to examine this came
from a group of pathologists in Austria, who describe cutaneous infiltrates of CLL in 42
patients. The five-year survival of these patients was 67 percent, suggesting that they had
a good, if not a more favorable, prognosis compared to those without skin involvement;
there was no attempt to match them for comparable prognostic features to patients
without skin involvement.2,3
Ive been following some patients for years with skin involvement. In particular, Im
thinking of one who has small deposits on both earlobes and has very indolent CLL thats
never required treatment. If these lesions are in a problematic area, low-dose radiation
might be an option, but generally this is not an indication for systemic therapy, and it can
simply be monitored.
1. Ugo V, Leporrier N, Salaun V, et al. Deciphering leukemic B-cell chronic lymphoproliferative disorders.
Leuk Lymphoma. 2006;47:2088-2095.
2.   Colburn DE, Welch MA, Giles FJ. Skin infiltration with chronic lymphocytic leukemia is consistent with a
good prognosis. Hematology. 2002;7:187-188.
3.  Cerroni L, Zenahlik P, Hfler G, et al. Specific cutaneous infiltrates of B-cell chronic lymphocytic
leukemia: a clinicopathologic and prognostic study of 42 patients. Am J Surg Pathol. 1996;20:1000-1010.

Dr. OBrien receives research support from Genentech.

Other B-cell lymphoproliferative disorders are usually CD5-negative, and the ones
that could potentially overlap with CLL are prolymphocytic leukemia (PLL), marginalzone lymphoma/leukemia (MZL), and follicular lymphoma in a leukemic phase. Again,
morphologies differ between all these diagnoses, but there can be overlap. The most
common overlap is between CLL and MZL. Thats because classically the diagnosis of
marginal zone is, as the name implies, based on pathologic examination of lymph nodes.
Patients presenting with leukemic disease without significant adenopathy render this
diagnosis more difficult to ascertain. A group of investigators in France evaluated 165
leukemic patients with B-cell lymphoproliferative disorders selected on the basis of a
score of 3 in the Royal Marsden Hospital scoring system, which implies atypical cases
that dont fit clearly into one diagnostic entity.1 Fifty-four cases were CD5-negative.
Morphological and phenotypical examinations were able to reveal the diagnosis in 24 of
these, including the most common diagnosis of MZL in 11 cases. Thirty CD5-negative cases
still could not be fully characterized after morphological and phenotypical examination.
Only five of these patients had superficial lymphadenopathy, and in three cases where
lymph node biopsy was done, the diagnosis of MZL was established. All in all, 14 were
diagnosed as MZL, which was the most common diagnosis, and eight patients remained
unclassified. These are likely the patients we would call atypical CLL.

M A L I G N A N T H E M A T O L O G Y 5

The Hematologist Compendium 2010-2015

Approach to Patient with a Bladder Tumor with Severe Hematuria and a


Hemoglobin Level of Eight with Associated Dyspnea July/August 2010
UPDATE/COMMENTARY
I would like to re-state the concept that hemorrhage occurs within Virchows triad.
To manage bleeding, one must identify and rank the pathophysiological factors
affecting vascular integrity, blood flow, and hemostasis.
1) The most important cause of painless gross hematuria in anyone, even when it
is self-limited, is vascular injury from bladder cancer. In a patient without known
transitional cell carcinoma with painless gross hematuria, one must direct all
interventions at getting tissue, and this usually begins with cystoscopy. Between
10 and 20 percent of these patients will be diagnosed with bladder cancer.U1
2) Local hyperfibrinolysis contributes to hematuria in patients with urothelial tumors,
but systemic hyperfibrinolysis is unlikely in the absence of disseminated cancer.
Recent data from liver transplant recipients indicate that thromboelastometry is
two to four times more sensitive than thromboelastography (TEG) for identifying
a systemic fibrinolytic state. TEG was 99 percent specific but only 23 percent
sensitive, so it cannot be used to rule out systemic hyperfibrinolysis.U2
3) Intractable hematuria from almost any cause with or without systemic
hyperfibrinolysis should be treated with systemic antifibrinolytic therapy plus
local measures, which could include intravesical instillation with formalin, alum,

MICHAEL H. KROLL, MD
Professor of Medicine, University of Texas MD Anderson Cancer Center, Houston, TX

(Editors Note: The original question was submitted to Dr. Kroll through Consult
a Colleague. He expanded his answer for print.)

The Question
A 78-year-old male with a bladder tumor had severe hematuria and a hemoglobin level of
8 g/dL with associated dyspnea. Urologists were unable to evaluate the tumor secondary
to severe bleeding in the bladder. Coagulation testing revealed a normal prothrombin time
(PT) of 10 seconds, but his activated partial thromboplastin time (aPTT) was elevated at
108 seconds, lowering to 56 seconds when mixed with normal plasma. Levels of coagulation
factors VIII, IX, X, XI, and XII were normal, although the report noted the possibility
of an inhibitor. Von Willebrand factor (VWF) studies were normal, as were assays for
anticardiolipin antibodies (ACA) and lupus anticoagulant (LA). The patient continues to
bleed and his hemoglobin continues to drop. Should any other work-up be done? Given that
he had no benefit from fresh frozen plasma infusions, should I give factor VII complex?

My Response
Bleeding is impaired hemostasis and therefore can be evaluated from the same perspective
that Virchow organized for thrombosis as dysfunction within the triad of blood (mainly
soluble coagulation factors, fibrinolytic apparatus, or platelets), blood vessel (mainly its
structural integrity or permeability), and/or blood flow (mainly the effects of ischemia or
hyperperfusion). In most cases of bleeding, a simple, single explanation emerges. But in
more complicated cases, like the one in question, I try to identify and rank the hemostatic
elements that have broken down.
When I see bleeding from the bladder, I focus first on tumor-associated hemorrhage. This
is because one must immediately direct therapy toward the cancer, if the fundamental
pathophysiological element is tumor-erosive hemorrhage. But how does one establish the
diagnosis if hematuria is so severe as to prevent visualization and biopsy of the bladder
mucosa? I would retreat slightly and look for disturbed and perhaps related hemostatic
elements that could be reversed and thereby permit proper evaluation of the root cause of
the hematuria.
When there is severe hematuria, perhaps related to transitional cell carcinoma, I ask the
question: Is there hyperfibrinolysis? The malignant urothelium is rich in urokinase-type
plasminogen activator,1 and local and sometimes systemic hyperfibrinolysis develops
and contributes to bleeding in patients with bladder cancer. Systemic hyperfibrinolysis
is not apparent by examining the PT and aPTT, but always results in elevated fibrin split
products and d-dimers. It can be identified by TEG or by measuring the euglobulin lysis
time. Neither test is available in every clinical laboratory, but in our institution we perform
TEG. When there is hyperfibrinolysis, or anything else that de-stabilizes polymerized
fibrin, the TEG tracing over 30 to 60 minutes shows a decline in the clot tensile strength
the plateau begins to decay or decline.2 When hyperfibrinolysis is identified, one can try to
control it with an antifibrinolytic drug such as aminocaproic acid or tranexamic acid.3 The
main risk of these treatments in someone with hematuria is ureteral obstruction if blood
refluxes and clots in a ureter, so one must monitor urine output and serum creatinine
concentration closely.

M A L I G N A N T H E M A T O L O G Y 6

or prostaglandin PGF2 (best for low flow bleeding); arterial embolization;


or hydrostatic pressure or urinary diversion (as a last resort). For patients
with bladder cancer, hypofractionated radiotherapy or chemoperfusion with
mitoxantrone often controls bleeding.U3
4) Today, more than five and a half years since this case was first reviewed, there
are no new data that recombinant activated factor VII (rFVIIa) should be used
for massive hematuria from any condition other than acquired hemophilia or
inherited hemophilia with an inhibitor. The therapeutic index of rFVIIa as a
nonspecific hemostatic agent is unfavorable.U4
U1. Nielsen M, Qaseem A, High Value Care Task Force of the American College of Physicians.
Hematuria as a marker of occult urinary cancer: advice for high-value care from the American
College of Physicians. Ann Intern Med. 2016;164:488-497.
U2. Abuelkasem E, Lu S, Tanaka K, et al. Comparison between thromboelastography and
thromboelastometry in hyperfibrinolysis detection during adult liver transplantation. Br J
Anaesth. 2016;116:507-512.
U3. Abt D, Bywater M, Engeler DS, et al. Therapeutic options for intractable hematuria in
advanced bladder cancer. Int J Urol. 2013;20:651-660.
U4. Levi M, Levy JH, Andersen HF, et al. Safety of recombinant activated factor VII in randomized
clinical trials. N Engl J Med. 2010;363:1791-1800.

In this patient, there may be a weak coagulation factor VIII inhibitor. The isolated elevated
aPTT must reflect a deficiency of, or an inhibitor to, either factor XII, XI, IX, or VIII and
the incomplete aPTT correction with mixing (from 108 to 56 seconds) indicates that
there is an inhibitor. The most commonly seen inhibitor is a lupus anticoagulant, which
is an antiphospholipid antibody that prolongs an in vitro clotting assay, which, in this
case, would be the aPTT. But lupus anticoagulants are not associated with bleeding
except in the rare case where they have antiprothrombin activity4 and this patients
anticardiolipin antibody assay and lupus anticoagulant screen were normal. So the most
parsimonious explanation for this case of bleeding and abnormal coagulation studies is
the presence of the most common acquired inhibitor encountered in a bleeding patient:
an acquired factor VIII inhibitor. It can emerge in patients with solid tumors,5 including
bladder cancer.6 It almost always causes clinically significant bleeding that can be
controlled with a pharmacological intervention such as rFVIIa.7 And the inhibitor itself (an
auto-antibody to factor VIII that causes rapid clearance) is often reduced or eradicated
with immunosuppressive therapy.8 To evaluate for this possibility, I suggest repeating the
factor VIII assay and measuring the inhibitor titer in Bethesda units, which is the inverse
of the dilution of the patients plasma that decreases factor VIII activity in control plasma
by 50 percent. My speculation is that when this patients factor VIII level is repeated, it will
be borderline low or low and that he will have an inhibitor level < 5 units. If an inhibitor
is identified, I would administer rFVIIa, although some would first try human factor VIII in
cases of low titer acquired factor VIII inhibitors.8
Should one use rFVIIa under any circumstances? Assuming that the PT and platelet
count remain satisfactory, I would first try bladder irrigation. If this or other urological
manuevers dont work and everyone remains mystified by the pathophysiology, I would
begin aminocaproic acid at an infusion rate of 1 gm/h. If aminocaproic acid doesnt work, I
would then add rFVIIa, but only at 20 to 30 mcg/kg and beginning at four- or even six-hour
intervals.9 This dose is prohemostatic but probably causes fewer thromboses than the
conventional dose of 90 mcg/kg every two hours.10
1. Dubeau L, Jones PA, Rideout III WM, et al. Differential regulation of plasminogen activators by epidermal
growth factor in normal and neoplastic human urothelium. Cancer Res. 1988;48:5552-5556.
2. Traverso CI, Caprini JA, Arcelus JI. The normal thromboelastogram amd its interpretation. Semin
Thromb Hemost. 1995;21:7-13.
3. Mannucci PM. Hemostatic drugs. New Eng J Med. 1998;339:245-253.
4. Galli M, Barbui T. Antiprothrombin antibodies: detection and clinical significance in the
antiphospholipid syndrome. Blood. 1999;93:2149-2157.
5. Hauser I, Lechner K. Solid tumors and factor VIII antibodies. Thromb Haemost. 1999;82:1005-1007.
6. Kreuter M, Retzlaff S, Enser-Weis U, et al. Acquired haemophilia in a patient with gram-negative
urosepsis and bladder cancer. Haemophilia. 2005;11:181-185.
7. Huth-Khne A, Baudo F, Collins P, et al. International recommendations on the diagnosis and treatment
of patients with acquired hemophilia A. Haematologica. 2009;94:566-575.
8. Franchini M, Lippi G. Acquired factor VIII inhibitors. Blood. 2008;112:250-255.
9. OConnell NM, Perry DJ, Hodgson AJ, et al. Recombinant FVIIa in the management of uncontrolled
hemorrhage. Transfusion. 2003;43:1711-1716.
10. Mayer SA, Brun NC, Begtrup K, et al. Efficacy and safety of recombinant activated factor VII for acute
intracerebral hemorrhage. N Engl J Med. 2008;358:2127-2137.

Dr. Kroll is part of Eisais Speakers Bureau.

The Hematologist Compendium 2010-2015

Recommendation of CHOP-Rituximab/More Intensive Regimen Such as DoseAdjusted EPOCH-Rituximab for Cyclin D1 Expression in DLBCL March/April 2011
UPDATE/COMMENTARY
Since this article was originally published in 2011, limited additional data have
emerged on diffuse large B-cell lymphoma (DLBCL) expressing cyclin D1. Although
most of these data have, as before, been anecdotal, an international consortium study
of patients with DLBCL treated with the rituximab, cyclophosphamide, doxorubicin,
vincristine, prednisone (R-CHOP) regimen, identified cyclin D1 expression in 30
patients of a total of 1,435 included in the study (2.1%). None of these patients had
rearrangements of the CCND1 gene and gene expression profiling showed them
to be roughly equally distributed between the germinal center B-cell (GCB) and
activated B-cell (ABC) subtypes. In terms of clinical behavior, patients with cyclin
D1 expression were predominantly male, and were significantly younger than those
without cyclin D1. No differences were observed in clinical behavior or overall and
progression-free survival between those with and without cyclin D1 expression,
suggesting that R-CHOP should remain the standard of care in this group.U1
With respect to the broader question of the clinical utility of molecular subtyping of
DLBCL, data from the phase I/II study of ibrutinib have demonstrated an encouraging
response rate, largely restricted to patients with the ABC subtype, which is known
to overexpress Brutons tyrosine kinase.U2 This observation is now being tested
in a randomized clinical trial in untreated DLBCL, comparing R-CHOP alone with

R-CHOP plus ibrutinib. The importance of evaluating these preliminary observations


in randomized trials is underlined by recent results from two studies addressing the
inclusion of bortezomib in first-line therapy of DLBCL. Despite the early encouraging
results cited in the original article, two recent randomized studies have failed to
show a benefit to the inclusion of bortezomib in patients with ABC-type diffuse large
B-cell lymphoma.U3,U4 We are still waiting on mature results from a third trial from the
United Kingdom, which has now closed.
U1. Ok CY, Xu-Monette ZY, Tzankov A, et al. Prevalence and clinical implications of cyclin D1
expression in diffuse large B-cell lymphoma (DLBCL) treated with immunochemotherapy:
a report from the International DLBCL Rituximab-CHOP Consortium Program. Cancer.
2014;120:1818-1829.
U2. Wilson WH, Young RM, Schmitz R, et al. Targeting B cell receptor signaling with ibrutinib in
diffuse large B cell lymphoma. Nat Med. 2015;21:922-926.
U3. Offner F, Samoilova O, Osmanov E, et al. Frontline rituximab, cyclophosphamide, doxorubicin,
and prednisone with bortezomib (VR-CAP) or vincristine (R-CHOP) for non-GCB DLBCL. Blood.
2015;126:1893-1901.
U4. Leonard JP, Kolibaba K, Reeves JA, et al. Randomized phase 2 open label study of R-CHOP
bortezomib in patients (pts) with untreated non-germinal center (non-GCB) subtype diffuse
large B-cell lymphoma (DLBCL): results from the Pyramid Trial (NCT 00931918). Blood.
2015;126:811.

and overall survival rates independent of the IPI. This difference in prognosis is seen for
patients treated both with CHOP and with CHOP-rituximab.3
JOHN W. SWEETENHAM, MD, FRCP, FACP
Professor of Medicine, Senior Director of Clinical Affairs, and Executive Medical Director,
Huntsman Cancer Institute, University of Utah, Salt Lake City, UT

The Question
A previously healthy 72-year-old male presented with a new diagnosis of clinical stage
IVB DLBCL, with maxillary sinus involvement and involvement of multiple bony sites.
His ECOG performance status was 0 and his International Prognostic Index (IPI) score
was 3 based on his age, clinical stage, and involvement of multiple extranodal sites, all of
which are adverse factors in the IPI. A biopsy from the maxillary sinus confirmed DLBCL
by morphology and was positive for cyclin D1. Since cyclin D1 expression in DLBCL is
reported to have a poor prognosis, should this patient be treated with R-CHOP or should
he receive a more intensive regimen such as dose-adjusted EPOCH-rituximab?

My Response
This clinical case raises several important questions regarding the management of
advanced DLBCL, partly related to diagnostic criteria and to the optimal treatment
regimen, but also regarding the broader issue of whether different immunophenotypic or
molecular subtypes of DLBCL should be treated differently.
Cyclin D1 is a nuclear protein with a well-defined role as a regulator of cell cycle
progression from G1. Its deregulation is central to the pathogenesis of some B-cell
neoplasms, especially mantle cell lymphoma (MCL), where it is over-expressed in about
90 percent of cases, as well as in myeloma and hairy cell leukemia. In the vast majority of
MCLs, cyclin D1 over-expression is associated with the t(11;14), whereas this association
is much less common in other B-cell neoplasms.1
Reports of the expression of cyclin D1 in DLBCL have been conflicting, partly because
different methodologies are used for detection of cyclin D1 and partly because the
morphologic distinction between DLBCL and the blastoid variant of MCL can be
problematic, particularly since other immunophenotypic characteristics of DLBCL and
MCL can sometimes overlap. Despite these challenges, there are several case reports
and some larger series that describe the presence of cyclin D1 in cases of DLBCL that are
negative for CD5 but have other classical markers of DLBCL, such as bcl-6 and MUM-1.2
As indicated by the questioner, there is a suggestion from the limited literature that
patients with cyclin D1-positive DLBCL have a worse prognosis. In the largest published
series, nine of 10 patients had died from progressive disease after a median of 29 months
from diagnosis.2 However, these data are anecdotal, and if this represents a true clinicopathologic entity, there are insufficient data to indicate whether it has a meaningfully
different prognosis from other cases of DLBCL.
In contrast, other immunophenotypic and molecular features of DLBCL have been shown
to have defined predictive and prognostic value. Examples include the adverse prognostic
effect of bcl-2 expression and lack of bcl-6 expression, both of which can apparently be
overcome by the addition of rituximab to standard chemotherapy regimens such as CHOP.
Gene expression profiling studies have identified two major molecular subtypes of DLBCL;
one has a gene expression profile (GEP) consistent with GCB (GCB-like), and one has a
profile consistent with ABC (ABC-like). Molecular subtyping by GEP has also been shown
to have prognostic value, as patients with ABC-like GEPs have poorer progression-free

M A L I G N A N T H E M A T O L O G Y 7

The clinical utility of these findings has been limited by the lack of routine availability
of suitable tissue and resources for conducting GEP studies. As a result, several groups
have explored the use of immunohistochemical (IHC) surrogates to assign GCB and ABC
subtypes.4 Since the correlation between cell of origin identified by GEP and IHC is high, these
algorithms are gaining widespread use as a method of risk stratification in clinical trials.
Although the prognostic significance of cell of origin (GCB versus ABC) in DLBCL
is well established, the question of whether this or any other biologic predictor of
outcome should be used to direct therapy for DLBCL is unknown. The dose-adjusted
EPOCH-R regimen mentioned by the questioner uses infusional drug scheduling and
pharmacodynamic dosing based on hematologic toxicity to exploit tumor proliferation
as a target mechanism. Phase II studies of this regimen have shown impressive results
and, interestingly, no difference in progression-free or overall survival according to GCB
or non-GCB cell of origin.5 As a result, this regimen is now being compared directly with
CHOP-rituximab in a randomized, prospective intergroup study led by the CALGB, which
includes an analysis of cell of origin by GEP. The results of this study will not be available
for several years, so for now, CHOP-rituximab remains the standard regimen for first line
therapy of DLBCL, irrespective of cell of origin or expression of other biomarkers.
Early results from some recent studies suggest that this is likely to change. Two recent
studies have suggested that the addition of bortezomib to standard chemotherapyrituximab combinations for DLBCL produces improvements in response rates and
progression-free survival rates, which may be restricted to patients with the ABC
subtype.6,7 Additionally, data are emerging to suggest that some recently identified
therapeutic targets such as Brutons tyrosine kinase (Btk) are differentially expressed in
ABC- versus GCB-like DLBCL, providing a rationale for limiting trials of these agents to
those patients with specific molecular subtypes.
Future progress in the treatment of DLBCL is likely to emerge from further
characterization of specific molecular targets. In the meantime, CHOP-rituximab should be
regarded as standard, first-line therapy for all patients with DLBCL outside clinical trials.
1. Swerdlow SH, Zukerberg LR, Yang WI, et al. The morphologic spectrum of non-Hodgkins lymphomas
with BCL1/cyclin D1 gene rearrangements. Am J Surg Pathol. 1996;20:627-640.
2. Ehinger M, Linderoth J, Christensson B, et al. A subset of CD5- diffuse large B-cell lymphomas expresses
nuclear cyclin D1 with aberrations at the CCND1 locus. Am J Clin Pathol. 2008;129:630-638.
3. Lenz G, Wright G, Dave SS, et al. Stromal gene signatures in large B-cell lymphomas. N Engl J Med.
2008;359:2313-2323.
4. Meyer PN, Fu K, Greiner TC, et al. Immunohistochemical methods for predicting cell of origin
and survival in patients with diffuse large B-cell lymphoma treated with rituximab. J Clin Oncol.
2011;29:200-207.
5. Wilson WH, Dunleavy K, Pittaluga S, et al. Phase II study of dose-adjusted EPOCH and rituximab in
untreated diffuse large B-cell lymphoma with analysis of germinal center and post-germinal center
biomarkers. J Clin Oncol. 2008;26:2717-2724.
6. Dunleavy K, Pittaluga S, Czuczman MS, et al. Differential efficacy of bortezomib plus chemotherapy
within molecular subtypes of diffuse large B-cell lymphoma. Blood. 2009;113:6069-6076.
7. Ruan J, Martin P, Furman RR, et al. Bortezomib plus CHOP-rituximab for previously untreated diffuse
large B-cell lymphoma and mantle cell lymphoma. J Clin Oncol. 2010. 2011;29:690-697.

Dr. Sweetenham indicated no relevant conflicts of interest.

The Hematologist Compendium 2010-2015

FDA Approval of the JAK Inhibitor Ruxolitinib and the Management of


Patients with Myelofibrosis March/April 2012
UPDATE/COMMENTARY
Five-year follow-up data are now available for the COMFORT-I and II trials of
ruxolitinib.U1,U2 In COMFORT-II, 53 percent of patients treated with ruxolitinib achieved
a 35 percent reduction in spleen volume from baseline at any time during treatment.
U1
Improvement in spleen size was maintained with long-term therapy, with the median
duration of maintenance of spleen volume reduction being 3.2 years. The KaplanMeier probability of maintaining this reduction was 51 percent at three years and 48
percent at five years. Median OS was not reached in the ruxolitinib arm and was 4.1
years in the best available therapy (BAT) arm. There was a 33 percent reduction in
risk of death with ruxolitinib compared with BAT (HR, 0.67; 95% CI, 0.44-1.02; p = .06).
The probability of survival at five years was 56 percent with ruxolitinib and 44 percent
with BAT. There were no new or unexpected adverse effects (AEs) with long-term
ruxolitinib exposure. The most commonly reported AEs in patients who received
ruxolitinib any time were thrombocytopenia (52%), anemia (49%), diarrhea (36%), and
peripheral edema (33%); grade 3/4 AEs included anemia (23%), thrombocytopenia
(15%), and pneumonia (6%). There were similar rates of evolution to leukemia between
the ruxolitinib (5.5%) and BAT (6.8%) arms. The long-term efficacy and safety results
of COMFORT-1U2 are very similar to COMFORT-II, and were presented at the ASCO
annual meeting in June 2016 at the time of this writing.U2
Ruxolitinib has immunosuppressive properties that are in part related to impairment
of dendritic cell, NK-cell, and T-cell function.U3-5 Atypical and/or opportunistic
infections have been observed in MF patients treated with ruxolitinib, including
reactivation of tuberculosis,U6 viral hepatitis,U7 and herpes zoster. Additionally,
there have been case reports of toxoplasmosis and cytomegalovirus retinitis,U8,9
as well as pneumocystis and fungal pneumonias. Although there are no consensus
guidelines regarding use of anti-infective prophylaxis (nor is the live zoster vaccine
recommended), treating physicians should perform risk assessments for these
infections in patients before commencing ruxolitinib, with tailored laboratory
screening where appropriate.
Phase III clinical development of another JAK inhibitor, fedratinib, was discontinued
because of the development of several cases of Wernickes encephalopathy. In
vitro studies indicate that oral absorption of dietary thiamine update is inhibited
by fedratinib, which blocks the human thiamine transporter.U10 Inhibition of the
thiamine transporter does not appear to be an issue with the other JAK inhibitors
currently being tested. Another JAK inhibitor, pacritinib, has been evaluated in two
phase III clinical trials vs. BAT: PERSIST-1 (JAK inhibitor-naive MF patients) and
PERSIST-2 (prior JAK2 inhibitors treatment allowed and platelet count < 100 109/L
required). In the PERSIST-1 trial, pacritinib produced reductions of spleen volume
and symptoms and a 26 percent rate of red blood cell transfusion-independence
with otherwise minimal myelosuppression.U11 However, in February 2016, the FDA
placed a clinical hold on pacritinib due to an interim analysis of the PERSIST-2 trial
which demonstrated a detrimental effect on survival in the ruxolitinib compared
to BAT arm. Specifically, a relatively higher rate of intracranial hemorrhage,
cardiac failure, and cardiac arrest was observed with ruxolitinib therapy.U11
Momelotinib remains the only other JAK inhibitor currently in phase III clinical
trial testing, with results expected soon from the SIMPLIFY-1 trial (momelotinib vs.
ruxolitinib; 1:1 randomization; no prior JAK inhibitor therapy) and SIMPLIFY-2 trial
(momelotinib vs. BAT; 2:1 randomization; a key inclusion criterion is anemia and/or
thrombocytopenia with prior ruxolitinib treatment).
Although a limited proportion of patients treated with ruxolitinib or the other
JAK inhibitors may demonstrate a reduction in bone marrow fibrosis and/or JAK2
V617F allele burden over time,U1 disappearance of fibrosis or complete molecular

JASON R. GOTLIB, MD, MS


Associate Professor of Medicine (Hematology) and Director, Hematology Fellowship Program,
Stanford University School of Medicine and Stanford Cancer Institute, Stanford, CA

The Question
How does the recent FDA approval of the JAK inhibitor ruxolitinib influence your
management of patients with myelofibrosis?

My Response
Myelofibrosis (primary and post-PV/ET MF) is a Philadelphia chromosome-negative
myeloproliferative neoplasm with a natural history characterized by progressive anemia,
spleen enlargement due to extramedullary hematopoiesis, and potential for evolution
to acute myeloid leukemia (AML). Impairment of quality of life is due to both massive
splenomegaly (e.g., early satiety and abdominal discomfort) and inflammatory cytokines,
which mediate debilitating symptoms such as night sweats, fevers, muscle/bone pain, and
M A L I G N A N T H E M A T O L O G Y 8

remissions is exceedingly rare. More rigorous data are needed to establish the
long-term effects of JAK inhibitors on marrow fibrosis improvement or stabilization
compared to conventional therapies. Meanwhile, phase I/II trials of ruxolitinib
in combination with other therapies with different mechanisms of action are
being undertaken to assess whether response rates and/or drug-related anemia
can be improved. Such medications include immunomodulatory agents (e.g.,
lenalidomide; pomalidomide); hypomethylating agents (e.g., azacitidine/decitabine);
phosphoinositide-3 kinase inhibitors (e.g., buparlisib; idelalisib); histone deacetylase
inhibitors (e.g., panobinostat); hedgehog pathway inhibitors (e.g., sonidegib);
and agents to stimulate erythropoiesis (erythropoietin; danazol).U12 Phase II
monotherapy trials of antifibrotics (e.g., PRM-151) and the telomerase inhibition
(imetelstat) are in progress in intermediate or high risk MF patients who did not
respond to, or did not tolerate ruxolitinib. Type II JAK inhibitors, which only inhibit
mutant JAK2 (e.g., CHZ868), are in pre-clinical development and may offer the hope
of more in-depth responses since currently available JAK inhibitors inhibit both wildtype and mutant JAK2.U13 Lastly, the role of ruxolitinib in the pre-and post-transplant
setting is the subject of several ongoing clinical trials.
U1. Harrison CN, Vannucchi AM, Kiladjian JJ, et al. Long-term efficacy and safety in COMFORT-II,
a phase 3 study comparing ruxolitinib with best available therapy for the treatment of
myelofibrosis: 5-year final study results. Blood. 2015;126:59.
U2. Gupta V, Verstovsek S, Mesa RA, et al. Long-term outcomes of ruxolitinib therapy in patients
with myelofibrosis: 5-year update from COMFORT-I. ASCO Meeting Abstracts. 2016:7012.
U3. Heine A, Held SA, Daecke SN, et al. The JAK-inhibitor ruxolitinib impairs dendritic cell function
in vitro and in vivo. Blood. 2013;122:1192-1202.
U4. Schnberg K, Rudolph J, Vonnahme M, et al. JAK Inhibition impairs NK cell function in
myeloproliferative neoplasms. Cancer Res. 2015;75:2187-2199.
U5. Parampalli Yajnanarayana S, Stbig T, Cornez I, et al. JAK1/2 inhibition impairs T cell function
invitro and in patients with myeloproliferative neoplasms. Br J Haematol. 2015;169:824-833.
U6. Hopman RK, Lawrence SJ, Oh ST. Disseminated tuberculosis associated with ruxolitinib.
Leukemia. 2014;28:1750-1751.
U7. Caocci G, Murgia F, Podda L, et al. Reactivation of hepatitis B virus infection following
ruxolitinib treatment in a patient with myelofibrosis. Leukemia. 2014;28:225-227
U8. Goldberg RA, Reichel E, Oshry LJ. Bilateral toxoplasmosis retinitis associated with ruxolitinib.
New Engl J Med. 2013;369:681-683.
U9. von Hofsten J, Johnsson Forsberg M, Zetterberg M. Cytomegalovirus retinitis in a patient who
received ruxolitinib. New Engl J Med. 2016;374:296-297.
U10. Zhang Q, Zhang Y, Diamond S, et al. The Janus kinase 2 inhibitor fedratinib inhibits thiamine
uptake: a putative mechanism for the onset of Wernickes encephalopathy. Drug Metab Dispos.
2014;42:1656-1662.
U11. Mesa RA, Egyed M, Szoke A, et al. Results of the PERSIST-1 phase III study of pacritinib (PAC)
versus best available therapy (BAT) in primary myelofibrosis (PMF), post-polycythemia vera
myelofibrosis (PPV-MF), or post-essential thrombocythemia-myelofibrosis (PET-MF). ASCO.
2015:LBA7006
U12. Mascarenhas J. Looking forward: novel therapeutic approaches in chronic and advanced

phases of myelofibrosis. Hematology Am Soc Hematol Educ Program. 2015;2015:329-339.

U13. Meyer SC, Keller MD, Chiu S, et al. CHZ868, a type II JAK2 inhibitor, reverses type I JAK

inhibitor persistence and demonstrates efficacy in myeloproliferative neoplasms. Cancer Cell.


2015;28:15-28.

Dr. Gotlib has received (or is pending) research funding from Incyte
(manufacturer of ruxolitinib), Sanofi-Aventis (fedratinib), Gilead (momelotinib;
idelalisib), Promedior (PRM-151), and imetelstat (Janssen).

cachexia. The activating JAK2 V617F mutation is present in 50 to 60 percent of patients,


and the MPL mutation (W515L/K), resulting in ligand-independent activation of the
thrombopoietin receptor, is identified in an additional 5 to 10 percent of individuals. It has
become abundantly clear, however, that MPNs such as MF are more genetically complex.
Molecular alterations in additional genes and dysregulation of the epigenetic machinery also
contribute to disease pathogenesis.
Prognostic scoring systems based on clinical and laboratory factors obtained either at the
time of diagnosis (IPSS)1 or during the disease course (dynamic IPSS, or DIPSS)2 have been
developed in order to estimate both overall survival and risk of progression to AML. The IPSS
uses five adverse prognostic factors: age >65, hemoglobin <10 g/dL, white blood cell count
>25,000/mm3, constitutional symptoms, and peripheral blood blasts >1 percent. The DIPSS-Plus
refines prognosis assessment by incorporating three additional adverse risk factors: platelet
count <100,000/mm3, the need for red blood cell transfusions, and poor-risk cytogenetics.
Using the IPSS as an example, patients can be stratified into one of four risk groups: low (score
0), intermediate-1 (score 1), intermediate-2 (score 2), or high (score >3), with a median overall
survival among the groups ranging from approximately 11 years to just over two years.1
Age, performance status, and prognostic risk group drive decision making about treatment
options. The Table illustrates this point using three patients. Patient 1 is a low-risk patient
with excellent performance status, minimally abnormal blood counts and splenomegaly,
The Hematologist Compendium 2010-2015

and no constitutional symptoms. Such patients do not warrant immediate treatment and a
watch-and-wait approach may be undertaken. At the other end of the spectrum, Patient 3 is
a younger, high-risk patient characterized by abnormal blood counts, marked splenomegaly,
and poor-risk cytogenetics. The DIPSS-Plus estimate of overall survival is 16 months,
and the five- and 10-year leukemia rates are 18 and 31 percent, respectively.2 Given this
patients younger age and relatively poor prognosis, evaluation for a potentially curative
myeloablative hematopoietic stem cell transplant would be encouraged.
For patients who require treatment and are not candidates for transplantation, available
therapies for MF-related cytopenias, splenomegaly, and symptoms are considered
palliative. These options have included chemotherapy such as hydroxyurea; erythropoiesisstimulating agents; immunomodulatory drugs such as thalidomide or lenalidomide, with or
without corticosteroids; splenectomy; splenic irradiation; and clinical trials. In November
2011, only four years after commencing clinical trial evaluation, ruxolitinib became the first
JAK inhibitor approved by the FDA for MF patients (intermediate- and high-risk).
The registration trials for ruxolitinib consisted of two large phase III trials: COMFORT-I
was a randomized (1:1), double-blind, multicenter study comparing ruxolitinib 15 or 20
mg twice daily (dose stratified according to baseline platelet count) versus placebo,3 and
COMFORT-II was a randomized (2:1), open-label, multicenter trial comparing ruxolitinib 15
or 20 mg bid versus BAT; investigator-selected including no treatment).4 Both trials met
the primary endpoint of the percentage of ruxolitinib versus control patients achieving >
35 percent reduction in spleen volume at week 24 (COMFORT-I: 41.9% vs. 0.7%) and week
48 (COMFORT-II: 28.5% vs. 0%). After 24 weeks in the COMFORT-I trial, the proportion of
patients with 50 percent improvement in total symptom score (using the myelofibrosis
symptom assessment form) was 45.9 percent versus 5.3 percent (ruxolitinib vs. placebo,
p<0.0001). Anemia and thrombocytopenia were common ruxolitinib-related adverse
events but rarely led to drug discontinuation, and the drug was otherwise well tolerated.
In an updated analysis of COMFORT-I, there was a significant overall survival benefit with
ruxolitinib; at a median follow-up of 51 weeks, there were 13 (8.4%) deaths in the ruxolitinib
group and 24 (15.7%) deaths in the placebo arm.5 The implications of these short-term data
are unclear since JAK inhibitors exert modest or no impact on fundamental disease-related
features such as JAK2 mutant allele burden or marrow fibrosis.
Ruxolitinibs potency as a spleen shrinker and symptom mitigator is shared by other JAK
inhibitors currently in clinical trials (e.g., fedratinib [formerly SAR302503 and TG101308],
momelotinib [formerly CYT387], pacritinib [formerly SB1518], etc.). Patients with or
without the JAK2 V617F mutation respond similarly. Therefore, first-line treatment with
a JAK inhibitor would be an ideal choice for Patient 2 described in the table and could be
considered a bridging option for Patient 3 until a transplant is performed.
In an ad hoc analysis of the COMFORT-I and COMFORT-II trials, worsening of spleen size
as well as symptoms and quality-of-life scores were similar between the placebo and BAT
control groups.6 Although it may be premature to abandon conventional treatments such as
hydroxyurea, the comparative superiority of ruxolitinib in these trials justifies its frontline
use for intermediate- and high-risk MF patients in whom the primary goal is improvement
of splenomegaly and constitutional symptoms. For MF patients with anemia or RBC
transfusion-dependence as the predominant clinical issue, no standard of care currently
exists. In this regard, differences among JAK inhibitors may prove informative in tailoring
particular agents to specific patient presentations. For example, the JAK 1/2 inhibitor
momelotinib (CYT387) has demonstrated improvements in hemoglobin/transfusiondependence in an ongoing phase II trial.7 Lenalidomide (or pomalidomide, on a trial basis)
may also elicit benefits in anemia in ~20 to 30 percent of MF patients.

Table. MF Patient Profiles


Age/Gender
ECOG Performance Status

Patient 1

Patient 2

Patient 3

63/F

61/M

48/M

WBC (/mm3)

9,200

18,700

31,000

Hb (g/dL)

11.1

10.5

8.2,
Transfusiondependent

155,000

218,000

85,000

Platelet Count (/mm3)


PB Blasts (%)
Constitutional Symptoms
Splenomegaly*
Cytogenetics
JAK2 V617F Mutation
IPSS Score/Risk Group

No

Yes

Yes

4 cm

15 cm

21 cm

Normal

Normal

Monosomy 5

Yes

No

Yes

0/Low

2/Intermediate-2

4/High

PB=peripheral blood
*below left costal margin by palpation

1. Cervantes F, Dupriez B, Pereira A, et al. New prognostic scoring system for primary myelofibrosis
based on a study of the International Working Group for Myelofibrosis Research and Treatment.
Blood. 2009;113:2895-2901.
2. Gangat N, Caramazza D, Vaidya R, et al. DIPSS plus: a refined dynamic International Prognostic Scoring
System for primary myelofibrosis that incorporates prognostic information from karyotype, platelet
count, and transfusion status. J Clin Oncol. 2011; 29:392-397.
3. Verstovsek S, Mesa RA, Gotlib JR, et al. Results of COMFORT-I, a randomized double-blind phase III
trial of JAK 1/2 inhibitor INCB18424 (424) versus placebo (PB) for patients with myelofibrosis (MF).
J Clin Oncol. ASCO Meeting Abstracts. 2011;29:6500.
4. Harrison CN, Kiladjian J, Al-Ali HK, et al. Results of a randomized study of the JAK inhibitor ruxolitinib
(INC424) versus best available therapy (BAT) in primary myelofibrosis (PMF), post-polycythemia
vera-myelofibrosis (PPV-MF) or post-essential thrombocythemia-myelofibrosis (PET-MF). J Clin Oncol.
ASCO Meeting Abstracts. 2011;29:LBA6501.
5. Verstovsek S, Mesa RA, Gotlib J, et al. Consistent benefit of ruxolitinib over placebo in spleen volume
reduction and symptom improvement across subgroups and overall survival advantage: results from
COMFORT-I. Blood. ASH Annual Meeting Abstracts. 2011;118: 278.
6. Mesa RA, Verstovsek S, Cervantes F, et al. Comparison of the efficacy of placebo and best available
therapy for the treatment of myelofibrosis in the COMFORT studies. Blood. ASH Annual Meeting
Abstracts. 2011;118:1753.
7. Pardanani A, Gotlib J, Gupta V, et al. An expanded multicenter phase I/II study of CYT387, a JAK- 1/2
inhibitor for the treatment of myelofibrosis. Blood. ASH Annual Meeting Abstracts. 2011;118:3849.
8. Bhagwat N, Koppikar P, Kilpivaara O, et al. Heterodimeric JAK-STAT activation as a mechanism of
persistence to JAK2 inhibitor therapy. Blood. ASH Annual Meeting Abstracts. 2011;118:122.

Dr. Gotlib receives research funding from Incyte (manufacturer of ruxolitinib), SanofiAventis (manufacturer of SAR302503), YMBiosciences (manufacturer of CYT387), and
Celgene (manufacturer of lenalidomide).

Given the rapid on/off action of JAK inhibitors, caution must be undertaken when stopping
these agents because of the potential for return of symptoms in a short period of time.
During therapy, disease persistence (incomplete regression or return of splenomegaly
and symptoms) can occur and may be partly explained biologically by reactivation
(phosphorylation) of JAK2 through heterodimerization with JAK family members JAK1 and
TYK2.8 Future directions will therefore be focused on combination trials of JAK inhibitors
with other targeted targets (e.g., histone deacetylase inhibitors, anti-fibrotics, PI3 kinase/
AKT inhibitors) to improve the quality and duration of responses.

New Open-Access Peer-Review Journal, Blood Advances


The new bi-weekly journal from ASH, Blood Advances, will make its debut at the 58th ASH
Annual Meeting in San Diego in December. Founding editor, Dr. Robert Negrin, describes
the journal as novel approaches for presentation of original content, including approaches
that can only be performed using electronic format, including enhanced graphics, video, and
discussion. This journal will complement ASHs flagship journal by publishing all new content
not covered by Blood. For more information, visit www.bloodadvances.org.

M A L I G N A N T H E M A T O L O G Y 9

The Hematologist Compendium 2010-2015

Evaluation of Young Adult with Bone Marrow Failure May/June 2012


UPDATE/COMMENTARY

UPDATE BY DARIA V. BABUSHOK, MD, PhD 1 AND TIMOTHY S. OLSON, MD, PhD 2
1. Instructor, Division of Hematology/Oncology, Department of Medicine, Hospital of the University of Pennsylvania; Comprehensive Bone Marrow Failure center, Division of Pediatric
Hematology, The Childrens Hospital of Philadelphia, Philadelphia, PA
2. Director, Comprehensive Bone Marrow Failure Center, Division of Pediatric Hematology, The Childrens Hospital of Philadelphia; Assistant Professor of Pediatrics, Perelman School
of Medicine, University of Pennsylvania; Blood and Marrow Transplant Section, Division of Pediatric Oncology, The Childrens Hospital of Philadelphia, Philadelphia, PA

With the increasing use of next-generation sequencing (NGS), our understanding


of the genetics underlying inherited bone marrow failure syndrome (IBMFS) and
predisposition to myelodysplastic syndrome (MDS) has grown significantly. The
numbers of genes linked to classic IBMF, such as Fanconi Anemia (FA; 17 genes),
Diamond Blackfan Anemia (DBA; 14 genes), and Dyskeratosis Congenita (DC; 11),
have increased. Additionally, throughout the last five years, five new genes (GATA2,
ANKRD26, SRP76, DDX41, and ETV6) have been linked to familial MDS and leukemia
(Figure 1). Importantly, it is increasingly recognized that these conditions can
initially present in adulthood. With the improved knowledge
of the inherited BMF and MDS, the differential diagnosis for a
Figure 2. Evaluation
young adult presenting with BMF or myelodysplasia should
be broadened to include not only the classical IBMFSs
such as DC and FA, but also the emerging familial MDS/
AML predisposition syndromes. Although the knowledge
of these disorders remains imperfect, we expect that our
diagnostic ability will improve as more genes and syndromes
are identified. Presently, the comprehensive evaluation of

a young adult with a suspected IBMFS or familial MDS/leukemia syndrome (Figure


2) continues to rely on functional testing of affected biological pathways to screen
for the classic polygenic IBMFSs DC and FA. With the ready availability of clinically
approved comprehensive NGS panels targeting genes known to cause IBMFS
and familial MDS/AML, these panels have become a useful and frequently more
economical alternative to sequential single-gene testing, particularly in cases in
which syndromic features suggestive of a specific diagnosis are not present.

of Bone Marrow Failure in Children and Young Adults

Figure 1. Timeline of Discovery of the Emerging


Inherited Myelodysplastic Syndrome/Acute Myeloid
Leukemia Predisposition Syndromes.

BEN OSHRINE, MD, 1 MONICA BESSLER, MD, PhD 2


1. Fellow in Hematology-Oncology, Childrens Hospital of Philadelphia,
Philadelphia, PA
2. Buck Family Professor in Hematology, Pediatric & Adult Comprehensive Bone
Marrow Failure Center, Childrens Hospital of Philadelphia and The Hospital of
the University of Pennsylvania, University of Pennsylvania School of Medicine,
Philadelphia, PA

The Case
The patient is a 29-year-old male who presented with a two-year history of progressive
pancytopenia (WBC 2.2 x 109/L, ANC 0.83 x 109/L; Hb 9.9 g/dL with 2.9% reticulocytes; platelet count 26 x 109/L). He was of short stature with discrete facial dysmorphisms and mildly
impaired cognitively. The family history was negative for BMF, but there was an extensive
history of early cancer on the maternal side. Bone marrow examination revealed marked
hypocellularity without dysplasia or excess blasts. The patient became transfusion-dependent, so an unrelated bone marrow transplant (BMT) had been recommended as definitive
treatment.

The Question
How do you evaluate a young adult with BMF? What features suggest an underlying IBMFS
as the etiology? How do you evaluate for these disorders, and why is it important?

Our Response
IBMFS is a collection of genetic disorders including FA, DC, DBA, Shwachman-Diamond
syndrome (SDS), severe congenital neutropenia, and congenital megakaryocytic
thrombocytopenia that are associated with insufficient blood cell production, congenital
anomalies, and, in some cases, an increased risk of malignancies including MDS, leukemia,
and certain solid tumors.1 IBMFS is not always inherited. Disease may also occur

M A L I G N A N T H E M A T O L O G Y 10

sporadically due to a de novo mutation, which may account for more than 50 percent
of all cases for some of the IBMFS (e.g., certain genetic forms of DBA or DC). Sporadic
appearance of disease may also be due to low penetrance of the causative mutation or
genetic anticipation in which clinical phenotype becomes more severe in successive
generations as observed in autosomal dominant DC. In this case, family members may
carry the mutation but not have clinical symptoms (silent mutation carriers). IBMFS
can affect one, two, or all blood cell lineages. Abnormal blood cell counts are frequently
not present at birth but develop during childhood, adolescence, or later in life.2 In some
cases, BMF never becomes clinically manifest, and the affected individual may seek
medical attention because of an unusually early onset of MDS, leukemia, cancer, or
other non-hematologic manifestations associated with the condition, such as pulmonary
fibrosis in individuals with DC.3 While once thought to be strictly the domain of pediatric
hematologists, with improved insights into the spectrum of disease expression and genetic
testing, IBMFS is increasingly being diagnosed and cared for in adult hematology clinics.4

When?
For all patients presenting with BMF, the possibility of an underlying IBMFS should be
considered. Such consideration is particularly relevant for patients in whom BMT is being
considered as treatment. IBMFS should also be considered in children and young adults
presenting with MDS or MDS/leukemia, as these clonal disorders are late complications
of IBMFS and may be the first symptom of disease in about 20 percent of cases, typically
in the young adult population.3 IBMFS, with disease onset in adults, often lack the classic
physical stigmata, such as abnormal thumbs (FA) or dystrophic fingernails (DC), and other
extra-hematopoietic manifestations, such as small stature or discrete facial anomalies,
may be subtle.4 Therefore, it is difficult to propose an age limit for the exclusion of an
IBMFS, as these disorders may be diagnosed even in old age. However, the therapeutic
consequences are probably most significant when diagnosed in pediatric and young adult
patients. The diagram above suggests the diagnostic genetic workup for the young adult
presenting with BMF.

The Hematologist Compendium 2010-2015

What?

Why?

Currently, there are more than 80 different genes that may cause IBMFS.5 With the
technology of whole-exome and whole-genome sequencing, the number of genetic
abnormalities associated with IBMFS is expected to increase dramatically, making it
impractical to perform testing for every gene in every affected individual. Many of the
mutated gene products fall into distinct biologic pathways (e.g., the FA DNA repair
pathway, the maintenance of telomeres in DC, or ribosome biogenesis in DBA). Some
clinically available screening tests investigate the integrity of the affected pathway
and, if abnormal, direct a focused and more economical approach to genetic testing.
Given the important therapeutic implications associated with diagnosis and the
availability of sensitive diagnostic tests, screening for FA and DC is recommended in
every patient presenting with BMF or early MDS/leukemia. Due to abnormal DNA repair,
chromosomes of dividing cells of patients with FA are hypersensitive to breakage induced
by cross-linking agents such as diepoxybutane (DEB) or mitomycin C (MMC), and this
characteristic is the basis of the diagnostic test for the disease.6 The DEB/MMC sensitivity
test uses patient lymphocytes in the analysis and is available in several major medical
centers with a specific interest in FA. Telomeres of nucleated peripheral blood cells are
excessively short in DC.7

The natural history, response to treatment, and surveillance requirements differ


significantly between patients with acquired BMF and IBMFS. Patients with IBMFS are
typically not responsive to immunosuppressive therapies that may be used for acquired
aplastic anemia. Furthermore, the presence of IBMFS requires special considerations
for BMT, both in terms of family member donor selection and choice of transplant
conditioning regimen, as conventional conditioning regimens can be excessively toxic or
fatal for some of the IBMFS.8 IBMFS may require surveillance for long-term complications
that may be accelerated by the choice of treatment (pulmonary fibrosis, osteopenia,
MDS/leukemia, and other non-hematologic malignancies). Finally, the diagnosis of IBMFS
requires informed genetic counseling for both the affected individual and the family.

Clinical telomere length measurements by PCR, flow cytometry, or Southern blotting


are available through several commercial laboratories. Results are usually provided as
percentile in comparison to normal controls. The interpretation of the clinical significance
of the measured telomere lengths can be difficult and often needs to be done in the context
of disease manifestations. In general, telomere lengths within the normal distribution
exclude DC to be the cause of BMF. In children and young adults with BMF caused by DC,
the telomere lengths of total peripheral blood lymphocytes are usually 2 to 3 standard
deviations below the telomere lengths distribution of age-matched healthy controls. Short
telomere lengths in granulocytes are less specific for DC and can be short in a number of
other IBMFS. As telomeres get shorter with age, telomere length becomes less diagnostic
for DC in older individuals. If one of these screening tests is abnormal, subsequent genetic
testing should be pursued, as only the identification of a disease-causing mutation can
confirm the diagnosis of an IBMFS. More specific tests for other IBMFS have to be chosen
on an individual basis depending on clinical presentation, family history, and laboratory
findings. For example, DBA may be considered in patients who present with macrocytic
anemia, low reticulocyte count, and a bone marrow characterized by an isolated erythroid
hypoplasia. An elevated erythrocyte adenosine deaminase activity may further support
a diagnosis of DBA. A history of symptoms of pancreatic insufficiency in a patient with
early-onset MDS may be suggestive for SDS. Genetic testing for mutations in the ribosomal
proteins most frequently associated with DBA and for mutations in SBDS (the gene that is
mutated in SDS) are available commercially. With wider applicability of novel sequencing
technologies, genetic testing for all major IBMFS genes using a single multiplex platform
is likely to become available in the not-too-distant future. However, with the increased
availability of genetic testing, the interpretation of results and consideration of their clinical
significance will become more complicated, requiring management by an experienced team
of physicians that includes geneticists, hematopathologists, and hematologists.

In the patient presented above, testing of peripheral blood lymphocytes revealed telomere
lengths within the normal distribution, excluding DC as the cause of his progressive
BMF.9 No increased chromosomal sensitivity to DEB was observed in his peripheral
blood lymphocytes and only a mildly increased chromosomal fragility with the formation
of few radials using MMC was observed. These findings were not sufficient for a firm
diagnosis but raised our suspicion of FA and led to further testing of skin fibroblasts that
demonstrated significant chromosomal breakage and numerous radial formations with
both DEB and MMC. Together, these results suggested FA as the cause of the patients
progressive BMF. Genetic reversion (the process by which a lymphohematopoietic cell
undergoes spontaneous correction of an FA-causing gene mutation) and blood mosaicism
are known to occur in about 15 percent of FA patients.10 This process is the likely cause of
the ambiguous FA screening results from the peripheral blood lymphocytes in this patient.
1. Bessler M, Mason PJ, Link DC, Wilson DB. Inherited bone marrow failure syndromes. In: Nathan DG,
Orkin SH, Ginsburg D, Look AT, eds. Nathans and Oskis hematology of infancy and childhood. 7th
edition. Philadelphia: W.B. Saunders Co; 2008;307-395.
2. Shimamura A. Clinical approach to marrow failure. Hematology (ASH Education Program Book).
2009:329-337.
3. Seif AE. Pediatric leukemia predisposition syndromes: clues to understanding leukemogenesis. Cancer
Genet. 2011;204:227-244.
4. Shimamura A, Alter BP. Pathophysiology and management of inherited bone marrow failure syndromes.
Blood Rev. 2010;24:101-122.
5. Parikh S, Bessler M. Recent insights into inherited bone marrow failure syndromes. Curr Opin Pediatr.
2012;24:23-32.
6. Cervenka J, Arthur D, Yasis C. Mitomycin C test for diagnostic differentiation of idiopathic aplastic
anemia and Fanconi anemia. Pediatrics. 1981;67:119-127.
7. Alter BP, Baerlocher GM, Savage SA, et al. Very short telomere length by flow fluorescence in situ
hybridization identifies patients with dykeratosis congenita. Blood. 2007;110:1439-1447.
8. Mehta P, Locatelli F, Stary J, et al. Bone marrow transplantation for inherited bone marrow failure
syndromes. Pediatr Clin North Am. 2010;57:147-170.
9. Du HY, Pumbo E, Ivanovich J, et al. TERC and TERT gene mutations in patients with bone marrow failure
and the significance of telomere length measurements. Blood. 2009;113:309-316.
10. Gregory JJ Jr., Wagner JE, Verlander PC, et al. Somatic mosaicism in Fanconi anemia: evidence of
genotypic reversion in lymphohematopoietic stem cells. Proc Natl Acad Sci USA. 2001;98:2532-2537.

Drs. Oshrine and Bessler indicated no relevant conflicts of interest.

The Hematologist:
ASH News and Reports Podcasts
The Hematologist offers a series of podcasts that
enhance the audiences reading experience by
presenting key results and additional insights from
published articles in a convenient audio format. Add
this podcast to your favorite RSS feed and follow us on
SoundCloud and iTunes. For a full listing of podcasts,
visit www.hematology.org/thehematologist/podcasts/.

M A L I G N A N T H E M A T O L O G Y 11

The Hematologist Compendium 2010-2015

Standard of Care for Treating Older Patients with Chronic


Lymphocytic Leukemia July/August 2012
UPDATE/COMMENTARY
Since our original article was published, the treatment landscape of chronic
lymphocytic leukemia (CLL) has evolved rapidly. The recently approved agents
for front-line CLL therapy (obinutuzumab and ibrutinib) demonstrate a favorable
toxicity profile, making them excellent treatment options for elderly CLL patients.
Obinutuzumab, an Fc-engineered anti-CD20 monoclonal antibody, combined
with chlorambucil, improved progression-free survival (PFS) compared with
chlorambucil alone in untreated elderly patients with CLL and comorbid conditions.
U1
The regimen was well tolerated. Therefore, obinutuzumab plus chlorambucil
was approved for front-line CLL treatment. In elderly untreated patients with
CLL without del(17p13.1), ibrutinib, an oral inhibitor of Brutons tyrosine kinase,
demonstrated significantly improved overall response and overall survival rates
compared with chlorambucil-treated patients.U2 In other studies, similar efficacy was
noted in ibrutinib-treated CLL patients with and without del(17p13.1).U3 Ibrutinib
was well tolerated. Uncommon but serious ( grade 3) adverse events in ibrutinibtreated patients were atrial fibrillation ( 6%) and major bleeding (1-4%, especially
in patients receiving warfarin). Subsequently, ibrutinib was approved for all frontline CLL patients. Based on the availability of these novel therapies, our previous

treatment algorithm must be revised. If an elderly untreated CLL patient does not
1) require anticoagulation with warfarin, 2) have uncontrolled atrial fibrillation, or
3) have malabsorption, ibrutinib should be considered. If the patient has one of the
listed comorbid conditions and does not express del(17p13.1), obinutuzumab plus
chlorambucil or the less-favored bendamustine plus rituximab regimen should be
considered. In the setting of comorbid conditions and expression of del(17p13.1),
high-dose methylprednisolone plus rituximab therapy should still be considered. In
summary, approval of the novel agents obinutuzumab and ibrutinib have established
a new standard of care for elderly patients with CLL.
U1. Goede V, Fischer K, Busch R, et al. Obinutuzumab plus chlorambucil in patients with CLL and
coexisting conditions. N Engl J Med. 2014;370:1101-1110.
U2. Burger JA, Tedeschi A, Barr PM, et al. Ibrutinib as initial therapy for patients with chronic
lymphocytic leukemia. N Engl J Med. 2015;373:2425-2437.
U3. Byrd JC, Brown JR, OBrien S, et al. Ibrutinib versus ofatumumab in previously treated chronic
lymphoid leukemia. N Engl J Med. 2014;371:213-223.

Standard frontline therapy for younger patients with cytogenetic abnormalities other than
del(17p13.1) or del(11q22.3) is the combination of fludarabine, cyclophosphamide, and
rituximab (FCR) or fludarabine and rituximab. However, a subset of CLL patients age 60
in a large study of frontline therapy with FCR were more likely to require early treatment
discontinuation due to myelosuppression and other non-hematologic toxicities,3 which
likely limits the clinical benefit of the regimen in this population. In addition, Eichhorst et
al. compared firstline CLL therapy consisting of either fludarabine or chlorambucil in a
group of 193 patients 65 years. Although fludarabine improved the overall response rate
(ORR), improved the percentage of complete remissions (CR), and increased the time to
treatment failure, there was no difference in PFS or overall survival (OS) between the two
groups. Notably, the fludarabine group demonstrated a shorter median survival time and
higher rate of toxicity, indicating that there is no major clinical benefit of using fludarabine
over chlorambucil in the older population.4 These findings were confirmed by Woyach

DEBORAH M. STEPHENS, DO, 1 AND JOHN C. BYRD, MD 2


1. Assistant Professor, Division of Hematology and Hematologic Malignancies,
University of Utah, Salt Lake City, UT
2. D. Warren Brown Chair of Leukemia Research, Professor, and Director of the
Division of Hematology, The Ohio State University, Columbus, OH

The Question
Is there a standard of care for treating older patients with CLL?

Our Response
Despite recent progress in the understanding and management of CLL, the
disease remains essentially incurable. Incidence of CLL is significantly higher
in the elderly (those > 65 years old), as the median age at diagnosis is 72 and
an estimated 70 percent of newly diagnosed patients are over 65.1 Despite the
predominance of elderly CLL patients, this group has been under-represented in
clinical trials because many have major medical co-morbidities or are perceived
likely to tolerate therapy poorly. This omission leaves clinicians who treat CLL
without definitive data on which therapy to choose for the large population of
older adults.

Figure

Therapeutic Recommendations for Elderly Patients with


Chronic Lymphocytic Leukemia who Meet Criteria for Therapy
Eligible for
clinical trial?

At initial presentation, the clinical spectrum of CLL ranges from an asymptomatic


patient, identified by routine blood work, to a symptomatic patient who
experiences rapid progression to death from complications of CLL. Therefore,
the first decision to make is whether or not treatment is required. Indications
for consideration of treatment include: clinical symptoms (fevers, night sweats,
weight loss, or painful lymphadenopathy or splenomegaly), non-immune-mediated
cytopenias (hemoglobin < 11g/dL or platelets < 100 1012/L), autoimmune
hemolytic anemia, or thrombocytopenia (ITP) that is poorly responsive to standard
immunosuppressive therapy and rapidly progressive disease (lymphocyte count
rising to > 300 109/L or rapidly enlarging lymph nodes, spleen, or liver). Isolated
mild thrombocytopenia (platelets 70-100 1012/L) often represents chronic ITP and
can be followed closely without treatment if symptoms are absent.
Treatment decisions can be informed by identifying factors that influence
response rates, duration of response, and prognosis, including specific
interphase cytogenetic abnormalities such as del(17p13.1) and del(11q22.3),
complex cytogenetics (> 3 abnormalities) determined by stimulated metaphase
karyotype, unmutated immunoglobulin heavy chain gene status, Rai stage III/
IV, Zap-70 expression in 20 percent or more of peripheral blood lymphocytes,
and elevated serum 2microglobulin concentration. Trials by the German CLL
Study Group have factored in co-morbid features assessed by the CIRS-G scale2
for choosing therapy in elderly patients; however, this score is subjective, and,
to date, no study has documented the benefit of this scale over simpler methods.
Only performance status and the presence of del(17p13.1) or del(11q22.3)
influence our treatment choice (Figure).
If an elderly CLL patient requires therapy and is eligible for and willing to participate,
we recommend enrollment in a clinical trial. Outside of a trial, if the patients
performance status will allow therapy, he or she should be assigned to one of three
groups according to the presence of FISH/cytogenetic abnormalities: del(17p13.1),
del(11q22.3), and all others.

Yes

No

Enroll in
clinical trial

Functional
performance status
acceptable for
treatment?

Yes

No

Cytogenetics other
than Del(11q)
or Del(17p)
expressed

Del(11q)
expressed

Del(17p)
expressed

Chlorambucil +/rituximab

FCR,* PCR,** or
bendamustine
+ rituximab

High-dose
methylprednisolone
+ rituximab

Consider
chlorambucil,
rituximab +
chlorambucil, or
palliative care

*FCR=fludarabine, cyclophosphamide, and rituximab; **PCR=pentostatin, cyclophosphamide, and rituximab

M A L I G N A N T H E M A T O L O G Y 12

The Hematologist Compendium 2010-2015

and colleagues5 who reviewed the experience of elderly CLL patients across all completed
Cancer and Leukemia Group B trials and demonstrated no benefit for fludarabine treatment
in patients > 70 years. In contrast, patients of all age groups appeared to benefit from the
addition of rituximab. Recently, Hillmen and colleagues published a phase II trial in which
rituximab was combined with chlorambucil in 50 patients (median age = 70.5 years). ORR
was 84 percent, with infection and neutropenia being the most common adverse effects.6
Therefore, for an elderly CLL patient without either del(17p13.1) or del(11q22.3), we
recommend therapy with chlorambucil (10 mg/m2 orally days 1-7 of a 28-day cycle) +/rituximab (375 mg/m2 day 1, cycle 1 and 500 mg/m2 day 1, cycles 2-6).

1. Howlader N, Noone AM, Krapcho M, et al(eds). SEER Cancer Statistics Review, 1975-2009 (Vintage
2009 Populations), SEER Stat Fact Sheet: Chronic Lymphocytic Lymphoma, National Cancer Institute.
Bethesda, MD, http://seer.cancer.gov/statfacts/html/clyl.html, based on November 2011 SEER data
submission, posted to the SEER web site, April 2012.Accessed June 1, 2012.

Patients with del(11q22.3) do not have favorable outcomes in the absence of fractionated
alkylator-based therapy. For older CLL patients with del(11q22.3), we recommend therapy
with FCR, PCR (pentostatin, cyclophosphamide, and rituximab) or bendamustine (70 mg/
m2 days 1 and 2 of 28-day cycle) and rituximab (375 mg/m2 first cycle, 500 mg/m2 subsequent
5 cycles). Using the latter regimen, Fischer et al. reported an ORR of 90.9 percent in 117
patients (median age = 64 years) with minimal major toxicities. A subgroup analysis of
patients with del(11q22.3) showed a 90.5 percent ORR.7

5. Woyach JA, Ruppert AS, Peterson B, et al. Impact of age on outcomes following initial therapy with
various chemotherapy and chemoimmunotherapy regimens in patients with chronic lymphocytic
leukemia: Results of CALGB studies. Blood. ASH Annual Meeting. 2011;118:289.

Treatment options are limited for older patients with del(17p13.1). Many propose the use
of alemtuzumab, but we dont recommend this therapy due to significant risk of infectious
complications and limited response duration. Instead, we recommend rituximab (375 mg/
m2 weekly for 12 weeks) in combination with high-dose methylprednisolone (1 g/m2 days 1-3
of each 4-week cycle). Using this regimen, James and colleagues reported that in a relatively
small study of 28 patients (29% age 70) the ORR was 96 percent (100% in the > 70-yearold subset) with CR observed in 32 percent (38% in > 70 years). Adverse effects were found
to be minimal.8 In general, for patients being treated with this regimen, we recommend
hospitalization during the first three days of cycle 1 as tumor lysis syndrome, metabolic
aberrations, and fluid retention can occur.

2. Miller MD, Paradis CF, Houck PR, et al. Rating chronic medical illness burden in geropsychiatric practice
and research: application of the Cumulative Illness Rating Scale. Psychiatry Res. 1992;41:237-248.
3. Keating MJ, OBrien S, Albitar M, et al. Early results of a chemoimmunotherapy regimen of fludarabine,
cyclophosphamide, and rituximab as initial therapy for chronic lymphocytic leukemia. J Clin Oncol.
2005;23:4079-4088.
4. Eichhorst BF, Busch R, Stilgenbauer S, et al. First-line therapy with fludarabine compared with
chlorambucil does not result in a major benefit for elderly patients with advanced chronic lymphocytic
leukemia. Blood. 2009;114:3382-3391.

6. Hillmen P, Gribben JG, Follows A, et al. An open-label phase II study to investigate the safety and efficacy
of rituximab plus chlorambucil in previously untreated patients with CD20-positive B-cell chronic
lymphocytic leukemia. Blood. ASH Annual Meeting. 2009;114:3428.
7. Fischer K, Cramer P, Stilgenbauer S, et al. Bendamustine combined with rituximab in first-line therapy of
advanced CLL: a multicenter phase II trial of the German CLL Study Group. Blood. ASH Annual Meeting.
2009;114:205.
8. James AF, Castro JE, Sandoval-Sus JD, et al. Rituximab and high-dose methylprednisolone for the intial
treatment of chronic lymphocytic leukemia is associated with promising clinical activity and minimal
hematologic toxicity. Blood. ASH Annual Meeting Abstracts. 2008;112:47.

Drs. Stephens and Byrd indicated no relevant conflicts of interest.

For elderly, unfit patients without del(17p13.1), for whom we have concerns that therapy will
be poorly tolerated, we typically recommend chlorambucil or rituximab alone or palliative
supportive care. Elderly, unfit patients with del(17 p13.1) are unlikely to respond to either of
these therapies and should be directed to supportive care.

WATCH THE EXPERTS

In summary, due to paucity of definitive data on management of the elderly CLL


population, we encourage enrollment of these patients in clinical trials. However, when
enrollment in a clinical study is not feasible, we use a treatment approach based on FISH/
cytogenetic analysis together with our assessment of the individual patients capacity
to tolerate therapy (Figure). The CLL treatment landscape is evolving rapidly. Promising
new therapies that are currently undergoing clinical trials include lenalidomide, ibrutinib,
GS1101, ofatumumab, GA101, ABT-199, and TRU-016. While we await the outcome of these
and other studies, a carefully considered approach to the management of elderly patients
with CLL is essential.

Jennifer Woyach, MD
Dr. Jennifer Woyach discusses the latest drug
treatments for CLL patients in this video from
the ASH Impact Series available on ASH On
Demand. To watch this video and more, visit
www.ashondemand.org/FreeContent/Videos.

M A L I G N A N T H E M A T O L O G Y 13

The Hematologist Compendium 2010-2015

Optimal, Upfront Management of Mantle Cell Lymphoma November/December 2012


The authors have provided a rewrite of the original article to present all content updates since the publication date in 2012.

OREOFE O. ODEJIDE, MD, MPH, 1 AND ANN S. LACASCE, MD, MSc 2


1. Physician, Dana-Farber Cancer Institute, Boston, MA
2. Assistant Professor of Medicine, Harvard Medical School, Boston, MA

Question
What is the optimal, upfront management of mantle cell lymphoma (MCL)?

Our Response
MCL is a rare type of lymphoma, representing about 6 percent of all cases of non-Hodgkin
lymphoma.1 Median age at diagnosis is 68 years, and the vast majority of patients are
diagnosed with advanced-stage disease. Patients with MCL typically present with
generalized lymphadenopathy and extranodal involvement, particularly in the bone
marrow, spleen, and gastrointestinal tract. Cyclin D1 expression as a result of t(11;14)
(q13;q32) translocation is present in more than 95 percent of cases. Notably, MCL has a
poor prognosis, though the median overall survival (OS) has improved over time and is
now approximately seven to eight years.
Given the small number of randomized clinical trials in previously untreated patients
with MCL, none of which have demonstrated an improvement in OS, the optimal initial
approach to MCL has not been clearly established. Subsequently, patient participation in
well-designed clinical trials is highly recommended. This article will focus on treatment
strategies that have been shown to offer the best chance of prolonged remission.

A Watch-and-Wait Strategy Is Appropriate for a Subset of Patients


Given the burden of disease, the majority of patients with MCL require therapy at diagnosis.
However, a subset of patients with splenomegaly and circulating lymphoma cells with a
low ki-67, typically without significant lymphadenopathy, may have an indolent course.
Data from Weill Cornell Medical Center also suggest that, for selected patients with
asymptomatic disease, there is no survival disadvantage to expectant management.2 For
those requiring therapy, patients may be divided into categories of young and fit ( 65 years
without significant comorbidity) or older (> 65 years with inability to tolerate high-dose
therapy).

Young and Fit Adults with MCL


Although the addition of rituximab to conventional
chemotherapy regimens has significantly improved
prognosis in nearly all subtypes of B-cell non-Hodgkin
lymphoma, its addition to cyclophosphamide, doxorubicin,
vincristine, and prednisone (CHOP) in MCL has not
translated into improved outcomes. The German Low
Grade Lymphoma Study Group found that, due to the short
durability of the response, rituximab with CHOP (R-CHOP)
compared with CHOP did not improve progression-free
survival (PFS) despite an improvement in overall response
rate (94% vs. 75%).3 Given the disappointing outcome with
R-CHOP therapy alone in younger patients, emphasis has
been placed on intensification of induction therapies and
consolidation strategies with high-dose therapies followed
by autologous stem cell rescue (ASCR).

six cycles of CHOP to R-CHOP/rituximab with DHAP, suggest both a PFS and OS advantage
for patients receiving the intensified regimen.7 In terms of post-transplantation strategies,
preliminary results from a randomized study from Europe demonstrated an improved threeyear event-free survival of 88 percent versus 73 percent in patients receiving maintenance
rituximab versus observation.8
Another approach to increase the durability of remission in MCL patients uses intensified
doses of chemotherapy including cytarabine without stem cell transplantation. A phase II
trial of R-HyperCVAD (rituximab, fractionated cyclophosphamide, vincristine, doxorubicin,
and dexamethasone alternating with high-dose methotrexate and cytarabine) by the MD
Anderson Cancer Center group showed a high complete remission rate of 87 percent and
three-year failure-free survival of 64 percent. However, this therapy has significant toxicity,
with 8 percent treatment-related mortality.9 The regimen has been tested in the cooperative
group setting by the Southwest Oncology Group, with demonstrated three-year PFS and
OS of 66 percent and 81 percent respectively.10 Of note, 39 percent of patients were unable
to complete all therapy due to significant myelosuppression and infection. Therefore, this
therapy should be reserved for young and fit patients with close monitoring for toxicity.
Finally, a comparative effectiveness study from the National Comprehensive Cancer
Network NHL database comparing initial MCL therapy in younger patients demonstrated
that R-CHOP alone was inferior to aggressive therapies such as R-HyperCVAD or R-CHOP
followed by HDT and ASCR.11
In general, we favor an intensive treatment approach consisting of an induction regimen
followed by HDT and ASCR for patients who are 65 years or younger without significant
comorbidity. The optimal induction chemotherapy has not been established, and options
include standard or intensified R-CHOP with or without the addition of cytarabinecontaining regimens. Post-transplant maintenance rituximab may be considered, but it is
associated with neutropenia and risk of sinopulmonary infections.

Older Adults with MCL


The use of dose-intensified, cytarabine-containing induction regimens followed by HDT
and ASCR is not feasible for most elderly patients, due to excessive toxicity. Therefore,
treatment strategies focus on regimens that improve PFS while minimizing toxicity. Results
of a double randomized trial comparing R-CHOP versus R-FC (rituximab, fludarabine, and
cyclophosphamide) in patients older than 60 years who were ineligible for HDT, with
second randomization of responders (complete or partial response) to either maintenance
rituximab or maintenance interferon- showed a clear OS benefit as well as less toxicity for
R-CHOP compared with R-FC. The remission duration was also doubled in the maintenance
rituximab versus the interferon- arm. Moreover, among patients who had a response to

Figure

Recommendations for Initial Management of Mantle Cell Lymphoma

A randomized trial comparing myeloablative


radiochemotherapy and ASCR versus maintenance
interferon- in patients 65 years achieving a partial or
complete response with CHOP-like induction, with or
without rituximab, demonstrated an improved PFS in the
radiochemotherapy arm (median PFS, 39 months vs. 17
months) though there was no OS difference.4 This study is
the basis for adoption of high-dose therapies and ASCR as
consolidative strategies in younger patients. The Nordic
Lymphoma Group in the MCL2 study adopted a cytarabinecontaining induction regimen consisting of six cycles of
dose-intensified CHOP alternating with high-dose cytarabine
with rituximab in cycles two through six, followed by highdose therapy (HDT) and ASCR. The four- and 10-year PFS
were highly encouraging at 73 percent and 55 percent,
respectively.5 A median event-free survival of 84 months
was reported by the French group in a phase II trial using an
induction regimen consisting of three cycles of CHOP and
three cycles of rituximab with DHAP (dexamethasone, highdose cytarabine, and cisplatin) followed by HDT and ASCR.6
Preliminary results of a randomized phase III trial comparing
M A L I G N A N T H E M A T O L O G Y 14

KEY
HDT: High-dose therapy
ASCR: Autologous stem cell rescue
R-CHOP: Rituximab, cyclophosphamide, doxorubicin, vincristine, prednisone
BR: Bendamustine and rituximab VR-CAP: Bortezomib, rituximab, cyclophosphamide,
doxorubicin, prednisone

The Hematologist Compendium 2010-2015

R-CHOP induction, maintenance rituximab significantly improved OS when compared with


maintenance interferon- (4-year OS, 87% vs. 63%), while it had no influence in the group
that received R-FC induction.12

1. A clinical evaluation of the International Lymphoma Study Group classification of non-Hodgkins


lymphoma: The Non-Hodgkins Lymphoma Classification Project. Blood. 1997;89:3909-3918.

Bendamustine is also a highly active drug in MCL. When compared to R-CHOP,


bendamustine plus rituximab (BR) resulted in an improved PFS of 35 months compared
to 22 months in a randomized study from Europe.13 In early results from a small study
in the upfront setting, the combination of lenalidomide plus rituximab yielded overall
and complete response rates of 92 and 64 percent respectively with a two-year PFS of 85
percent.14 In a randomized study comparing bortezomib, rituximab, cyclophosphamide,
doxorubicin, and prednisone (VR-CAP) to R-CHOP in transplant-ineligible patients,
VR-CAP resulted in a superior median PFS of 24.7 months compared with 14.4 months
with R-CHOP.15 However, there were significantly increased rates of toxicity with VR-CAP,
and it is not clear whether its superior PFS will be sustained in the setting of rituximab
maintenance after R-CHOP. An ongoing intergroup study in the United States is currently
comparing BR with or without bortezomib followed by random assignment to maintenance
rituximab with or without lenalidomide. Until these data are mature, we favor R-CHOP or BR
therapy followed by maintenance rituximab as upfront therapy in older patients with MCL.

3. Lenz G, Dreyling M, Hoster E, et al. Immunochemotherapy with rituximab and cyclophosphamide,


doxorubicin, vincristine, and prednisone significantly improves response and time to treatment failure,
but not long-term outcome in patients with previously untreated mantle cell lymphoma: results of a
prospective randomized trial of the German Low Grade Lymphoma Study Group (GLSG). J Clin Oncol.
2005;23:1984-1992.

2. Martin P, Chadburn A, Christos P, et al. Outcome of deferred initial therapy in mantle-cell lymphoma. J
Clin Oncol. 2009;27:1209-1213.

4. Dreyling M, Lenz G, Hoster E, et al. Early consolidation by myeloablative radiochemotherapy followed


by autologous stem cell transplantation in first remission significantly prolongs progression-free
survival in mantle-cell lymphoma: results of a prospective randomized trial of the European MCL
Network. Blood. 2005;105:2677-2684.
5. Geisler CH, Kolstad A, Laurell A, et al. Long-term progression-free survival of mantle cell lymphoma
after intensive front-line immunochemotherapy with in vivo-purged stem cell rescue: a nonrandomized
phase 2 multicenter study by the Nordic Lymphoma Group. Blood. 2008;112:2687-2693.
6. Delarue R, Haioun C, Ribrag V, et al. CHOP and DHAP plus rituximab followed by autologous stem cell
transplantation in mantle cell lymphoma: a phase 2 study from the Groupe dEtude des Lymphomes de
lAdulte. Blood. 2013;121:48-53.
7. Hermine O, Hoster E, Walewski J, et al. Alternating courses of 3x CHOP and 3x DHAP plus rituximab
followed by a high dose ARA-C containing myeloablative regimen and autologous stem cell
transplantation (ASCT) increases overall survival when compared to 6 courses of chop plus rituximab
followed by myeloablative radiochemotherapy and asct in mantle cell lymphoma: final analysis of the
MCL younger trial of the European Mantle Cell Lymphoma Network. Blood. 2012;623:151.
8. Le Gouill S, Deconinck E, Ghesquieres H, et al. Rituximab maintenance versus WW after R-DHAP plus
autologous stem cell transplantation in untreated patients with MCL: Interim analysis of the LYMA trial,
a LYSA study. Haematologica. 2015:387-387.
9. Romaguera JE, Fayad L, Rodriguez MA, et al. High rate of durable remissions after treatment of newly
diagnosed aggressive mantle-cell lymphoma with rituximab plus hyper-CVAD alternating with rituximab
plus high-dose methotrexate and cytarabine. J Clin Oncol. 2005;23:7013-7023.
10. Bernstein SH, Epner E, Unger JM, et al. A phase II multicenter trial of hyperCVAD MTX/Ara-C and
rituximab in patients with previously untreated mantle cell lymphoma; SWOG 0213. Ann Oncol.
2013;24:1587-1593.
11. LaCasce AS, Vandergrift JL, Rodriguez MA, et al. Comparative outcome of initial therapy for
younger patients with mantle cell lymphoma: an analysis from the NCCN NHL Database. Blood.
2012;119:2093-2099.
12. Kluin-Nelemans HC, Hoster E, Hermine O, et al. Treatment of older patients with mantle-cell lymphoma.
N Engl J Med. 2012;367:520-531.
13. Rummel MJ, Niederle N, Maschmeyer G, et al. Bendamustine plus rituximab versus CHOP plus
rituximab as first-line treatment for patients with indolent and mantle-cell lymphomas: an open-label,
multicentre, randomised, phase 3 non-inferiority trial. Lancet. 2013;381:1203-1210.
14. Ruan J, Martin P, Shah B, et al. lenalidomide plus rituximab as initial treatment for mantle-cell
lymphoma. N Engl J Med. 2015;373:1835-1844.
15. Robak T, Huang H, Jin J, et al. bortezomib-based therapy for newly diagnosed mantle-cell lymphoma. N
Engl J Med. 2015;372:944-953.

WATCH THE EXPERTS

Dr. Odejide and Dr. LaCasce indicated no relevant conflicts of interest.

John C. Byrd, MD
Dr. John Byrd of The Ohio State University explains
how the identification of novel targets in leukemia
cells has impacted patients lives at the 2015
ASH Meeting on Hematologic Malignancies in
Chicago. Watch the video at www.hematology.org/
Malignancies/Speakers.

Read an article by Dr. Byrd in this Compendium on page 12.

M A L I G N A N T H E M A T O L O G Y 15

The Hematologist Compendium 2010-2015

Approach to the Diagnosis and Management of Mastocytosis January/February 2013


UPDATE/COMMENTARY
Several new developments have emerged in the field of systemic mastocytosis (SM)
since the publication of the Ask The Hematologist article on this subject in 2013.
Classification
In the revised 2016 World Health Organization Classification of hematolymphoid
neoplasms, mastocytosis was removed as one of the subtypes under the major
category of myeloproliferative neoplasms and is now classified as its own major
category.U1 Smoldering systemic mastocytosis is now a full variant of SM, and
no longer a provisional subvariant under indolent SM. Additionally, systemic
mastocytosis with an associated hematologic non-mast cell lineage disease
(SM-AHNMD) has been given a name makeover; the designation SM with an
associated hematologic neoplasm (SM-AHN), a simpler designation, can now be
used in place of, or interchangeably with, SM-AHNMD.U1
More attention is being focused on how to better capture the clinical heterogeneity
within SM subtypes. For example, for patients with aggressive systemic mastocytosis
(ASM) and increased numbers of neoplastic mast cells on a bone marrow aspirate
(e.g., 5-19%), the term ASM-t (ASM in transformation) has been proposed to reflect
the worse prognosis of these patients and higher risk of transformation to mast cell
leukemia (MCL) compared to individuals with ASM patients with <5 percent mast
cells on a marrow aspirate.U2 Although the life expectancy of MCL is often less than
six months, such patients may also exhibit variable clinical outcomes. The terms
acute MCL (one or more signs of organ damage) and chronic MCL (no evidence
of organ damage) have been proposed as variants that respectively denote relatively
worse and better outcomes within the spectrum of MCL.U3 However, retrospective and
prospective data are needed to confirm whether the absence of presence of organ
damage adequately discriminates prognosis between these two MCL subgroups.
Biology
Although the KIT D816V mutation is identified in approximately 90 percent of SM
patients, SM-AHNMD is usually characterized by multiple myeloid mutations, the
most common being TET2, SRSF2, ASXL1, CBL, and RUNX1.U4 Patients with a mutation in one or more of these genes (and especially SRSF2, ASXL1, or RUNX1) exhibit
a worse prognosis compared to patients who only exhibit the KIT D816V mutation.
U5
Sequencing of granulocyte-macrophage colony-forming progenitor cells (CFU-GM)
indicates that such mutations often precede the acquisition of KIT D816V.U6 This parallels the finding that TET2 mutations often arise before JAK2 V617F in Philadelphia
chromosome-negative MPNs.U7 Notably, CFU-GM from ISM or SSM patients only
exhibited the KIT D816V mutation, indicating that indolent SM may be primarily
driven by this mutation alone.U6
Treatment
In the fully accrued global trial of the multikinase/KIT inhibitor midostaurin in 89
evaluable patients with advanced SM (ASM, SM-AHNMD, and MCL), the overall
response rate was 60 percent; 45 percent of patients achieved a major response,
defined by complete resolution of 1 type of organ damage.U8 The median duration
of response was 24.1 months. Median overall survival was 28.7 months and
progression-free survival was 14.1 months. Response rates were similar regardless
of advanced SM subtype, KIT mutation status, or prior therapy. The serum tryptase
level and bone marrow mast cell burden decreased by >50 percent in the majority
of patients. In addition to reducing splenomegaly, midostaurin elicited significant
improvement of patient-reported symptoms and quality of life that may be related to

JASON GOTLIB, MD, MS


Associate Professor of Hematology, Stanford University School of Medicine, Stanford
Cancer Institute, Stanford, CA

The Question
What is your approach to the diagnosis and management of mastocytosis?

Defining Mastocytosis
Mastocytosis is characterized by the accumulation and proliferation of neoplastic mast
cells in one or more organs (e.g., skin, bone marrow, spleen, lymph nodes, liver, and
gastrointestinal tract). Flushing, diarrhea, anaphylaxis, and neuropsychiatric symptoms
resulting from mast cell release of bioactive molecules such as histamine, leukotrienes, and
various inflammatory cytokines can impose a significant burden on a patients quality of life.
Such mediator symptoms may be triggered by physical stimuli, exercise, alcohol, NSAIDs,
opioids, insect stings, or certain foods. In advanced disease, mast cell infiltration of tissues
can lead to organ damage and shortened survival.
The World Health Organization (WHO) includes mastocytosis within the category of
myeloproliferative neoplasms (MPN) and divides these disorders into cutaneous and
systemic forms.1 The diagnosis of systemic mastocytosis (SM) is based on consensus
M A L I G N A N T H E M A T O L O G Y 16

the combined inhibitory effects of midostaurin on neoplastic mast cell proliferation


and mediator release. In the MCL patients, where median survival of <6 months is
typical, the median overall survival was 9.4 months and not reached in the eight
of 16 MCL patients who responded. Further studies are warranted to evaluate the
role of combining midostaurin with other drugs with activity in advanced SM (e.g.
2-chlorodeoxadenosine; 2-CdA) and in the pre- and post-transplant setting. New
KIT D816V-specific inhibitors (e.g. BLU-285) have recently commenced clinical trial
evaluation of patients with advanced SM.
Recent data from Dr. Celalettin Ustun and colleagues have provided more clarity on
the role of hematopoietic stem cell transplantation (HSCT) in advanced SM which
heretofore has been primarily based on case reports and small series.U9 In a large,
multicenter retrospective analysis published in 2014, Dr. Ustun and colleagues
evaluated the outcomes of 57 SM patients (SM-AHNMD, n=38; ASM, n=7; and MCL,
n=12) who underwent allogeneic HSCT. Responses were observed in 70 percent
of patients, including a 16 percent complete remission (CR) rate. The remaining
30 percent of responses were split between stable disease (21%) and primary
refractory disease (9%). All 38 patients with SM-AHNMD achieved CR of the AHNMD
component, but 10 subsequently relapsed with AHNMD, and half of these patients
died. The median overall survival for all patients at three years was 57 percent, for
all patients, consisting of 74 percent for patients with SM-AHNMD, and 43 and 17
percent for ASM and MCL patients, respectively. The strongest risk factor for worse
overall survival was a diagnosis of MCL. Additionally, lower survival was observed
in patients undergoing reduced intensity vs. myeloablative conditioning. These data
suggest that transplantation can provide extended survival in selected patients,
particularly for patients with SM-AHNMD.
U1. Arber DA, Orazi A, Hasserjian R, et al. The 2016 revision to the World Health Organization
(WHO) classification of myeloid neoplasms and acute leukemia. Blood. 2016;127:2391-2405.
U2. Valent P. Diagnosis and management of mastocytosis: an emerging challenge in applied
hematology. Hematology Am Soc Hematol Educ Program. 2015;2015:98-105.
U3. Valent P, Sotlar K, Sperr WR, et al. Refined diagnostic criteria and classification of mast cell
leukemia (MCL) and myelomastocytic leukemia (MML): a consensus proposal. Ann Oncol.
2014;25:1691-1700.
U4. Schwaab J, Schnittger S, Sotlar K, et al. Comprehensive mutational profiling in advanced
systemic mastocytosis. Blood. 2013;122:2460-2466.
U5. Jawhar M, Schwaab J, Schnittger S, et al. Additional mutations in SRSF2, ASXL1, and/or RUNX1
identify a high-risk group of patients with KIT D816V(+) advanced systemic mastocytosis.
Leukemia. 2016;30:136-143.
U6. Jawhar M, Schwaab J, Schnittger S, et al. Molecular profiling of myeloid progenitor cells in multimutated advanced systemic mastocytosis identified KIT D816V as a distinct and late event.
Leukemia. 2015;29:1115-1122.
U7. Ortmann CA, Kent DG, Nangalia J, et al. Effect of mutation order on myeloproliferative
neoplasms. New Engl J Med. 2015;372:601-612.
U8. Gotlib J, Kluin-Nelemans HC, George TI, et al. Efficacy and safety of midostaurin in advanced
systemic mastocytosis. N Engl J Med. 2016;374:2530-2541.
U9. Ustun C, Reiter A, Scott BL, et al. Hematopoietic stem-cell transplantation for advanced
systemic mastocytosis. J Clin Oncol. 2014;32:3264-3274.

Dr. Gotlib is Chairman of the Study Steering Committee for the global trial of
midostaurin and receives reimbursement for travel expenses from Novartis;
Dr. Gotlib receives research funding for administration of the BLU-285 trial.

diagnostic criteria (Table) and requires one major plus one minor criterion or three minor
criteria. Clinicopathologic criteria are used to further classify SM into subtypes that generally
reflect clinical severity and prognosis. In indolent SM (ISM), where bone marrow mast cell
burden is relatively low, survival is similar to age-matched populations, and clinical issues are
usually related to mediator symptoms.2 Smoldering SM (SSM) is a subtype of ISM that reflects
SM in transition toward more advanced disease and is characterized by two or more so-called
B-findings (Figure), including organomegaly, and indicators of a relatively higher mast cell
burden (e.g., > 30% bone marrow mast cells and serum tryptase level > 200 ng/ml). Advanced
SM collectively refers to SM subtypes in which organ damage (referred to as C findings) is
present. This includes the WHO-defined subtypes ASM, MCL, and SM-AHNMD. Approximately
90 percent of AHNMDs represent a myeloid neoplasm such as myelodysplastic syndrome
(MDS), MPN, MDS/MPN (e.g., chronic myelomonocytic leukemia), eosinophilic disorders, or
acute myeloid leukemia (AML). Rarely, a lymphoid neoplasm may be concurrently diagnosed
with SM.
Mastocytosis is a challenging disease for many reasons. In the normal state, mast cells
are not identified on the peripheral blood differential and represent a small fraction of the
myeloid lineage on visual inspection of the marrow aspirate. The idiom out of sight, out
of mind is apt in the case of SM and, together with its rare incidence, tests the physician
to think about this neoplasm in the first place. If present, prototypic mediator symptoms
such as flushing, diarrhea, and anaphylaxis may make this task easier. In advanced disease,
organ damage such as cytopenias, liver dysfunction, ascites, lytic bone lesions, and
hypoalbuminemia with weight loss due to mast cell infiltration of the gastrointestinal tract
are sufficiently protean to conceal a unifying diagnosis. Because SM often partners with a
myeloid disease, immunohistochemical (IHC) stains (e.g., tryptase, CD117 [KIT], and CD25)
are necessary to identify and quantify neoplastic mast cells in trephine biopsies and can

The Hematologist Compendium 2010-2015

additionally help unmask SM when it lives in the shadow of


other neoplasms. Clinical and biologic heterogeneity, even
within specific SM disease subtypes, can make it difficult
to gauge disease trajectory and prognosis. In patients with
an AHNMD, prognosis is usually related to the associated
myeloid neoplasm, highlighting the need to fully characterize
and stage the concomitant disease.

Diagnosis

Table

World Health Organization Diagnostic Criteria for Systemic Mastocytosis*

Major
criterion

Multifocal dense infiltrates of mast cells (>15 mast cells in aggregates) detected in sections of bone marrow and/
or other extracutaneous organ(s)

Minor
criteria

a. In biopsy sections of bone marrow or other extracutaneous organs, >25% of the mast cells in the infiltrate are
spindle shaped or have atypical morphology or, of all mast cells in bone marrow aspirate smears, >25% are
immature or atypical.

b. Detection of an activating point mutation at codon 816 in KIT in bone marrow, blood, or another extracutaneous organ

In addition to a high index of suspicion, obtaining the



c. Mast cells in bone marrow, blood, or other extracutaneous organs express CD2 and/or CD25 in addition to normal
expertise of a cadr of subspecialists (hematologists,

mast cell markers
dermatologists, and allergists/immunologists) as well as

d. Serum total tryptase persistently exceeds 20 ng/ml (unless there is an associated clonal myeloid disorder, in which
hematopathologists versed with the nuances of mast cell

case this parameter is not valid)
disease is important in establishing the correct diagnosis. A
combination of clinical, morphologic, immunophenotypic,
*Requires 1 major + 1 minor criterion or 3 minor criteria
and molecular studies is required to establish the diagnosis
of SM and its subtype. Bone marrow mast cell burden is best
symptoms who do not fulfill criteria for SM (e.g., only one or two minor criteria and/or exhibit
quantified by morphologic analysis, and use of the aforementioned IHC stains on the core
a low burden of atypical mast cells without aggregates on biopsy) have been given the
biopsy is essential. The major criterion for SM requires demonstration of multifocal mast
provisional diagnosis of mast cell activation syndrome (MCAS).2 The diagnostic criteria for
cell aggregates in the bone marrow (or other extracutaneous organ), of which > 25 percent
this mast cell disorder are still evolving and its natural history is not well-defined.
are atypical, often spindle-shaped mast cells. In MCL, mast cells account for > 20 percent of
nucleated cells on the BM aspirate and form a diffuse infiltrate on the core biopsy with or
without circulating mast cells. Multi-parameter flow cytometry of the BM aspirate can also
Treatment
be used to quantify mast cells in the marrow and complements morphologic evaluation. The
overwhelming majority of SM patients (> 80%) carry the activating KIT D816V mutation, which
Individuals with symptomatic skin-only disease or SM with mediator symptoms are
should be obtained from the bone marrow aspirate or from a core biopsy that is preserved in
educated to avoid known triggers and are encouraged to carry an Epipen, particularly
formalin to avoid degradation of DNA for PCR analysis. Determination of KIT D816V mutation
those with a history of anaphylaxis or anaphylactoid symptoms. Antihistamines (H1- and
status is critical to the diagnostic evaluation of SM, but it also guides treatment decisions.
H2-blockers) serve as the foundation for symptom palliation; leukotriene antagonists (e.g.,
Although KIT D816V is an imatinib-resistant mutation, this tyrosine kinase inhibitor has
monteleukast) and mast cell stabilizers (e.g., cromolyn sodium) are typically used for
been reflexively misapplied and overused in the treatment of SM. However, a small minority
refractory mediator symptoms. Osteoporosis is treated with conventional approaches, and
of patients (< 5%) may exhibit juxtamembrane KIT mutations, which exhibit sensitivity to
radiotherapy and/or IV bisphosphonates may be used to treat osteolyses or pathologic
imatinib in vitro as well as in clinical practice. Such cases illustrate that sequencing the
fractures. Patients with advanced SM exhibit shortened survival, and cytoreductive therapy
remainder of the KIT gene in SM patients who are negative for codon 816 mutations may prove
is used in an attempt to reverse organ damage. Interferon- with or without corticosteroids
fruitful. Total serum tryptase levels generally reflect the increased burden of mast cells in
and 2-chloro-deoxyadenosine demonstrate overall response rates in the range of 30 to
patients with SM. A serum tryptase level > 20 ng/ml is an additional minor criterion for the
60 percent, including major responses denoting normalization of organ dysfunction.3
diagnosis of SM. Although not always concordant, it is the most useful blood marker to assess
Intensive chemotherapy has been used in MCL with modest benefit, and there is a paucity
changes in mast cell burden in response to cytoreductive therapy. In advanced disease,
of published data regarding the utility of stem cell transplantation for advanced SM. The
staging studies include CT of the abdomen to assess hepato/splenomegaly, lymphadenopathy,
generally short-lived responses and tolerability of these approaches have reinforced the
and ascites; metastatic skeletal survey to evaluate osteolyses and/or pathologic fractures;
need to enroll patients in clinical trials evaluating novel agents. The tyrosine kinase inhibitors
and DEXA scans to follow osteoporosis, which is common in SM. Some patients with mediator
dasatinib and midostaurin exhibit in vitro activity against D816V-mutated KIT, with the latter
demonstrating encouraging activity in an ongoing international, multi-center clinical trial.4
In patients with SM-AHNMD, the clinical approach for such patients has been to treat the SM
Figure
component as if the myeloid neoplasm were not present and to treat the myeloid neoplasm
as if SM were not present. However, in clinical practice, priority should be given to treating
the disease component that is contributing to the most urgent clinical concerns. Because
KIT D816V or other pathogenetic abnormalities may reside in both the abnormal mast cell
and associated myeloid clonal cell populations, the future availability of agents that inhibit
shared therapeutic targets may make the distinction between the two disease compartments
less relevant. Next-generation sequencing approaches of sorted cell populations should
inform this approach. The recent identification of CD30 expression on neoplastic mast cells5
also provides an opportunity for testing anti-CD30 antibody approaches (e.g., brentuximab
vedotin) in advanced SM.
1. Horny HP, Akin C, Metcalfe DD, et al. Mastocytosis. In: Swerdlow S, Campo E, Lee Harris N, et al.,
editors. WHO Classification of Tumours of Haematopoietic and Lymphoid Tissues. Lyon: IARC Press.
2008:53-63.
2. Akin C, P, Metcalfe DD. Mast cell activation syndrome: Proposed diagnostic criteria. J Allergy Clin
Immunol. 2010;126:1099-1104.
3. Lim KH, Pardanani A, Butterfield JH, et al. Cytoreductive therapy in 108 adults with systemic
mastocytosis: Outcome analysis and response prediction during treatment with interferon-alpha,
hydroxyurea, imatinib mesylate or 2-chlorodeoxyadenosine. Am J Hematol. 2009;84:790-794.
4. Gotlib J, Kluin-Nelemans, George TI, et al. KIT inhibitor midostaurin in patients with advanced systemic
mastocytosis: Results of a planned interim analysis of the global CPKC412D2201 trial. Blood. ASH
Annual Meeting Abstracts. 2012;120:799.
5. Sotlar K, Cerny-Reiterer S, Petat-Dutter K, et al. Aberrant expression of CD30 in neoplastic mast cells in
high-grade mastocytosis. Mod Pathol. 2011;24:585-595.

Dr. Gotlib is the chairman of the Study Steering Committee for the Novartis-sponsored
global trial of midostaurin in advanced systemic mastocytosis. He also receives funding
for administration of the clinical trial and payment for travel expenses from Novartis.
Dr. Gotlib also serves as the principal investigator on, and receives funding for a trial of
brentuximab vedotin in systemic mastocytosis.

M A L I G N A N T H E M A T O L O G Y 17

The Hematologist Compendium 2010-2015

Assessment and Management of Patients with ChemotherapyInduced Peripheral Neuropathy May/June 2014
UPDATE/COMMENTARY
The causes, presentation, and National Cancer InstituteCommon Terminology Criteria
for Adverse Events (NCI-CTCAE) classification of disability from chemotherapy-induced
neuropathy have not changed since the original publication of this article in 2014.
Unfortunately, no new proven therapies have been identified.U1 Herbal medicines are
also ineffective.U2 A pilot study of an electrical method (the Scrambler) was positive,
and phase III trials are being planned.U3

JANET L. ABRAHM, MD
Professor of Medicine at Harvard Medical School; Institute Physician at Dana-Farber Cancer
Institute, Boston, MA

U1. Schloss J, Colosimo M, Vitetta L. Herbal medicines and chemotherapy induced peripheral
neuropathy: a critical literature review. Crit Rev Food Sci Nutr. 2015. [Epub ahead of print].
U2. Majithia N, Temkin SM, Ruddy KJ, et al. National Cancer Institute-supported chemotherapyinduced peripheral neuropathy trials: outcomes and lessons. Support Care Cancer.
2016;24:1439-1447.
U3. Pachman DR, Weisbrod BL, Seisler DK, et al. Pilot evaluation of Scrambler therapy for the
treatment of chemotherapy-induced peripheral neuropathy. Support Care Cancer. 2015;23:943-951.

Diagnosis of CIPN is clinical, and nerve conduction studies, EMGs, and skin biopsies are
not needed for diagnosis or management. Chemotherapy doses are generally adjusted for
patients with grade 3 or greater neuropathy based on NCI-CTCAE.

NCI-CTC for Neuropathy


Peripheral Sensory Neuropathy

The Case

Grade 1: Asymptomatic or loss of deep tendon reflexes or paresthesia

The patient is a 54-year-old woman with IgA lambda multiple myeloma, receiving
dexamethasone, lenalidomide, and bortezomib therapy. She was healthy before her
diagnosis but had mild hypercalcemia and mild renal insufficiency at diagnosis, both of
which have resolved with treatment.

Grade 3: Severe symptoms limiting self-care ADL

She is about to begin her third treatment cycle, but reports painful (pain score of 7/10)
numbness and paresthesias in her fingers and feet. The former problem causes clumsiness
when using her touch-screen tablet and phone because she cannot accurately feel her fingers
touch the screens, and she needs the light on at night because she cannot reliably feel when
her feet touch the floor. Her Zubrod (ECOG) performance status is 1. She had similar, but less
severe, symptoms at the beginning of her last chemotherapy cycle, but she did not mention
them to anyone for fear that treatment would be delayed or discontinued.

Grade 2: Moderate symptoms limiting instrumental activities of daily living (ADL)


Grade 4: Life-threatening consequences; urgent intervention indicated
Grade 5: Death
Peripheral Motor Neuropathy
Grade 1: Asymptomatic clinical or diagnostic observations only; intervention not
indicated
Grade 2: Moderate symptoms limiting instrumental ADL
Grade 3: Severe symptoms limiting self-care ADL; assistive device indicated
Grade 4: Life-threatening consequences; urgent intervention indicated
Grade 5: Death

During her exam, she was anxious, alert, and oriented. She denied both spine pain and
bowel or bladder incontinence. Her exam was unremarkable except for the neurologic
examination. She had impaired proprioception, decreased vibration and light touch in her
hands to the wrists and feet to the ankles, 4/5 hand grip and finger strength bilaterally, and
4/5 ankle strength. Her ankle deep tendon reflexes (DTRs) were absent (flaccid).

The patient under discussion had grade 2 sensory neuropathy by these criteria, which
underestimated her disability and functional losses. The NCI-CTC are not as sensitive to
patient-reported symptoms or functional loss as are some other scoring systems, but no
consensus has yet been reached on a standard for grading neurotoxicity.4,5 Quality-of-life
scales may be more relevant (e.g., the CIPN 20 or FACT/GOG-Ntx).6

Laboratory studies showed normal potassium, BUN, creatinine, glucose, calcium, and
magnesium. B12 concentration and thyroid function studies were normal. Her paraprotein
concentration showed a continuing decrease in response to treatment.

Treatment

The Question
What is your approach to assessment and management of patients with chemotherapyinduced peripheral neuropathy?

My Response
Presentation/Physical Findings
The patients presentation is typical of chemotherapy-induced peripheral neuropathy (CIPN)
that is characterized by tingling (71%), numbness (58%), and paresthesias or dysesthesias
(45%), or spontaneous burning, cramping, or aching pain (40%) that is usually symmetric,
begins peripherally and progresses proximally.1 Patients often report allodynia (i.e., pain
from something that is usually not painful, like cold temperatures or the touch of clothing),
or hyperalgesia (i.e., experiencing more pain than usual from a mildly painful stimulus).
The mechanisms of axonal nerve injury vary by agent and include damage to mitochondria
(bortezomib, platinum compounds, vincristine, paclitaxel), glial cells and macrophages
(bortezomib, oxaliplatin, vincristine, paclitaxel), vasa vasorum (thalidomide, paclitaxel),
endoplasmic reticulum (bortezomib), or microtubules (bortezomib, taxanes, and vinca
alkaloids).2,3
CIPN induced by vinca alkaloids causes numbness, not pain, and loss of DTRs,
proprioception, and motor impairment, such as foot drop.2 Patients receiving thalidomide,
bortezomib, or platinum have a mainly sensory neuropathy, which may include loss of
vibration, proprioception, and DTRs.2 Taxanes cause both a sensory neuropathy and
proximal weakness.2 Only CIPN induced by thalidomide is irreversible.

Evaluation
In a patient whose CIPN appears only after chemotherapy, the evaluation should include
laboratory testing for B12 deficiency, hypothyroidism, hypocalcemia, hypomagnesemia,
and hyperglycemia (Figure 1). Because patients with leptomeningeal carcinomatosis or
spinal cord compression can present with these symptoms, an MRI of the spine with
contrast is recommended for patients with back pain. If the test is negative, an MRI of the
brachial or lumbosacral plexus, depending on the location of the symptoms and clinical
suspicion, may be warranted.
M A L I G N A N T H E M A T O L O G Y 18

Dose reduction or drug discontinuation is the only specific therapy for most CIPNs;
patients may tolerate higher doses of bortezomib if the drug is given by a subcutaneous
injection rather than by IV infusion. Symptomatic therapies for CIPN are limited (Figure 1).
Of the adjuvant agents effective for patients with neuropathic pain, only duloxetine (30-60
mg/day) has clearly shown efficacy for the CIPN induced by taxanes and platinum.7 Despite
the lack of proven efficacy of other agents in CIPN, insurance companies usually require
that patients first fail a trial of gabapentin or pregabalin before authorizing duloxetine.
Occupational therapy or physical therapy is recommended for patients with
grade 2 sensory or motor toxicity to improve function and quality of life. Patients should
also be assessed for depression, which often accompanies uncontrolled chronic pain.8
While vitamin E, vitamin B6, magnesium, calcium, acetyl-L-carnitine, glutamine,
glutathione, n-acetyl cysteine, omega-3 fatty acids, alpha lipoic acid, and cannabinoids
each have shown efficacy in non-randomized, non-placebo-controlled trials, none can be
recommended yet as standard therapy.1, 2, 9
Amitriptyline does not prevent or ameliorate chemotherapy-induced neuropathy from
vinca alkaloids, platins, or taxanes.1 Venlafaxine, gabapentin, the N-methyl-d-aspartate
(NMDA) receptor antagonist dextromethorphan, the oral anesthetics memantine and
mexiletine, and low-dose continuous capsaicin treatment are also ineffective.2
Opioids are recommended for CIPN patients with severe pain.10 Patients will usually require
a long-acting agent to provide basal pain relief (e.g., sustained-release morphine or a fentanyl
patch) and short-acting opioids for breakthroughs. Methadone (Figure 2) is particularly
helpful for basal pain relief. Methadone includes d- and l-isomers that act as opioid-receptor
agonists and NMDA receptor antagonists.11 Because NMDA antagonizes the activity of the
opiate receptors, blocking NMDA receptors enhances the analgesic effect of externally
administered opioids. Methadone also inhibits the reuptake of serotonin and norepinephrine;
therefore, it functions as a serotonin-norepinephrine reuptake inhibitor (SNRI).
Methadone is usually given orally bid to tid. Dose adjustment is needed for hepatic
failure but not for renal failure. The steady state is not reached until 72 hours after the
initial dose or after a dose increase. Therefore, short-acting opioids should be used to
control symptoms in the interim. If pain level falls to 3 within the first 24 to 48 hours of
initiating methadone, reduce the dose by half immediately to avoid excessive sedation
and respiratory depression that may otherwise occur in the next 24 hours. This delay in
reaching steady-state plasma levels, along with potential cardiac toxicity (prolongation
of the rate corrected QT [QTc] interval) and its interactions with inducers and inhibitors
of the cytochrome P450 system, CYP1A2, CYP3A4, and CYP2D6 make using methadone
safely somewhat complex. Palliative care specialists can be helpful in managing initiation

The Hematologist Compendium 2010-2015

Evidence-Based Algorithm for Evaluation and Symptomatic Treatment of Patients with CIPN
Figure 1

First, exclude spinal or leptomeningeal metastases and plexopathies and correct vitamin or hormone
abnormalities, if present. For patients who have NCI-CTC grade 3 CIPN, follow standard
protocol guidelines for dose adjustments of anti-neoplastic agents. Use duloxetine for patients with
unacceptable pain levels who are receiving either platinum or taxanes. For refractory pain, add an
opioid. If standard opioids are not effective, or if they cause excessive sedation, myoclonus, or other
side effects, consider using methadone as the basal opioid, and use the standard opioids for
breakthrough pain.

Figure 2

To safely prescribe methadone, monitor the QTc. If the QTc is prolonged, correct contributing
metabolic abnormalities or discontinue agents that can prolong the QTc. If the patient requires
concomitant therapy with a CYP3A4 competitor (such as voriconazole), use lower doses of
methadone. See text for other agents that require adjustments of methadone doses. Increase
methadone doses no more than every 72 hours.

*Despite the lack of proven efficacy, insurance companies may require that patients first fail a trial of gabapentin or
prebabalin before authorizing duloxetine.

1. Finnerup NB, Sindrup SH, Jensen TS. Management of painful neuropathies. Handb Clin Neurol.
2013;115:279-290.

or adjustment of methadone doses or for consulting on patients with seemingly refractory


pain syndromes.
Patients who, at baseline, have a prolonged QTc interval, or who need high doses of
methadone, have developed cardiac arrhythmias (torsades de pointes; polymorphic
ventricular tachycardia). Fatalities have been reported but were associated with doses
of > 600 mg per day. Common drugs used in hematology patients that also prolong the
QTc include the quinolone antibiotics (especially levofloxacin), typical and atypical
antipsychotics (such as haloperidol and olanzapine), selective serotonin reuptake
inhibitors (SSRIs) (but not SNRIs), and metoclopramide.
Methadones drug interactions are listed on websites including www.drugs.com or www.
qtdrug.org. Fluconazole, voriconazole, fluoxetine, and fluvoxamine raise methadone
levels. SSRIs may raise methadone levels in CYP2D6 rapid metabolizers but SNRIs
(e.g., venlafaxine) do not. Additionally, grapefruit juice and acute alcohol ingestion can
increase methadone levels. Drug levels of desipramine (or other tricyclic antidepressants)
and zidovudine increase when methadone is added. Antiretrovirals, phenobarbital,
carbamazepine, phenytoin, rifampin, somatostatin, spironolactone, and risperidone lower
the levels of methadone and have precipitated withdrawal symptoms.

Conclusion
The patient continued therapy, but bortezomib delivery was switched from IV infusion to
SQ injection. Her pain symptoms fell to tolerable levels on 2 mg tid of methadone such that
she was able to work, and her functional level was further enhanced by participation in an
occupational therapy program.

2. Ferrier J, Pereira V, Busserolles J, et al. Emerging trends in understanding chemotherapy-induced


peripheral neuropathy. Curr Pain Headache Rep. 2013;17:364-372.
3. Diezi M, Buclin T, Kuntzer T. Toxic and drug-induced peripheral neuropathies: updates on causes,
mechanisms and management. Curr Opin Neurol 2013;26:481-488.
4. Alberti P, Rossi E, Cornblath DR, et al. Physician-assessed and patient-reported outcome measures
in chemotherapy-induced sensory peripheral neurotoxicity: two sides of the same coin. Ann Oncol.
2014;25:257-264.
5. Driessen CML, de Kleine-Bolt KME, Vingerhoets AJJM, et al. Assessing the impact of chemotherapyinduced peripheral neurotoxicity on the quality of life of cancer patients. Support Care Cancer.
2012;20:877-881.
6. Grisold W, Cavaletti G, Windebank AJ. Peripheral neuropathies from chemotherapeutics and targeted
agents: diagnosis, treatment, and prevention. Neuro Oncol. 2012;14:iv45-iv54.
7. Smith EML, Pang H, Cirrincione C, et al. Effect of duloxetine on pain, function, and quality of life among
patients with chemotherapy-induced painful peripheral neuropathy: a randomized clinical trial. JAMA.
2013;309:1359-1367.
8. Glover J, Dibble SL, Dodd MJ, et al. Mood states of oncology outpatients: does pain make a difference? J
Pain Symptom Manage. 1995;10:120-128.
9. Schloss JM, Colosimo M, Airey C, et al. Nutraceuticals and chemotherapy induced peripheral neuropathy
(CIPN): a systematic review. Clin Nutr. 2013;32:888-893.
10. Dworkin RH, OConnor AB, Audette J, et al. Recommendations for the pharmacologic management of
neuropathic pain: an overview and literature update. Mayo Clin Proc. 2010;85:S3-S14.
11. Rhondali W, Tremellat F, Ledoux M, et al. Methadone rotation for cancer patients with refractory pain in
a palliative care unit: an observational study. J Pall Med. 2013;16:1382-1387.

Dr. Abrahm receives royalties from her book, A Physicians Guide to Pain and Symptom
Management in Cancer Patients, 2nd edition, 2005, from Johns Hopkins University
Press. She is the paid section editor for UpToDate on the topic of pain in palliative care
patients. She also receives expense reimbursement and honoraria from Knowledge to
Practice for her lectures on palliative care.

CIPN can be a disabling complication of chemotherapy. Patient education that encourages


reporting relevant symptoms, chemotherapy dose adjustment, occupational therapy,
physical therapy, and symptomatic therapy using duloxetine and opioids, including
methadone, can often help patients maintain or regain function and minimize pain and
disability.
M A L I G N A N T H E M A T O L O G Y 19

The Hematologist Compendium 2010-2015

Approach to the Evaluation of Patients with an IgM Monoclonal Protein


September/October 2014
UPDATE/COMMENTARY
Since this article was published there has indeed been a very important development
in multiple myeloma (MM) that has implications for IgM monoclonal proteins.
Historically, MM was defined by the presence of CRAB criteria (elevated calcium,
renal insufficiency, anemia and bony disease). However, three additional criteria
have now been added to this definition as endorsed by the International Myeloma
Working Group, published in late 2014.U1

pending organ damage, and if untreated, that damage may be permanent. It has
therefore shifted a small subset of smoldering myeloma patients to active myeloma
warranting treatment. I remember these three criteria with the acronym SLiM (60
percent plasma cells; more than 100 light chains involved/uninvolved; MRI evidence
of one or more focal lesions) now creating the overall acronym SLiM CRAB for
multiple myeloma.

These three criteria include clonal bone marrow plasma cell percentage of 60
percent; involved/uninvolved serum-free light chain (FLC) ratio 100 (involved FLC
level must be 100 mg/L); and more than one focal lesion on magnetic resonance
imaging (MRI) studies (at least 5 mm in size).

This new definition of MM, therefore, affects my approach to IgM monoclonal


proteins; in the algorithm, I ask the question, does the patient have end-organ
damage attributable to the IgM monoclonal protein? Instead of only considering
CRAB criteria, one should now consider SLiM CRAB criteria, as that would invoke
the IgM myeloma diagnosis if present, as opposed to IgM monoclonal gammopathy
of undetermined significance (MGUS).

Each of these criteria was validated by at least two large databases. They do reflect
a change in the concept that one must have established end organ damage to be
treated for myeloma; this is important, as patients with these three criteria have

U1. Rajkumar SV, Dimopoulos MA, Palumbo A, et al. International Myeloma Working Group updated
criteria for the diagnosis of multiple myeloma. Lancet Oncol. 2014;15:e538-e548.

JOSEPH MIKHAEL, MD, MEd, FRCPC


Professor of Medicine, Mayo College of Medicine, Mayo Clinic, Phoenix, AZ

The Question
What is your approach to the evaluation of patients with an IgM monoclonal protein?

My Response
Benign monoclonal proteins were first described by Dr. Jan Waldenstrm in 1960
after he detected abnormal narrow hypergammaglobulinemia bands in serum protein
electrophoresis (SPEP) samples from healthy individuals.1 The term MGUS was coined by
Dr. Robert Kyle in 1978 to describe an asymptomatic plasma cell dyscrasia characterized
by a monoclonal protein of <3 gm/dL, bone marrow plasma cells <10 percent, and the
absence of end-organ damage commonly associated with MM.2 MGUS is a relatively common
condition with a prevalence of three to five percent in the adult population over the age
of 50 years.3 SPEP is a frequently performed laboratory test ordered by primary-care
physicians evaluating patients with anemia, by nephrologists evaluating patients with renal
insufficiency (possibly with associated proteinuria), and by neurologists evaluating patients
with peripheral neuropathy. When the SPEP reveals a monoclonal protein, referral to a
hematologist usually follows. Therefore, it is part of routine practice for hematologists to
see patients with monoclonal proteins that are initially identified as the result of screening
assessments. The hematologist must then decide which additional diagnostic studies are
warranted, and based upon those results, develop management and follow-up plans.
Although it accounts for only 15 to 20 percent of MGUS cases (that also include IgGMGUS, IgA-MGUS, and light chain-MGUS), IgM-MGUS poses a unique diagnostic challenge
because of the association of monoclonal IgM proteins with B-cell lymphoproliferative
disorders (particularly Waldenstrm macroglobulinemia [WM], amyloidosis, and peripheral
neuropathy).4 This review is intended to provide the practicing hematologist with a focused
diagnostic approach to patients with a monoclonal IgM protein that takes into account its
associations with other disease processes.

MGUS

Figure 1

Three distinct classes of MGUS


Natural History of MGUS
are recognized: Non-IgM-MGUS
(essentially IgG-MGUS or IgA-MGUS
MGUS
as both IgD-MGUS and IgE-MGUS
are fleetingly rare), light chainMGUS, and IgM-MGUS5, 6 (Figure 1).
Non IgM
IgM
Light Chain
Distinguishing among these subgroups is important, as doing so
directs both the diagnostic plan and
MM +/WM
MM +/the follow-up recommendations,
Amyloid
Amyloid
Amyloid
IgM Myeloma
facilitates identification of diseases
associated with MGUS, and impacts
on management recommendations
and prognosis. Although the majority
of patients with IgM-MGUS will have a benign course, it is critical for the clinician to rule out a
concurrent associated disease and to monitor for progression or transformation into a distinct
entity that requires specific therapy.

The Risk of MGUS Progression


Unstratified patients with non-IgM MGUS have approximately a one percent per-year risk of
their disease transforming into MM; however, the risk of transformation is double in patients
M A L I G N A N T H E M A T O L O G Y 20

with IgM MGUS. Risk for transformation can be more precisely stratified based on three
parameters: IgG subtype versus non-IgG subtype, monoclonal protein concentration <1.5 gm/
dL versus 1.5 gm/dL, and normal versus abnormal serum free light chain ratio.7 Stratification
of risk can help guide the diagnostic evaluation and follow-up recommendations.
IgM MGUS not only has a higher risk of transformation than non-IgM MGUS, but also the
spectrum of diseases associated with IgM MGUS transformation is broader than that of nonIgM MGUS. Whereas non-IgM MGUS can progress into smoldering and active MM and AL
amyloidosis, IgM MGUS can transform into WM, AL amyloidosis, and less commonly, IgM
smoldering myeloma or IgM MM.8 For this reason, patients with IgM MGUS require closer
follow-up than patients with non-IgM MGUS, and from a conceptual perspective, IgM-MGUS
can be thought of as a lymphoproliferative MGUS while non-IgM MGUS behaves as a
plasma cell proliferative MGUS.

Special Concerns for IgM MGUS


a. Association with neuropathy Neurologists routinely screen patients with peripheral
neuropathy for the presence of a monoclonal protein, and approximately five to 10
percent of such patients will be found to have a monoclonal protein by SPEP. A causal
relationship between MGUS and peripheral neuropathy is supported by association of
peripheral neuropathy with other plasma cell dyscrasias including WM, MM, AL amyloid,
and POEMS (polyneuropathy, organomegaly, endoocrinopathy, monoclonal gammopathy,
skin changes). The peripheral neuropathy of MGUS is classically bilateral, peripheral,
and sensory with electrophysiologic studies showing a demyelinating pattern; and in
patients with progressive, debilitating disease, biopsy may reveal axonal loss. Although
peripheral neuropathy can occur with all forms of MGUS, it is most commonly associated
with IgM-MGUS. Demonstration of IgM anti-myelin-associated glycoprotein (MAG)
antibodies supports a causal relationship between IgM-MGUS and polyneuropathy but is
not essential for the diagnosis. Because peripheral neuropathy can be caused by other
processes that may co-exist with IgM-MGUS, the clinician is often faced with the dilemma
of whether to assign the neuropathy to MGUS, and if so, what to do about it. The decision
can be guided by the observations that the neuropathy associated with IgM-MGUS is
characteristically a relatively benign, slowly progressive sensory process (although some
cases can be severe and debilitating). Management is challenging as effective therapy
is lacking. A minority of patients respond to rituximab, and other immunomodulatory
treatments are generally ineffective.9 As is the case with other plasma cell dyscrasiaassociated neuropathies, the monoclonal protein concentration in IgM-MGUS-associated
peripheral neuropathy does not correlate with disease severity, arguing against the use
of myeloma-directed therapy to reduce the plasma cell burden as a treatment strategy for
the neuropathy of IgM-MGUS.
b. Association with AL amyloid This complex disease can be associated with any form of
myeloma, although usually one of the two processes dominates the clinical picture. That
is to say that patients typically have either a myeloma-phenotype manifested by some
combination of hypercalcemia, renal insufficiency, anemia, and skeletal involvement or
an amyloid phenotype characterized by organ (liver, heart, kidney) infiltration by the
pathologic immunoglobulin light chain. AL amyloid is more commonly associated with
IgM-MGUS than other forms of MGUS. Thus, it is imperative that AL amyloid be carefully
considered when evaluating patients with an IgM monoclonal protein.
c. Association with WM The hallmark of this disease is an IgM monoclonal protein,
lymphoadenopathy, hepatosplenomegally, and bone marrow involvement by
plasmacytoid lymphocytes. Frank disease may be preceded by a relatively asymptomatic
smoldering phase in which IgM MGUS is the only apparent clinical manifestation.
d. Association with other B-cell lymphoproliferative disorders Although IgM-MGUS is more
commonly associated with WM, it can be observed in association with another B-cell
lymphoproliferative disease such as CLL or non-Hodgkin lymphoma. As indicated in
the algorithm (Figure 2), determining if organ damage is attributable to the monoclonal
protein is critical, as in most cases the monoclonal protein simply co-exists with the
lymphoproliferative disorder. Although the monoclonal protein may be discovered first
as part of a laboratory evaluation, it is not usually the presenting clinical manifestation of
lymphoproliferative diseases other than WM.
The Hematologist Compendium 2010-2015

e. Association with IgM MM This is a rare form of myeloma that is genuinely distinct from
WM. IgM MM is distinguished from WM primarily by skeletal involvement (lytic bone
lesions) in the former, but genetic studies may also be informative (e.g., t[4:14] in IgM
MM and somatic mutation of MYD88 in WM).10, 11

Figure 2

Algorithm for Investigation of IgM Monoclonal Protein


IgM Monoclonal Protein Detected

Suggested Approach to the Evaluation of Patients


with an IgM Monoclonal Protein
The strategy that I use in the evaluation of patients with an IgM monoclonal protein is
described below and illustrated in Figure 2.
1. History This remains a critical aspect of the evaluation, as symptoms elicited from
a careful history focus attention on specific issues that require further investigation.
Indeed, even subtle symptoms become important when the differential diagnosis
includes such a wide spectrum of disorders as amyloidosis, POEMS, MM, and lymphoma.
Key symptoms to address include the following:
a. Constitutional (weight loss, extreme fatigue) Such symptoms suggest AL amyloid
or lymphoma.

History/Physical + Labs* + Skeletal Imaging** + Bone Marrow Evaluation

Does the patient have end organ damage attributable to the IgM Monoclonal Protein?
YES

Are there osteolytic


lesions or t(11;14)?

NO

Are there 10% Plasma Cells or


is the M Spike 3gm/dL?

Are there findings


consistent with AL
Amyloidosis?

YES

IgM Myeloma

Obtain echo and


fat aspirate +/organ biopsy

b. Gastrointestinal Upper GI bleeding, early satiety, and chronic diarrhea raise the
possibility of GI amyloid.

NO

IgM MGUS

If positive

c. Cardiac Progressive shortness of breath, presyncope/syncope, and chest pain


are consistent with cardiac amyloid.

Smoldering
Waldenstrm
Macroglobulinemia

d. Neurologic Bilateral, sensory neuropathy is consistent with the neuropathy


associated with plasma cell dyscrasias; and vision changes, headache, vertigo, or
dizziness raise the possibility of hyperviscosity associated with WM.

AL Amyloid

e. Skeletal pain This symptom suggests IgM myeloma.


f. Skin Urticarial rash raises the possibility of Schnitzler syndrome.12

Waldenstrm
Macroglobulinemia

2. Physical Exam The physical exam can be informative as patients with an IgM
monoclonal protein and WM or other non-Hodgkin lymphoma may present with
lymphadenopathy and heptosplenomegaly. Patients with concurrent AL amyloid may
have hepatosplenomegally or macroglossia, while patients with IgM myeloma may have
discrete sites of skeletal pain. A careful neurologic exam is essential for identifying and
characterizing a concurrent neuropathy.
3. Labs
a. The following lab studies should be obtained:
i. CBC, serum calcium, creatinine, -2-microglobulin, LDH, liver enzymes
ii. Serum protein electrophoresis with immunofixation electrophoresis
iii. Serum free light-chain assay
iv. Quantitative immunoglobulins
v. Spot urine sample for protein quantitation, protein electrophoresis, and
immunofixation electrophoresis. For patients with significant proteinuria
and patients with a monoclonal light chain (Bence Jones protein) detected
in the spot urine, a 24-hour urine sample should be collected to quantitate
total protein and light-chain excretion.
b. Other laboratory tests that should be considered based on the clinical
circumstances:
i. Cardiac function biomarkers (NT-proBNP and troponin) for suspected amyloid
ii. Anti-MAG antibodies in patients with a sensory neuropathy
iii. Peripheral blood flow cytometry for suspected CLL
4. Radiologic and Other Diagnostic Testing
a. Skeletal survey (radiographic or MRI myeloma protocol) is recommended for
patients with bone pain and for patients in whom IgM MM is a consideration.
b. Other diagnostic studies that may be considered based on clinical circumstances:
i. Abdominal US to assess for hepatic amyloid
ii. Echocardiogram and ECG to assess for cardiac amyloid
iii. Abdominal/pelvic CT to assess for non-Hodgkin lymphoma including WM
iv. PET scan to assess for lymphoma
5. Bone marrow evaluation Although many patients with MGUS do not require this
procedure, in higher-risk cases (monoclonal-protein concentration >1.5 gm/dL and
IgM-MGUS), bone marrow aspirate and biopsy is recommended and may also include
cytogenetic, myeloma FISH, flow cytometric analyses, and targeted gene sequencing.13 I
do not, however, recommended bone marrow analysis for all patients with a monoclonal
protein. Specifically, I may defer bone marrow analysis in the very elderly, in those in
whom the monoclonal-protein concentration is low (< 0.5 gm/dL), and in patients with an
inflammatory condition, as low-risk MGUS is relatively common in these settings.
a. Myeloma FISH and targeted gene sequencing Recommended if marrow findings
are consistent with MM or lymphoma. Finding t(11;14) by cytogenetics or FISH
suggests IgM MM, and mutant MYD88 supports a diagnosis of WM.
b. Flow cytometry This analysis is informative in patients suspected of having a
B-cell lymphoproliferative disorder including CLL, non-Hodgkin lymphoma, or WM.
6. Fat pad aspirate I do not routinely subject patients to this procedure; however, if
there is clinical suspicion of AL amyloid, fat pad aspirate with congo red staining is an
excellent screening tool. I have a slightly higher index of suspicion for AL amyloid when
the IgM monoclonal protein is lambda-restricted. If the fat pad aspirate is negative for
AL amyloid but suspicion remains high, targeted organ biopsy with congo red staining is
warranted.
M A L I G N A N T H E M A T O L O G Y 21

* Laboratory studies include CBC, serum calcium, creatinine, b-2microglobulin, LDH, liver enzymes, serum protein electrophoresis with
immunofixation, serum free light chain assay, quantitative immunoglobulins,
24-hour urine testing for protein, and urine electrophoresis.
** Skeletal survey is the preferred first-line imaging choice.

Marrow evaluation includes immunohistochemical studies, flow cytometry,


conventional cytogenetics, and myeloma FISH.

Confirm by MYD88 sequence analysis.

Follow-Up
If after the initial evaluation a patient is diagnosed with IgM MGUS, regular surveillance is
imperative as risk of progression/transformation is a continuous variable. I recommend
that patients be seen twice annually with a clinical assessment, along with the following
laboratory studies: CBC, comprehensive metabolic panel, SPEP, serum-free light-chain assay,
and quantitative immunoglobulins.14 I do not recommend repeat imaging or bone marrow
analysis unless there is suspicion of progression.

Conclusion
Monoclonal IgM is associated with a diverse set of diseases that range from a generally
benign process requiring no specific therapy (IgM-MGUS) to overt disease requiring a specific
management plan. Approaching the initial evaluation systematically and comprehensively
will enable the clinician to accurately characterize the disease process.
1. Waldenstrom J. Studies on conditions associated with disturbed gamma globulin formation
(gammopathies). Harvey Lect. 1960;56:211-231.
2. Kyle RA. Monoclonal gammopathy of undetermined significance. Natural history in 241 cases. Am J Med.
1978;64:814-826.
3. Kyle RA, Therneau TM, Rajkumar V, et al. Prevalence of monoclonal gammopathy of undetermined
significance. N Engl J Med. 2006;354:1362-1369.
4. Kyle RA, Therneau TM, Rajkumar SV, et al. Long-term follow-up of IgM monoclonal gammopathy of
undetermined significance. Blood. 2003;102:3759-3764.
5. Rajkumar SV, Kyle RA, Buadi FK. Advances in the diagnosis, classification, risk stratification, and
management of monoclonal gammopathy of undetermined significance: implications for recategorizing
disease entities in the presence of evolving scientific evidence. Mayo Clin Proc. 2010;85:945-948.
6. van de Donk NW, Palumbo A, Johnsen HE, et al. The clinical relevance and management of monoclonal
gammopathy of undetermined significance and related disorders: recommendations from the European
Myeloma Network. Haematologica. 2014;99:984-996.
7. Rajkumar SV, Kyle RA, Therneau TM, et al. Serum free light chain ratio is an independent risk factor for
progression in monoclonal gammopathy of undetermined significance. Blood. 2005;106:812-817.
8. Baldini L, Goldaniga M, Guffanti A, et al. Immunoglobulin M monoclonal gammopathies of undetermined
significance and indolent Waldenstroms macroglobulinemia recognize the same determinants of
evolution into symptomatic lymphoid disorders: proposal for a common prognostic scoring system. J
Clin Oncol. 2005;23:4662-4668.
9. Latov N. Diagnosis and treatment of chronic acquired demyelinating polyneuropathies. Nat Rev Neurol.
2014;10:435-446.
10. Schuster SR, Rajkumar, Dispenzieri A, et al. IgM multiple myeloma: disease definition, prognosis, and
differentiation from Waldenstroms macroglobulinemia. Am J Hematol. 2010;85:853-855.
11. Treon SP, Xu L, Yang G, et al. MYD88 L265P somatic mutation in Waldenstroms macroglobulinemia. N
Engl J Med. 2012;367:826-833.
12. Simon A, Asil B, Braun-Falco M, et al. Schnitzlers syndrome: diagnosis, treatment, and follow-up. Allergy.
2013;68:562-568.
13. Kyle RA, Durie BG, Rajkumar SV, et al. Monoclonal gammopathy of undetermined significance (MGUS)
and smoldering (asymptomatic) multiple myeloma: IMWG consensus perspectives risk factors for
progression and guidelines for monitoring and management. Leukemia. 2010;24:1121-1127.
14. Korde N, Kristinsson SY, Landgren O. Monoclonal gammopathy of undetermined significance (MGUS)
and smoldering multiple myeloma (SMM): novel biological insights and development of early treatment
strategies. Blood. 2011;117:5573-5581.

Dr. Mikhael indicated no relevant conflicts of interest.

The Hematologist Compendium 2010-2015

Treatment of Elderly Patients with Acute Myeloid Leukemia January/February 2015


Due to the recent publication date of this article, an update has not been requested.

These data suggest that age alone is a poor predictor of treatment intent and outcomes,
and is likely a surrogate for other covariates associated with inferior prognosis.
BRUNO MEDEIROS, MD
Associate Professor of Medicine (Hematology) at the Stanford Cancer Institute, Stanford, CA

The Question
What is your approach to the treatment of elderly patients with acute myeloid leukemia?

My Response
Acute myeloid leukemia (AML) is not a significant public health hazard, accounting for less
than 2 percent of all cancers diagnosed yearly in the United States. The American Cancer
Society estimates that 18,860 new cases of AML were diagnosed in 2014.1 Nonetheless,
AML is second only to chronic lymphocytic leukemia as the most common subtype of
leukemia in adults. The median age at diagnosis is 67 years, and more than 60 percent of
newly diagnosed patients are older than 60 years.2 The management of elderly patients
with AML poses unique therapeutic challenges. These individuals disproportionally
account for greater than 75 percent of AML deaths yearly. In addition, recent data from the
Surveillance, Epidemiology, and End Results (SEER) Program demonstrate that 50 to 60
percent of newly diagnosed AML patients older than 65 years do not receive any form of
antileukemia therapy.3

Therapeutic Options for Elderly Patients with AML


In routine clinical practice, it remains challenging to know how to best utilize these data to
inform patients and families on the optimal therapeutic approach for an individual with AML.
Although these predictive models provide some guidance on the potential risks and benefits
associated with conventional induction chemotherapy, treatment decisions regarding therapeutic intensity remain a highly individualized exercise. In my practice, I direct patients with
low treatment-related mortality scores or favorable AML risk prediction scores to intensive
standard or investigational therapy; conversely, I consider patients with high treatmentrelated mortality scores or unfavorable AML risk profiles for lower-intensity investigational
regimens (Figure).
Results from population studies and randomized clinical trials add to the challenge of deciding optimal therapeutic intensity. Recent SEER-Medicare linked data showed that, after
using a propensity score-matched survival analysis to account for all confounders, similar
reductions in the risk of death were observed for elderly patients receiving either intensive
chemotherapy or hypomethylating agents.3 Similar results have been reported in randomized clinical trials with similar overall survival between single agent hypomethylating agents
(decitabine or azacitidine) and conventional care regimens.9,10

The Future is Now: The Rise of Targeted Therapies


It was the best of times, it was the worst of times, it was the age of wisdom, it was the age of
foolishness... This opening sentence from A Tale of Two Cities describes the sentiment of
hematologists regarding the current state of management of AML in older patients. While
our understanding of the molecular alterations responsible for leukemogenic transformation has flourished in recent years, none of these discoveries has yet translated into
approved treatment strategies for patients beyond "7+3" conventional induction chemotherapy. However, characterization of the mutational landscape of AML has accelerated development of targeted therapeutics that may soon be within the reach of patients. Activating
mutations in signaling pathways, including FLT3, KIT, RAS, and others, have been described
in roughly 60 percent of AML patients.11 Clinical trials testing the activity of FLT3, KIT and
MEK inhibitors (as single agents or in combinations) are available in most academic cen-

Definition of Elderly in Patients with AML

Older patients can be classified into three categories of chronological age: 1) young-old
patients are 65 to 75 years of age; 2) old patients are 76 to 85 years of age; and 3) oldestold patients are older than 85 years.4 In AML, clinical outcomes worsen with advancing
age.2 Recognizing this relationship between older age and outcomes, contemporary AML
treatment protocols have typically used an arbitrary cutoff of 55 to 65 years to distinguish
between younger and older subjects. However, chronologic age by itself is not reliable in estimating life expectancy, functional reserve, or the risk of treatment complications. Comprehensive geriatric assessments (CGA) are multidisciplinary, in-depth evaluations designed
to assess life expectancy and attendant morbidity and mortality
risks in older patients. CGAs include tools to predict the
functional age based on functional status, comorbidities,
Figure
polypharmacy, nutritional status, and geriatric syndromes.5
Treatment Algorithm
For this reason, pre-treatment CGAs or more focused geriatric assessment tools may replace chronologic age as an
New diagnosis of AML
identifier of functionally older AML patients ineligible for
aggressive treatment approaches.

AML and NonAML-Related Factors


Influencing Treatment Decisions
Recent studies suggest that age may be a suboptimal sole
criterion for allocation to intensive AML treatment protocols, and that additional variables may improve the ability
to predict toxicity and outcomes. Population-based data
demonstrate that older age was only one of the several
covariates (including history of antecedent hematologic disorder [sAML], higher comorbidity score, poor performance
indicators, marital status, and lower household incomes)
associated with lack of antileukemia therapy in newly diagnosed older patients with AML.3 A retrospective analysis
showed that a pre-treatment geriatric assessment, focused
on cognitive and physical function, improved the prediction
of survival among older adults with AML treated with conventional induction chemotherapy.6 Two large retrospective
studies have identified several AML and non-AML covariates
that predict early outcomes, such as induction mortality and
likelihood of complete remission, following intensive chemotherapy. In the first, investigators developed a simplified
early death score with moderate discriminatory power that
incorporates performance status at diagnosis, age, platelet
count, albumin, sAML, white blood cell count, percentage
of peripheral blood blasts, and serum creatinine (cstaging.
fhcrc-research.org/TRM/Default.aspx).7 Interestingly, removal
of age as a covariate had minimal impact on the predictive
power of this model. Similarly, investigators from the Study
Alliance Leukemia demonstrated that the risk of induction
mortality and the chance of complete remission could be
predicted in older patients with AML using a combination of
pre-treatment covariates, including age, hemoglobin, platelet count, fibrinogen, type of AML, karyotype and limited
molecular abnormalities at diagnosis (www.aml-score.org).8

for Older Patients with AML

Age

Patient-related
characteristics

Performance status
Renal and hepatic function
Coagulopathy

High treatment-related
mortality score or
unfavorable AML score

Karyotype

AML-related
characteristics

sAML (t-AML or AML-MRC)


Blood counts at diagnosis
(WBC, Hb, platelets)

Acceptable treatment-related
mortality score or favorable
AML score

Non-intensive treatment
Vulnerable

Focused geriatric
assessment or
comorbidity index

Investigational therapy
Hypomethylating agents
Low-dose cytarabine

Fit

Intensive treatment
Investigational therapy
Conventional induction therapy

Abbreviations: AML, acute myeloid leukemia; sAML, antecedent hematologic disorder; t-AML, therapy-related AML; AML-MRC, AML with
myelodysplastic syndrome--related changes.

M A L I G N A N T H E M A T O L O G Y 22

The Hematologist Compendium 2010-2015

ters and may reshape treatment strategies for these patients. Mutations in the DNMT3A
and TET2 genes have been linked to increased sensitivity to hypomethylating agents.12,13
Examples of novel agents in clinical development include small molecule inhibitors of mutant IDH1 and IDH2,14,15 DOT1-L inhibitors for patients with MLL rearranged acute leukemia,
novel epigenetic modifiers such as the bromodomain (BRD4) inhibitors, and development
of second-generation anti-CD33 conjugated monoclonal antibodies. Also, addition of multikinase (sorafenib) or aurora kinase inhibitors to conventional induction chemotherapy is
associated with promising antileukemic activity and improved survival. Finally, improved
patient selection (genomic profile, AML-MRD, or adverse-risk karyotype) or combination
with novel agents (pracinostat or pevonedistat) may optimize the clinical benefit of DNA
methyltransferase inhibitors such as azacitidine.
At last, physicians caring for older patients with AML may say farewell to the age of foolishness when patients and physicians were painstakingly faced with choosing between nonspecific and toxic regimens or palliative care. Instead, they can usher in the age of wisdom
wherein genomic-driven targeted and risk-adapted treatment strategies are incorporated into
the management of AML. Current enthusiasm in the field is married to the promise that the
best of times will soon be upon us.

1. American Cancer Society. Cancer Facts & Figures 2014. www.cancer.org/research/cancerfactsstatistics/


cancerfactsfigures2014. Accessed December 2014.
2. National Cancer Institute. SEER Stat Fact Sheets. seer.cancer.gov/statfacts/html/amyl.html. Accessed
November 2013.
3. Medeiros BC, Satram-Hoang S, Hurst D, et al. Big data analysis of treatment patterns and outcomes
among elderly acute myeloid leukemia patients in the United States. 2014. Manuscript in preparation.
4. Hurria A, Browner IS, Cohen HJ, et al. Senior adult oncology. J Natl Compr Cancer Netw. 2012;10:162-209.
5. Extermann M, Hurria A. Comprehensive geriatric assessment for older patients with cancer. J Clin Oncol.
2007;25:1824-1831.
6. Klepin HD, Geiger AM, Tooze JA, et al. Geriatric assessment predicts survival for older adults receiving
induction chemotherapy for acute myelogenous leukemia. Blood. 2013;121:4287-4294.
7. Walter RB, Othus M, Borthakur G, et al. Prediction of early death after induction therapy for newly
diagnosed acute myeloid leukemia with pretreatment risk scores: a novel paradigm for treatment
assignment. J Clin Oncol. 2011;29:4417-4423.
8. Krug U, Rllig C, Koschmieder A, et al. Complete remission and early death after intensive chemotherapy
in patients aged 60 years or older with acute myeloid leukaemia: a web-based application for prediction
of outcomes. Lancet. 2010;376:2000-2008.
9. Kantarjian HM, Thomas XG, Dmoszynska A, et al. Multicenter, randomized, open-label, phase III trial of
decitabine versus patient choice, with physician advice, of either supportive care or low-dose cytarabine
for the treatment of older patients with newly diagnosed acute myeloid leukemia. J Clin Oncol.
2012;30:2670-2677.
10. Dombret H, Seymour JF, Butrym A, et al. Results of a phase 3, multicenter, randomized, open-label study
of azacitidine (AZA) VS conventional care regimens (CCR) in older patients with newly diagnosed acute
myeloid leukemia (AML). Paper presented at 19th Congress of the European Hematology Association.
June 12-15, 2014. Milan, Italy.
11. Cancer Genome Atlas Research Network. Genomic and epigenomic landscapes of adult de novo acute
myeloid leukemia. N Engl J Med. 2013;368:2059-2074.
12. Metzeler KH, Walker A, Geyer S, et al. DNMT3A mutations and response to the hypomethylating agent
decitabine in acute myeloid leukemia. Leukemia. 2012;26:1106-1107.
13. Bejar R, Lord A, Stevenson K, et al. TET2 mutations predict response to hypomethylating agents in
myelodysplastic syndrome patients. Blood. 2014;124:2705-2712.
14. Study of orally administered AG-120 in subjects with advanced hematologic malignancies with an IDH1
mutation. ClinicalTrials.gov Identifier: NCT02074839.
15. Phase 1 study of AG-221 in subjects with advanced hematologic malignancies with an IDH2 mutation.
ClinicalTrials.gov Identifier: NCT01915498.

Dr. Medeiros indicated no relevant conflicts of interest.

Check Out the ASH Image Bank


The ASH Image Bank is a public web-based image
library that offers a comprehensive collection of
high-quality, peer-reviewed images relating to a wide
range of hematologic topics. The Image Bank was
re-launched March 2016 and contains more than
2,100 images. Recently added features include the
Atlas, which provides a quick way to view images
of blood cells in a normal and abnormal state, and
reference cases describing common blood disorders
through image sets and descriptive text. ASH
encourages new submissions, subject to review
by the Image Bank Editor, and new images are
continuously being published to the collection. To
browse the Image Bank, submit, or download images,
members need to log in with their standard ASH
username and password and nonmembers need to
create an account, currently free of charge. To submit
comments, questions, and requests for the inclusion
of specific images, send an email to imagebank@
hematology.org. To view the ASH Image Bank, go to
imagebank.hematology.org.

M A L I G N A N T H E M A T O L O G Y 23

The Hematologist Compendium 2010-2015

Treatment of Relapsed/Refractory Hairy Cell Leukemia July/August 2015


Due to the recent publication date of this article, an update has not been requested.

ROBERT J. KREITMAN, MD
Chief, Clinical Immunotherapy Section, Laboratory of Molecular Biology, National Cancer
Institute, National Institutes of Health, Bethesda, MD

The Question
What is your management approach to patients with relapsed/refractory hairy cell leukemia?

My Response
Hairy cell leukemia (HCL) is an indolent B-cell malignancy, originally with a median survival of
approximately four years.1 Single-agent purine analog therapy with pentostatin or cladribine
achieves complete remission (CR) rates of 70 to 90 percent and median relapse-free survivals
in excess of 15 years.2-4 However, minimal residual disease
(MRD) often remains, and may be associated with relapse,
Figure
particularly in young patients with longer follow-up.5 Thus,
in the past 25 to 30 years, many patients have neither
died nor been cured, leading to the increasingly common
question of how to treat relapsed and refractory patients.
The answer requires accurate diagnosis, understanding
the history of response and toxicity to prior agents, and
a strategy for prioritizing options including exciting new
targeted therapies.

Diagnosis of HCL and Recognition of


Variant Disease

both regimens and allows crossover to the other regimen if needed. Most can achieve MRDnegative CR, but with more toxicity than patients receiving non-chemotherapy approaches
such as moxetumomab pasudotox.

Targeted Approaches in HCL


In the BRAF pathway of normal cells, BRAF phosphorylates MEK, phospho-MEK phosphorylates
ERK, and phospho-ERK leads to cell proliferation. BRAF containing the V600E mutation,
present in nearly half of malignant melanomas, in lower percentages of other tumors, but in
nearly all cases of classic HCL, exhibits uncontrolled phosphorylation leading to the malignant
phenotype. Inhibition of this mutant BRAF with vemurafenib, or with dabrafenib combined
with the MEK inhibitor trametinib, is approved for malignant melanoma and is currently being
tested in HCL. Oral BRAF inhibitors in HCL can rapidly reverse severe cytopenias but have not
yet been reported to clear MRD and prevent relapse. The oral agent ibrutinib, which targets the
Brutons tyrosine kinase pathway, is also being tested in patients with HCL and HCLv. Though
most patients achieve stable disease, this can have palliative benefit, particularly for patients
with HCLv.

Summary and Algorithm for Treatment


Approach to Patients with Relapsed/
Refractory HCL
The Figure shows our algorithm for prioritization of clinical
trials for relapsed HCL; additional options can be used
off-protocol. Patients with once-relapsed HCL after purine
analog can be retreated with the same or different purine
analog as a single-agent, particularly if the first remission
duration was long, but we prefer combining cladribine with
rituximab on study to eliminate MRD. For patients with
multiply relapsed HCL, moxetumomab pasudotox is our
preferred option due to its unique ability to achieve MRDnegative CR without chemotherapy toxicities. For patients
who responded to anti-CD22 recombinant immunotoxins
(BL22 or moxetumomab pasudotox) but need additional
therapy, we target CD25 with recombinant immunotoxin
LMB-2. For patients ineligible for moxetumomab
pasudotox or LMB-2, we use BRAF or BRAF/MEK inhibition,
provided patients are able to handle toxicities and
understand that remission durations may be limited if
MRD persists. For patients with HCL/HCLv ineligible for
these approaches, we prefer either the ibrutinib protocol
or our protocol using rituximab combined with either
bendamustine or pentostatin. Ibrutinib offers oral therapy
without chemotherapy toxicity, while the rituximabchemotherapy combination offers a likely path to high
MRD-negative CR rates and the potential (as in patients
after moxetumomab pasudotox) of not needing therapy for
many years. Further follow-up and testing will be needed to
determine the long-term benefit of these approaches and
whether MRD-negative CR can translate into cure.

HCL diagnosis today is most accurately made by flow


cytometry showing strong expression of B-cell antigens
CD20 and CD22; strong CD11c; and positive CD25, CD103,
and CD123.6 Bone marrow immunohistochemistry is
positive for CD20, tartrate-resistant acid phosphatase
(TRAP), annexin 1A (Anxa1), and for the V600E mutated
form of BRAF.7 Variant HCL (HCLv) has a similar
Newly diagnosed HCL patients typically receive a purine analog, most
immunophenotype but lacks CD25, Anxa1, CD123,
commonly cladribine in five daily doses, less commonly pentostatin in six
6
and TRAP. Additionally, BRAF is wild type. HCLv is
to 12 biweekly doses. Newly diagnosed or once-relapsed patients may
more aggressive, with worse splenomegaly, malignant
receive repeat courses of single-agent purine analog, but at MD Anderson
and the NIH, are also eligible for protocol treatment with rituximab and
lymphocytosis rather than cytopenias, and exhibits a
cladribine (CDAR). At the NIH, multiply relapsed HCL/HCLv patients
poor response to single-agent purine analog; it also may
eligible for moxetumomab pasudotox receive that option (also available
require adding rituximab. Although classified in 2008 by
in other centers), while ineligible patients may receive LMB-2, BRAF
the World Health Organization as a separate disorder,
inhibition, ibrutinib, and BRPR (bendamustine-rituximab vs pentostatinHCLv is also discussed here. Another aggressive variant is
rituximab) in that order of priority. Common off-protocol options for
characterized by an immunophenotype of HCLv or classic
multiply-relapsed HCL include repeat courses of single-agent purine
analog, single-agent rituximab, and pentostatin-rituximab. Splenectomy
HCL, but with the IGHV4-34 type of immunoglobulin
and even splenic radiation can have palliative benefit at least for a limited
rearrangement that is more than 98 percent homologous
period of time.
(unmutated) to its germline sequence.8 Even with the
classic immunophenotype, these patients resemble those
with HCLv with malignant lymphocytosis, purine analog
resistance, and wild-type BRAF.8,9 By shipping blood samples and unstained bone marrow
1. Golomb HM, Catovsky D, Golde DW. Hairy cell leukemia: a clinical review based on 71 cases. Ann Intern
slides to an expert center, accurate diagnosis is also possible without travel. Treatment
Med. 1978;89:677-683.
should only be initiated after accurate diagnosis is established.
2. Grever M, Kopecky K, Foucar MK, et al. Randomized comparison of pentostatin versus interferon alfa-2a in
previously untreated patients with hairy cell leukemia: an intergroup study. J Clin Oncol. 1995;13:974-982.

Treatment of Patients in First Relapse


With some exceptions, remission rates are lower after second-line versus after first-line purine
analog therapy3,4; however, if the length of the first remission is at least two to four years, many
will recommend repeating the initial purine analog. The shorter the treatment-free interval, the
greater the risk of overlapping toxicity. Patients relapsing after shorter intervals often receive
the anti-CD20 MAb rituximab alone in four to eight weekly doses. While single-agent rituximab
can achieve response without overlapping chemotherapy toxicity, response rates are relatively
low in patients with prior purine analog therapy who have cytopenias requiring treatment.
A more effective strategy is to combine a purine analog with rituximab. Elimination of MRD,
detectable by immunohistochemistry of the bone marrow biopsy, as well as by flow cytometry
of the blood or bone marrow aspirate, is possible with rituximab one month after cladribine.10
We treat once-relapsed HCL/HCLv with rituximab, starting immediately with cladribine to effect
synergy,11 or delayed at least six months so MRD status can be determined.

Therapy of Multiply Relapsed HCL


Several options are being tested for multiply relapsed HCL. In order to avoid chemotherapy
toxicities, but to retain cytotoxic power, we use recombinant immunotoxins containing an Fv
fragment of a Mab and a truncated form of Pseudomonas exotoxin. High CR rates in multiply
relapsed HCL were reported in 2001 using the anti-CD22 recombinant immunotoxin BL22,12
including patients with HCLv. Mutations were made in the VH domain of BL22 that improve its
binding to CD22, and the resulting affinity-matured recombinant immunotoxin moxetumomab
pasudotox achieved CRs in approximately 50 percent of patients with multiply relapsed
HCL, many without MRD.13 Another strategy to achieve MRD-negative CR is to use six cycles
of either pentostatin plus rituximab,4 or bendamustine plus rituximab.14 Our protocol at the
National Institutes of Health (NIH) prospectively evaluates and randomizes patients between
M A L I G N A N T H E M A T O L O G Y 24

3. Saven A, Burian C, Koziol JA, et al. Long-term follow-up of patients with hairy cell leukemia after cladribine
treatment. Blood. 1998;92:1918-1926.
4. Else M, Dearden CE, Matutes E, et al. Long-term follow-up of 233 patients with hairy cell leukaemia,
treated initially with pentostatin or cladribine, at a median of 16 years from diagnosis. Br J Haematol.
2009;145:733-740.
5. Rosenberg JD, Burian C, Waalen J, et al. Clinical characteristics and long-term outcome of young hairy cell
leukemia patients treated with cladribine: a single-institution series. Blood. 2014;123:177-183.
6. Matutes E, Wotherspoon A, Catovsky D. The variant form of hairy-cell leukaemia. Best Pract Res Clin
Haematol. 2003;16:41-56.
7. Tiacci E, Trifonov V, Schiavoni G, et al. BRAF mutations in hairy-cell leukemia. N Engl J Med.
2011;364:2305-2315.
8. Arons E, Suntum T, Stetler-Stevenson M, et al. VH4-34+ hairy cell leukemia, a new variant with poor
prognosis despite standard therapy. Blood. 2009;114:4687-4695.
9. Xi L, Arons E, Navarro W, et al. Both variant and IGHV4-34-expressing hairy cell leukemia lack the BRAF
V600E mutation. Blood. 2012;119:3330-3332.
10. Ravandi F, OBrien S, Jorgensen J, et al. Phase 2 study of cladribine followed by rituximab in patients with
hairy cell leukemia. Blood. 2011;118:3818-3823.
11. Kreitman RJ, Wilson W, Calvo KR, et al. Cladribine with immediate rituximab for the treatment of patients
with variant hairy cell leukemia. Clin Cancer Res. 2013;19:6873-6881.
12. Kreitman RJ, Wilson WH, Bergeron K, et al. Efficacy of the anti-CD22 recombinant immunotoxin BL22 in
chemotherapy-resistant hairy-cell leukemia. N Engl J Med. 2001;345:241-247.
13. Kreitman RJ, Tallman MS, Robak T, et al. Phase I trial of anti-CD22 recombinant immunotoxin
moxetumomab pasudotox (CAT-8015 or HA22) in patients with hairy cell leukemia. J Clin Oncol.
2012;30:1822-1828.
14. Burotto M, Stetler-Stevenson M, Arons E, et al. Bendamustine and rituximab in relapsed and refractory
hairy cell leukemia. Clin Cancer Res. 2013;19:6313-6321.

Dr. Kreitman is the co-inventor on the NIH patent for Moxetumomab pasudotox.
The Hematologist Compendium 2010-2015

Diagnosis of Immune Pathophysiology in Patients with Low-Risk


Myelodysplastic Syndromes September/October 2015
Due to the recent publication date of this article, an update has not been requested.

patients owing to age-related fatty changes in the BM. Therefore, hematologists should
keep in mind that BM hypercellularity does not necessarily preclude a diagnosis of
immune-mediated BM failure.

SHINJI NAKAO, MD, PHD


Professor of Cellular Transplantation Biology (Hematology/Respirology), Kanazawa
University Graduate School of Medical Science, Kanazawa, Japan

Laboratory Markers Representing Immune Pathophysiology of


BM Failure

The Question

The presence of predominant thrombocytopenia is the most fundamental feature of immunemediated BM failure that should prompt further examinations of other laboratory markers.
An increased percentage of paroxysmal nocturnal hemoglobinuria (PNH) type cells,
which can be detected using high-sensitivity flow cytometry (FCM), represents a reliable
marker of immune pathophysiology.7,8 The usefulness of detecting increased PNH-type
cells for predicting the response to immunosuppressive therapy (IST) in AA patients was
confirmed in recent prospective studies utilizing high-sensitivity FCM-based methods that
can precisely measure 0.01 percent or more PNH-type cells.9,10 Figure 1 shows the results
of FCM in the current case, which revealed 0.126 percent of granulocytes and 0.001 percent
of erythrocytes to be PNH-type cells. Such results can be obtained on the same day as the
patients visit, and if those results are positive, physicians have the option of commencing
IST shortly thereafter. The presence of HLA-DRB1*15:01 has been reported to predict a good
response to IST in patients with MDS.11 However, in our previous study using a multivariate
analysis of several factors, including the presence or absence of DRB1*15:01, only the
detection of PNH-type cells was associated with a good response to IST in patients with AA.12
Therefore, DRB1 typing to clarify the immune pathophysiology of BM failure is unnecessary
if high-sensitivity FCM is available and detects PNH-type cells.

What is your approach to the diagnosis and treatment of patients with low-risk
myelodysplastic syndrome (MDS) related to immune pathophysiology?

Case
A 53-year-old man was found to have pancytopenia during follow-up for hyperuricemia.
The laboratory findings were as follows: WBC, 2.44109/L with 26 percent neutrophils,
1 percent eosinophils, 8 percent monocytes, and 65 percent lymphocytes; RBC,
1.491012/L; hemoglobin, 5.7 g/dL; mean corpuscular volume (MCV), 112.8 fL; platelets,
22109/L; reticulocytes, 45109/L; LDH=195 IU/L. The patients bone marrow (BM) was
normocellular with 2 percent blasts and showed signs of dysplasia in erythroid precursors
and granulocytes without increased ring sideroblasts. The megakaryocyte count was
decreased. The karyotype of the BM cells was 46,XY in 20 dividing cells. The patient was
therefore diagnosed as having refractory cytopenia with multilineage dysplasia (RCMD)
with an International Prognostic Scoring System (IPSS) risk group of intermediate-1 (IPSSrevised risk category: intermediate). His physician suggested that he receive red blood cell
transfusions and possibly hematopoietic stem cell transplantation or azacitidine therapy if
the pancytopenia progressed further.

Figure 1

Detection of PNH-type cells using high sensitivity flow cytometry


Q2

Q2

10

10

0.001%

P5
10

10

P6
10

CD11bPE-A

Q1
5

0.126%

10

Q1

10

10

Diagnosing MDS is often challenging in patients


with pancytopenia or bicytopenia who do
not show definitive poor prognostic markers,
such as karyotypic abnormalities or increased
blasts or ring sideroblasts in the BM, as in
the present case. The differential diagnosis of
such cases of gray zone BM failure includes
refractory cytopenia with unilineage dysplasia,
RCMD, idiopathic cytopenia(s) of undetermined
significance, and moderate aplastic anemia
(AA).1 It is generally difficult to make an exact
diagnosis based on the established diagnostic
criteria because these disease entities overlap,
and each diagnosis relies on a subjective
judgement of the cell morphology by physicians
and/or pathologists.2-4 Giving a disease name
to such an equivocal condition has virtually
little value. Instead, it is clinically relevant to
distinguish benign subsets of BM failure that
are likely to respond to immunosuppressive
therapy (IST) from cases of nonimmunemediated BM failure, including those associated
with pre-leukemic features and inherited BM
failure syndromes.

Glytophorin APE-A

Distinguishing Cases of ImmuneMediated BM Failure

My Response

Another solid marker of immune pathophysiology in patients with BM failure is the


presence of HLA-A allelelacking leukocytes (HLA-LLs) derived from HSCs that develop
copy numberneutral loss of heterozygosity of the HLA haplotype owing to uniparental

Q3
1

0 10

Q4
2

10

10

10

10

Q3
M

-1,639

Q4
0

10

10

FLAER FITC-A

CD55+59 FITC-A

Granulocytes

Erythrocytes

10

Detection of paroxysmal nocturnal hemoglobinuria (PNH) -type cells using high-sensitivity flow cytometry. The patient possessed increased PNH-type cells
only in granulocytes.

Difficulties in Evaluating BM Cellularity in Patients with BM Failure


BM aspiration and trephine biopsies are essential for obtaining the differential diagnosis
of BM failure syndromes, and the detection of a hypocellular BM is a prerequisite for
diagnosing AA.5 However, assessing BM cellularity in patients with BM failure is often
difficult, particularly when the cytopenias are not severe. Even when the BM of one bone
site is grossly replaced with fat tissue as a result of the immune-mediated destruction of
hematopoietic stem cells (HSCs), some hematopoietic nests remain in other bone sites
and may show hypercellularity due to increased BM activity that compensates for the
decreased hematopoiesis.6 BM aspiration or biopsies of these hot spots can sometimes
produce erroneous results. When pathological reports of BM examinations show hyperor normal cellularity, the physician may not generally consider the differential diagnosis
of AA. Hematologists should be careful with respect to cellularity when interpreting
the results of BM examinations, considering that BM cellularity cannot be accurately
determined based on BM biopsies of limited sites in the iliac bone. Magnetic resonance
imaging (MRI) of the thoracolumbar spine can be used to supplement an assessment of
cellularity with a BM biopsy. However, MRI often produces equivocal results in elderly
M A L I G N A N T H E M A T O L O G Y 25

disomy of the short arm of chromosome 6 (6pUPD).13 Detecting 6pUPD(+) leukocytes


requires a single nucleotide polymorphism array analysis, which takes several weeks. On
the other hand, HLA-LLs can be easily detected using monoclonal antibodies specific to
HLA-A alleles with FCM if the patient is heterozygous for the HLA-A allele.
Another useful examination that does not require advanced techniques is measurement
of the plasma thrombopoietin (TPO) level. Patients with MDS who have a TPO level of
320 pg/mL or greater (TPOhigh patients) exhibit a high progression-free survival rate and
good response to IST, similar to cases of AA.14 Virtually all patients having increased PNHtype cells or HLA-LLs fall into the TPOhigh group. Accordingly, if data for the TPO level are
available, physicians may not need to examine the peripheral blood in patients with BM
failure to determine the presence of other markers. The recent National Comprehensive
Cancer Network guidelines recommend using hypomethylating agents for the treatment of
MDS associated with thrombocytopenia; however, this option may be hazardous to TPOhigh
patients, as BM failure in these cases is not based on the presence of abnormal stem cells
with pre-leukemic features.15

The Hematologist Compendium 2010-2015

Figure 2 illustrates the mutual relationship of the markers that are potentially useful
for diagnosing immune pathophysiology in patients who do not show any karyotypic
abnormalities, increased blasts or ring sideroblasts in the BM. The precise role of
these markers in the management of gray zone BM failure must be evaluated in a large
prospective study because the predictive values of HLA-LLs and plasma TPO levels have
only been assessed in Japanese patients with BM failure. When deciding on treatment,
physicians also need to take patient age and disease duration into account, both of which
are known to affect the response to IST.11

Patient Follow-up
Based on the positive results for PNH-type granulocytes, we treated the patient with
cyclosporine at a dose of 4 mg/kg/day. The pancytopenia gradually improved in response
to this therapy, and the blood cell count seven months after the initiation of treatment was
as follows: WBC, 3.0109/L with 44 percent neutrophils; RBC, 2.791012/L; hemoglobin, 9.4
g/dL; MCV, 101.4 fL; platelets, 57109/L; reticulocytes, 75109/L. He continues to receive
cyclosporine without any signs of significant toxicities.
1. Valent P, Horny HP, Bennett JM, et al. Definitions and standards in the diagnosis and treatment of the
myelodysplastic syndromes: consensus statements and report from a working conference. Leuk Res.
2007;31:727-736.

Figure 2

2. Matsuda A, Jinnai I, Iwanaga M, et al. Correlation between dysplastic lineage and type of cytopenia in
myelodysplastic syndromes patients with refractory anemia according to the FAB classification. Am J
Clin Pathol. 2013;140:253-257.
3. Yamazaki H, Nakao S. Border between aplastic anemia and myelodysplastic syndrome. Int J Hematol.
2013;97:558-563.
4. Parmentier S, Schetelig J, Lorenz K, et al. Assessment of dysplastic hematopoiesis: lessons from healthy
bone marrow donors. Haematologica. 2012;97:723-730.

TPOhigh

5. Young NS, Calado RT, Scheinberg P. Current concepts in the pathophysiology and treatment of aplastic
anemia. Blood. 2006;108:2509-2519.

HLA-DRBI*15:01(+)

6. Nishimura R, Mase S, Araki R, et al. Massive hyper-reactive hematopoietic nests in bilateral iliac bones in
a patient with mild aplastic anemia. Pediatr Blood Cancer. 2014;61:1903-1904.
7. Wang H, Chuhjo T, Yasue S, et al. Clinical significance of a minor population of paroxysmal nocturnal
hemoglobinuria-type cells in bone marrow failure syndrome. Blood. 2002;100:3897-3902.

PNH-type cells(+)
HLA-A allelelacking cells (+)

TPOlow

8. Sugimori C, Chuhjo T, Feng X, et al. Minor population of CD55-CD59- blood cells predicts response to
immunosuppressive therapy and prognosis in patients with aplastic anemia. Blood. 2006;107:1308-1314.
9. Kulagin A, Lisukov I, Ivanova M, et al. Prognostic value of paroxysmal nocturnal haemoglobinuria clone
presence in aplastic anaemia patients treated with combined immunosuppression: results of two-centre
prospective study. Br J Haematol. 2014;164:546-554.
10. Tutelman PR, Aubert G, Milner RA, et al. Paroxysmal nocturnal haemoglobinuria phenotype cells
and leucocyte subset telomere length in childhood acquired aplastic anaemia. Br J Haematol.
2014;164:717-721.
11. Sloand EM, Wu CO, Greenberg P, et al. Factors affecting response and survival in patients with
myelodysplasia treated with immunosuppressive therapy. J Clin Oncol. 2008;26:2505-2511.
12. Sugimori C, Yamazaki H, Feng X, et al. Roles of DRB1 *1501 and DRB1 *1502 in the pathogenesis of
aplastic anemia. Exp Hematol. 2007;35:13-20.
13. Katagiri T, Sato-Otsubo A, Kashiwase K, et al. Frequent loss of HLA alleles associated with copy numberneutral 6pLOH in acquired aplastic anemia. Blood. 2011;118:6601-6609.
14. Seiki Y, Sasaki Y, Hosokawa K, et al. Increased plasma thrombopoietin levels in patients with
myelodysplastic syndrome: a reliable marker for a benign subset of bone marrow failure. Haematologica.
2013;98:901-907.

WATCH THE EXPERTS

Mutual relationship of the markers that are potentially useful for diagnosing immune
pathophysiology in myelodysplastic syndrome patients who do not show any karyotypic
abnormalities, increased blasts, or ring sideroblasts in the bone marrow. Each colored circle
represents a subset of patients with immune pathophysiology, such that the darker the area is, the
more likely immune pathophysiology is involved. TPOhigh, patients with a plasma thrombopoietin
(TPO) level 320 pg/mL; TPOlow, patients with a plasma TPO level <320 pg/mL; PNH-type cells
(+), patients possessing an increased percentage of paroxysmal nocturnal hemoglobinuria (PNH)
type cells; HLA-A allele-lacking cells (+), patients possessing HLA-A allele lacking leukocytes as
a result of uniparental disomy of the short arm of chromosome 6.

15. Greenberg PL, Attar E, Bennett JM, et al. Myelodysplastic syndromes: clinical practice guidelines in
oncology. J Natl Compr Canc Netw. 2013;11:838-874.

Dr. Nakao receives research funds and honoraria from Alexion.

Joseph Mikhael, MD, Med, FRCPC


Dr. Joseph Mikhael recaps the Social Media for
Hematologist session at the 2015 ASH Annual
Meeting. Attendees discussed the professional
benefits of social media and how it can be used
by professional hematology research and clinical
communities to promote conversations that could
have a positive impact on patient outcomes. Watch
the video on the ASH Youtube page. Learn about
ASHs social media presence at www.hematology.
org/Newsroom/465.aspx.
Read an article by Dr. Mikhael in this Compendium on page 20.

M A L I G N A N T H E M A T O L O G Y 26

The Hematologist Compendium 2010-2015

Hematopathologic Overview of Lymphocytosis November/December 2015


Due to the recent publication date of this article, an update has not been requested.

Figure 2
TRACY I. GEORGE, MD
Associate Professor of Pathology; Director of the Hematopathology Fellowship Program,
University of New Mexico School of Medicine, Albuquerque, NM

Small, round
nuclei

Folded or
cleaved nuclei

The Question

DIFFERENTIAL DIAGNOSIS

CLL
MBL

MCL
T-PLL

Flow cytometry
CLL FISH panel
FISH CCND1/IGH

FL
MCL
Atypical CLL

T-cell
lymphomas
Pertussis*

Flow cytometry
FISH CCND1/IGH,
BCL2
Tissue biopsy

What is your approach to lymphocytosis?

Convoluted
nuclei

Case
A 71-year-old man with a history of atypical chronic lymphocytic leukemia (CLL) last
treated in 2007 with a rituximab and chlorambucil-based regimen presents with an
increasing M protein of 4.1 g/dl (IgG). The laboratory findings were as follows: WBC,
7.4 109/L with 29 percent neutrophils, 66 percent lymphocytes, and 5 percent monocytes;
RBC, 3.94 1012/L; hemoglobin, 11.0 g/dL; mean corpuscular volume, 83 fL; platelets, 91
109/L. The patients bone marrow was hypercellular (90%) with a marked lymphoid
infiltrate present in nodular (paratrabecular and interstitial) and focal diffuse patterns
involving 75 percent of bone marrow cellularity. Lymphocytes were small and round with
condensed chromatin and occasional plasmacytoid lymphocytes were also observed. The
karyotype of the bone marrow was 46,XY,add(9)(p24),der(11)del(11)(p13)del(11)(q23) in
four cells with a sideline containing all of these abnormalities and +13 in two cells, and an
unrelated clone showing 45,X,Y in six cells, with 46,XY in seven cells. Fluorescence in situ
hybridization (FISH) found a deletion of the 13q14.3 region and was negative for deletions
of TP53, ATM, and LAMP1, and aneuploidy for chromosome 12.

Examining the Blood Smear


A slide review is appropriate in all patients with an unexplained lymphocytosis in order
to confirm the automated cell counts or to perform a manual differential for leukocyte
classification. In manually prepared blood smears, larger white blood cells tend to collect
at the edges of the smear and in the feathered edge. Good practice for slide review requires
assessments of all cell types (leukocytes, red blood cells [RBCs], and platelets) in both
quantity and quality. It is not uncommon for fragile leukocytes such as in CLL, infectious
mononucleosis, or acute leukemia to smudge on blood smears. In these situations, a few
drops of albumin can be added to peripheral blood before preparing the blood smear.
These albumin smears allow for proper identification of leukocytes and reduce the
number of smudge or basket cells. However, examination of RBCs and platelets should
still be performed on the original blood smear because the albumin can affect platelet and
erythrocyte morphology.

Reactive Lymphocytosis
Separating a monomorphic lymphocytosis from a pleomorphic lymphocytosis can help
distinguish a lymphoproliferative disorder from a reactive lymphocytosis, respectively.1
Most reactive lymphocytoses show a wide range of sizes and shapes in lymphocytes. The
classic example of a pleomorphic lymphocytosis is infectious mononucleosis, where the
lymphocytes range in size from small and round, to intermediate with abundant cytoplasm
(reactive lymphocytes),
to frank immunoblasts.
Figure 1
It is this spectrum of
morphology that points
to a greater likelihood
that a patient has a
reactive lymphocytosis;
younger age is also
a helpful clue. The
causes of a reactive
lymphocytosis
are extensive and
include infections
(viral, bacterial, and
parasitic), autoimmune
disease, vaccination,
drug hypersensitivity,
endocrine disorders,
stress (trauma,
Bordetella pertussis infection in a child showing three reactive
cardiac, extreme
lymphocytes that are small with deeply cleaved nuclei and scant
exercise), smoking, and
cytoplasm, in contrast with the monocyte in the lower right-hand
malignancy.
corner. Reprinted with permission from I Pereira, TI George,
While most of these
reactive lymphocytoses
are pleomorphic, a few

M A L I G N A N T H E M A T O L O G Y 27

Szary syndrome
Adult T-cell leukemia

Flow cytometry
T-cell clonality

HCL
SMZL
HCLV

Flow cytometry
BRAF

T-PLL
LPL

Plasmacytoid

LPL
Plasma cell myeloma
Plasma cell leukemia

Flow cytometry
SPEP/UPEP
MYD88 L265P
Myeloma FISH panel

Granules

T-LGL
NK cell leukemia

Flow cytometry
T-cell clonality
KIR profile

T-PLL
B-PLL
HCLV
MCL

Flow cytometry
Cytogenetics

Burkitt Leukemia
DLBCL
MCL
ALCL

Flow cytometry
FISH MYC
CCND1/IGH
ALK

Prominent
nucleoli

My Response

DA Arber. Atlas of Peripheral Blood. The Primary Diagnostic


Tool. Wolters Kluwer, Lippincott Williams and Wilkins, Inc.,
Philadelphia, PA (2012), page 130, Figure 13.5.

Villous
cytoplasm

ANCILLARY TESTS

Large cells

The differential diagnosis for a neoplastic lymphocytosis based on morphology in the peripheral
blood smear. The bolded diagnosis represents the image shown. Further recommended ancillary
testing is listed in the third column. CLL, chronic lymphocytic leukemia; MBL, monoclonal B-cell
lymphocytosis; MCL, mantle cell lymphoma; T-PLL, T-prolymphocytic leukemia; FL, follicular
lymphoma; HCL, hairy-cell leukemia; SMZL, splenic marginal-zone lymphoma; HCLV, hairycell leukemia variant; LPL, lymphoplasmacytic lymphoma, T-LGL, T large granular lymphocytic
leukemia; B-PLL, B-prolymphocytic leukemia; DLBCL, diffuse large B-cell lymphoma; ALCL,
anaplastic large-cell lymphoma. Adapted with permission from Chabot-Richards D, George TI.
Leukocytosis. Int J Lab Hematol 2014;36:279-88, page 282, Figure 2.
*Pertussis infection is a reactive cause of monomorphic lymphocytosis.

important exceptions are worth mentioning. The first is Bordetella pertussis, the causative
agent of whooping cough. The lymphocytes of B. pertussis are small and deeply clefted
with mature chromatin as shown in Figure 1. As this is commonly seen in the pediatric and
pregnant populations, clinical correlation will readily separate this from lymphomas, which
can show similar morphologic features (e.g., follicular lymphoma or Szary syndrome). The
second exception is polyclonal B-lymphocytosis, which typically shows lymphocytes with
distinct nuclear clefts but will demonstrate a spectrum of morphologic changes including
nuclear lobation and binucleate forms. This uncommon disorder is found in young to
middle-aged female smokers with a high association with human leukocyte antigen DR7,
and several genetic abnormalities have also been documented.2 The final exception is a
large granular lymphocytosis. Increased numbers of large granular lymphocytes (reactive
lymphocytes with scattered azurophilic granules) are commonly seen with viral infections,
malignancy, after bone marrow transplantation, and following chemotherapy. These
populations of large granular lymphocytes will wax and wane. However, persistence of a
large granular lymphocytosis with accompanying neutropenia and variable anemia should
raise suspicion for large granular lymphocytic leukemia.3 This is typically T cell in origin,
though a chronic lymphoproliferative disorder of natural killer cells is also well described.
Flow cytometry is recommended in these cases, followed by either T-cell clonality or KIR
analysis, if involving T cells or natural killer cells, respectively.

Neoplastic Lymphocytosis
Lymphoma cells tend to be monomorphic in appearance. While a blood smear may contain
a subset of lymphoma cells, these cells will resemble one another and stand out against
a background of normal bland lymphocytes. While CLL is the most common leukemia in
adults in the western world and is frequently seen in peripheral blood (or its monoclonal
B-cell lymphocytosis counterpart), peripheral blood involvement by bone marrow
lymphoma is found in up to 30 percent of subjects in some studies.4 Lymphoma cells will
show a wide variety of morphologic appearance, and this appearance raises a differential
diagnosis as shown in Figure 2.5 Further identification of the type of lymphoproliferative
disorder typically proceeds with flow cytometry. Although each laboratory has its own
cocktail of antibodies used for flow cytometry, consensus guidelines have been published.6
While the results from flow cytometry narrow down ones differential diagnosis to a short
The Hematologist Compendium 2010-2015

list, additional genetic or other ancillary studies are typically needed for confirmation, such
as FISH for CCND1/IGH to evaluate for mantle cell lymphoma. Additionally, bone marrow
biopsy or biopsy of another involved site is often necessary for a final diagnosis.
A common question is when should flow cytometry be performed. Some studies have
looked at this question in adults. In one study, the authors retrospectively reviewed flow
cytometry results of 71 patients 50 years of age and older with an absolute lymphocyte
count of 4 109/L or greater that had been called suspicious for a lymphoproliferative
disorder after smear review by a pathologist.7 Using receiver operating characteristic
(ROC) analysis, they found that an absolute lymphocyte count greater than 6.7 109/L for
patients 50 to 67 years of age, and 4 109/L or greater for patients older than 67 years had a
95 percent sensitivity and 76 percent specificity for predicting an abnormal flow cytometry
phenotype. A more recent retrospective single-center study examined 71 adults with
newly detected lymphocytosis greater than 5 109/L in a consecutive three-month period
and found that 6.8 109/L was the best cut-off value for predicting a lymphoproliferative
disorder with ROC analysis (sensitivity 90%, specificity 59%).8 In my own practice,
other triggers for flow cytometry include a persistent unexplained lymphocytosis or a
morphology that does not correlate with the diagnosis, as discussed below.

This case was presented at a multidisciplinary tumor board conference. While the
immunophenotype of the lymphocytes switched from CD5 to CD10 expression, the IGH data
supported that the same neoplastic clone was present in both the 2007 and current bone
marrow. Thus, a low-grade B-cell lymphoma with plasmacytic differentiation was found,
raising a differential diagnosis of lymphoplasmacytic lymphoma, versus a marginal zone
lymphoma with plasmacytic differentiation. While the lack of a MYD88 L265P mutation
argues against lymphoplasmacytic lymphoma, my colleagues have found this to be 96
percent sensitive for a diagnosis of lymphoplasmacytic lymphoma in a study of 317 cases
of low-grade B-cell lymphomas.10 A diagnosis of atypical CLL seemed less likely, as the
lymphocytes lacked LEF1 expression a marker identified based on gene expression
profiling data that has been described as nearly 100 percent sensitive and specific for CLL.11
The lymphocytes also lacked expression of CD200 by flow cytometry, another marker
overexpressed in CLL as well as lymphoplasmacytic lymphoma, though few cases of the
latter were tested.12 While the patient lacked splenomegaly, a diagnosis of marginal zone
lymphoma with plasmacytic differentiation was considered. The patient is scheduled to
receive ibrutinib.
1. George TI. Malignant or benign leukocytosis. Hematology Am Soc Hematol Educ Program.
2012;2012:475-484.
2. Mossafa H, Tapia S, Flandrin G, et al. Chromosomal instability and ATR amplification gene in patients
with persistent and polyclonal B-cell lymphocytosis (PPBL). Leuk Lymphoma. 2004;45:1401-1406.

Patient Follow-up
Review of the patients original diagnostic material confirmed that the atypical CLL
diagnosis was given based on immunophenotypic expression of FMC7, in addition to the
usual phenotype for CLL (CD20+, CD5+, CD23+). Morphology in the current blood and
bone marrow showed lymphocytes and plasmacytoid cells and not the usual small round
lymphocytes with coarsely clumped chromatin (soccer balls) of CLL; the plasmacytoid
cells were not readily identified on the earlier bone marrow smears. I performed flow
cytometry on the patients bone marrow and identified a light chainrestricted B-cell
population that expressed CD19, CD20, CD10, and CD23, and lacked expression of CD5
and CD200. Additionally, a light chainrestricted plasma cell population was identified.
Immunohistochemistry was performed on the bone marrow clot section. Cyclin D1 and
SOX-11 were negative in the B-cells, excluding the diagnosis of mantle cell lymphoma;
SOX11 is a newer marker for mantle cell lymphoma that has been found to be expressed
even in mantle cell lymphomas that lack overexpression of cyclin D1.9 LEF1 was negative,
providing no support for a diagnosis of CLL. Markers of follicle center cell origin, BCL6
and LM02, were also negative, providing no support for a lymphoma of follicle center cell
origin. The cytogenetic karyotype, while abnormal, was not specific for any particular B-cell
lymphoma; the lack of t(11;14) and t(14;18) argued against both mantle cell lymphoma and
follicular lymphoma. Molecular testing for MYD88 L265P mutation was performed and was
negative. DNA polymerase chain reaction analysis for immunoglobulin heavy chain gene
(IGH) was performed on the 2007 bone marrow and on the current 2015 bone marrow. A
clone was detected in both samples that was identical in amplicon size.

3. Dhodapkar MV, Li CY, Lust JA, et al. Clinical spectrum of clonal proliferations of T-large granular
lymphocytes: a T-cell clonopathy of undetermined significance? Blood. 1994;84:1620-1627.
4. Arber DA, George TI. Bone marrow biopsy involvement by non-Hodgkins lymphoma: frequency of
lymphoma types, patterns, blood involvement and discordance with other sites in 450 specimens. Am J
Surg Pathol. 2005;29:1549-1557.
5. Chabot-Richards DS, George TI. Leukocytosis. Int J Lab Hematol. 2014;36:279-288.
6. Bn MC, Nebe T, Bettelheim P, et al. Immunophenotyping of acute leukemia and lymphoproliferative
disorders: a consensus proposal of the European LeukemiaNet Work Package 10. Leukemia.
2011;25:567-574.
7. Andrews JM, Cruser DL, Myers JB, et al. Using peripheral blood smear review, age and absolute
lymphocyte count as predictors of abnormal peripheral blood lymphocytoses diagnosed by flow
cytometry. Leuk Lymphoma. 2008;49:1731-1737.
8. Sun P, Kowalski EM, Cheng CK, et al. Predictive significance of absolute lymphocyte count and
morphology in adults with a new onset peripheral blood lymphocytosis. J Clin Pathol. 2014;67:1062-1066.
9. Soldini D, Valera A, Sol C, et al. Assessment of SOX11 expression in routine lymphoma tissue sections:
characterization of new monoclonal antibodies for diagnosis of mantle cell lymphoma. Am J Surg Pathol.
2014;38:86-93.
10. Insuasti-Beltran G, Gale JM, Wilson CS, et al. Significance of MYD88 L265P mutation status in the
subclassification of low-grade B-cell lymphoma/leukemia. Arch Pathol Lab Med. 2015;139:1035-1041.
11. Tandon B, Peterson L, Gao J, et al. Nuclear overexpression of lymphoid-enhancer-binding factor 1
identifies chronic lymphocytic leukemia/small lymphocytic lymphoma in small B-cell lymphomas. Mod
Pathol. 2011;24:1433-1443.
12. Challagundla P, Medeiros LJ, Kanagal-Shamanna R, et al. Differential expression of CD200 in B-cell
neoplasms by flow cytometry can assist in diagnosis, subclassification, and bone marrow staging. Am J
Clin Pathol. 2014;142:837-844.

WATCH THE EXPERTS

Dr. George indicated no relevant conflicts of interest.

Eytan Stein, MD
Dr. Eytan Stein discusses molecularly targeted
therapies for acute myeloid leukemia (AML) in this
video from the ASH Impact Series available on
ASH On Demand. To watch this video and more,
visit www.ashondemand.org/FreeContent/Videos.

M A L I G N A N T H E M A T O L O G Y 28

The Hematologist Compendium 2010-2015

Ask the Hematologist


N O N M A L I G N A N T H E M AT O L O GY
This collection contains updated clinical information on malignant
and nonmalignant hematology as profiled in The Hematologist.

Specific Thrombophilia Work-Up Approach March/April 2010


UPDATE/COMMENTARY
Breast cancer is associated with an increased risk for venous thromboembolism
(VTE). Among more than 13,000 women with breast cancer in four U.K. databases,
the rate of VTE was 3.5 times that of age-matched controls.U1 Similarly, among more
than 100,000 patients with breast cancer in two California databases, the one- and
two-year incidences of VTE were 0.9 and 1.2 percent, with the incidence greatest in
the first six months after diagnosis.U2 VTE incidence also increased while patients
were receiving chemotherapy, in the month after therapy was discontinued,
and during the initial three months of treatment with tamoxifen.U1 Nonetheless,
compared with cancers of the pancreas, stomach, ovary, lung, and kidney, as well as
gliomas and lymphomas, this risk is relatively low.U3
There remains a paucity of data regarding thromboprophylaxis in ambulatory
patients with cancer in general and with breast cancer in particular. One relevant
study, TOPIC-1, was halted prematurely because of no difference in VTE incidence
between ambulatory breast cancer patients treated with low-molecular-weight
heparin or placebo.U4 It is noteworthy that neither the American Society of Clinical
Oncology,U5 the American College of Chest Physicians,U6 nor the European Society of
Medical Oncology3 currently recommend thromboprophylaxis in ambulatory patients
with any cancer, unless the individual patient is considered to be at high risk.

JOEL S. BENNETT, MD
Professor of Medicine, Hematology/Oncology Division, Department of Medicine, Perelman
School of Medicine, University of Pennsylvania, Philadelphia, PA

(Editors Note: The original question was submitted to Dr. Bennett through
Consult a Colleague. He expanded his answer for print.)

Clinical Problem
I have been treating a 54-year-old woman with a resected breast cancer with adjuvant
docetaxel, cyclophosphamide, and trastuzumab. She developed a central line thrombosis
during treatment and was treated with warfarin for three months, after which the central
line was removed. I plan to discontinue the warfarin in the next several weeks, assuming it
is safe to do so while she is receiving trastuzumab. Should the patient undergo a work-up
for thrombophilia despite a negative family history, and should warfarin be restarted if she
needs another central line to complete a one-year course of trastuzumab?

In 2010, I discussed the treatment of an ambulatory patient with breast cancer with
adjuvant chemotherapy who developed a central-line thrombosis. Based on the
information available in 2016, I would not change my recommendations.
U1. Walker AJ, West J, Card TR, et al. When are breast cancer patients at highest risk of venous
thromboembolism? A cohort study using English health care data. Blood. 2016;127:849-857.
U2. Chew HK, Wun T, Harvey DJ, et al. Incidence of venous thromboembolism and the impact on
survival in breast cancer patients. J Clin Oncol. 2007;25:70-76.
U3. Mandala M, Falanga A, Roila F, et al. Management of venous thromboembolism (VTE) in
cancer patients: ESMO Clinical Practice Guidelines. Ann Oncol. 2011;22 Suppl 6:vi85-92.
U4. Haas SK, Freund M, Heigener D, et al. Low-molecular-weight heparin versus placebo for the
prevention of venous thromboembolism in metastatic breast cancer or stage III/IV lung
cancer. Clin Appl Thromb Hemost. 2012;18:159-165.
U5. Lyman GH, Khorana AA, Kuderer NM, et al. Venous thromboembolism prophylaxis and
treatment in patients with cancer: American Society of Clinical Oncology clinical practice
guideline update. J Clin Oncol. 2013;31:2189-2204.
U6. Kahn SR, Lim W, Dunn AS, et al. Prevention of VTE in nonsurgical patients: antithrombotic
therapy and prevention of thrombosis, 9th ed: American College of Chest Physicians evidencebased clinical practice guidelines. Chest. 2012;141:e195S-226S.

As I mentioned earlier, I think the central venous catheter itself, as a foreign body in the
circulation, was the proximate cause of the thrombosis in your patient. So, it would seem
logical to ask whether prophylactic anticoagulation could prevent this from happening
again. There have been a number of studies addressing this question. However, a recently
reported meta-analysis of eight randomized controlled trials of thromboprophylaxis in
patients with cancer and central venous catheters concluded that thromboprophylaxis
had no significant effect on the risk of catheter-related thrombosis.6 Similarly, in the WARP
study of warfarin thromboprophylaxis in cancer patients with central venous catheters,7 it
was found that neither fixed-dose warfarin at 1 mg per day nor dose-adjusted warfarin to
maintain an INR of 1.5 to 2.0 reduced the incidence of catheter-related thromboses. Thus,
the available evidence does not support the use of prophylactic anticoagulation should
the central line be re-inserted.
Although your patient has several recognized predispositions to thrombosis, the currently
available evidence does not support the use of thromboprophylaxis for her clinical
situation. Further, on the basis of this conclusion, as well her age and history, I dont see
the utility of a thrombophilia work-up.

My Response

1. Blom JW, Doggen CJ, Osanto S, et al. Malignancies, prothrombotic mutations, and the risk of venous
thrombosis. JAMA. 2005;293:715-722.

This is a common but complex situation. This patient has several predispositions to
venous thrombosis, including breast cancer, chemotherapy, and an indwelling venous
catheter. The thrombosis she experienced could have resulted from the cumulative effect
of these factors, but I suspect the presence of the indwelling catheter was the major
precipitating cause.

3. Geerts WH, Bergqvist D, Pineo GF, et al. Prevention of venous thromboembolism: American College of
Chest Physicians Evidence-Based Clinical Practice Guidelines
(8th Edition). Chest. 2008;133:381S-453S.

As you know, cancers in particular malignancies of the brain, adenocarcinomas of the


lung and gastrointestinal tract, and hematologic malignancies are associated with a
sixfold to sevenfold increase in the risk for venous thromboembolism (VTE).1,2 Although
this risk is greatest in the months following diagnosis and declines with time, the risk
remains higher than in individuals without cancer even two years after diagnosis.
Moreover, the risk for patients with metastatic disease is increased further compared
to patients without. Although the risk for VTE in the first six months after a diagnosis of
breast cancer is twofold to fourfold less than for patients with the malignancies noted
above, the cumulative incidence of VTE for patients with breast cancer approaches that
of the others, because patients with breast cancer live substantially longer. This raises
the question whether prophylactic anticoagulation in a patient with breast cancer would
be beneficial. This issue was addressed in the most recent American College of Chest
Physicians Evidence-Based Clinical Practice Guidelines.3 After reviewing the available
data, the authors of the guidelines concluded that the evidence does not support routine
thromboprophylaxis for the primary prevention of VTE in ambulatory patients such as this
one.

2. Blom JW, Vanderschoot JP, Oostindir MJ, et al. Incidence of venous thrombosis in a large cohort of
66,329 cancer patients: results of a record linkage study. J Thromb Haemost. 2006;4:529-535.

4. Levine M, Hirsh J, Arnold A, et al. Double-blind randomised trial of a very-low-dose warfarin for
prevention of thromboembolism in stage IV breast cancer. Lancet. 1994;343:886-889.
5. Khorana AA. Cancer and thrombosis: implications of published guidelines for clinical practice. Ann
Oncol. 2009;20:1619-1630.
6. Chaukiyal P, Nautiyal A, Radhakrishnan S, et al. Thromboprophylaxis in cancer
patients with central venous catheters. A systematic review and meta-analysis. Thromb Haemost.
2008;99:38-43.
7. Young AM, Billingham LJ, Begum G, et al. Warfarin thromboprophylaxis in cancer
patients with central venous catheters (WARP): an open-label randomised trial. Lancet.
2009;373:567-574.

Dr. Bennett indicated no relevant conflicts of interest.

Chemotherapy administration and hormonal manipulation are also associated with


an increased risk of VTE. Therefore, it is surprising that there is a paucity of studies
addressing the efficacy of thromboprophylaxis in ambulatory cancer patients who are
receiving chemotherapy or hormonal therapy. One widely cited study from 1994 reported
that giving low-dose warfarin (INRs of 1.3-1.9) in patients with stage IV breast cancer
receiving chemotherapy resulted in a significant reduction in VTE.4 However, these results
were not replicated in subsequent studies,5 and thromboprophylaxis is not currently
recommended for patients with cancer receiving chemotherapy or hormonal therapy.3
With specific regard to trastuzumab, after reviewing the prescribing information and
searching the literature, I could not find evidence that VTE is a common adverse event
related to the use of this drug.

N O N M A L I G N A N T H E M A T O L O G Y 30

The Hematologist Compendium 2010-2015

Recommendations for Thrombophilia Testing for Women before Oral


Contraceptives are Prescribed November/December 2010
The authors have provided a rewrite of the original article to present all content updates since the publication date in 2010.

DONNA DIMICHELE, MD, 1 AND ANDRA H. JAMES, MD, MPH 2


1. Deputy Director, Division of Blood Diseases and Resources, National
Heart, Lung, and Blood Institute, National Institutes of Health, Bethesda, MD
2. Consulting Professor of Obstetrics & Gynecology, Duke University,
Durham, NC

The Question
A healthy 14-year-old girl presented with a hemoglobin of 2.9 g/lL, MCV 61 lL, and
reticulocyte count of 0.9 percent. Pregnancy test was negative. Menarche began at age
11; periods were somewhat irregular and lastedfive to sevendays with clots. A diagnosis
of severe iron deficiency due to menorrhagia was made and appropriately managed with
transfusion and iron. Should an evaluation be done for an underlying bleeding disorder?
What tests? What about such an evaluation if the anemia due to menorrhagia recurs three
years later?
The patient is now interested in contraception. A paternal aunt and maternal grandmother
have a vague history of thrombosis requiring treatment. What thrombophilia testing
should be undertaken in this patient before contraception is prescribed? Indeed, should
thrombophilia testing be recommended for all women before oral contraceptives (OCs)
are prescribed? What form of contraception and medication for regularization of periods
should be advised?

Our Response
This case represents a clinical situation encountered frequently by pediatric hematologists.
Unfortunately, clinical management decisions for adolescents must still be largely
extrapolated from published data and recommendations on and for women.1 However,
in a 2015 Committee Opinion endorsed by the American Academy of Pediatrics, the
American College of Obstetricians and Gynecologists revised and updated their guidance
on menstruation in girls and adolescents: menstrual cycle duration is typically 21 to 45
days; menstrual flow is seven days or less; and menstrual product use is three to six pads
or tampons per day. 2 Menorrhagia or heavy menstrual bleeding is defined as greater than
80 mL of blood per cycle and can be predicted on the basis of clots 1 inch diameter or
greater, low ferritin, and flooding, defined as a change of pad or tampon more frequently
than hourly.3 In the case of the 14-year-old girl, heavy menstrual bleeding could be inferred
on the basis of clots, cycles of up to seven days and severe iron-deficiency anemia in the
absence of any other source of bleeding.
The causes of heavy menstrual bleeding include acquired abnormalities of the uterine
cavity (polyps, adenomyosis; leiomyoma; malignancy and hyperplasia; other endometrial
abnormality) ovulatory dysfunction; iatrogenic causes (usually hormonal); causes not yet
classified; and coagulopathy.4 However, in an adolescent, the cause is less likely to be an
abnormality of the uterus and more likely to be ovulatory dysfunction, a hormonal cause,
or possibly, an underlying bleeding disorder. A bleeding disorder may be the sole cause of
heavy menstrual bleeding, or it may be a contributing cause. Consequently, an adolescent
presenting with heavy menstrual bleeding requires a thorough evaluation that includes
a thorough history and physical examination, an assessment of the uterus, an endocrine
evaluation if there is anovulation or hypo-ovulation, and possibly, an evaluation for an
underlying bleeding disorder.4 If anemia due to menorrhagia recurs three years later, a
re-evaluation would be indicated.
Not only is heavy menstrual bleeding more prevalent among women and girls with bleeding
disorders, but bleeding disorders are more prevalent among those with heavy menstrual
bleeding. In adolescents who present with heavy menstrual bleeding, the prevalence of
von Willebrand disease (vWD) has been reported to be 5 to 36 percent; the prevalence
of platelet dysfunction (depending on how it was defined) was 2 to 44 percent; the
prevalence of factor VIII deficiency was 8 percent; and the prevalence of thrombocytopenia
13 to 20 percent. Dr. Claire S. Philipp and colleagues5 reported on the importance of
various signs and symptoms as predictors of a bleeding disorder in women with heavy
menstrual bleeding. After administering a 12-page questionnaire of bleeding symptoms, the
investigators were able to develop a screening tool that was considered to be positive if one
of the following four criteria was met:
Duration of menses greater than or equal to seven days and flooding or impairment of
daily activities with most periods

The laboratory evaluation for an underlying bleeding disorder would include a


platelet count, mean platelet volume (MPV), and review of the blood smear to rule
out thrombocytopenia and platelet morphologic disorders; prothrombin (PT), partial
thromboplastin (aPTT), and thrombin times along with fibrinogen activity to screen for
clotting factor deficiency; and a von Willebrand factor profile and platelet function studies.
Additionally, specific factor activity levels would be performed if dictated by a prolonged
PT or aPTT or otherwise suggested by high ethnic prevalence or an X-linked family history
of a hemorrhagic diathesis. Any history or physical signs of connective tissue laxity should
also prompt some consideration of collagen evaluation. If the bleeding is severe and the
initial evaluation is negative, evaluation of the fibrinolytic system should be performed.
Hormonal therapy is a frequent and usually effective approach to treating heavy menstrual
bleeding in women and adolescents.1 That this adolescent is asking for contraception
makes this an ideal first-line therapy. The choice of hormonal therapy is made more
complicated by the family history of thrombosis and raises the question of thrombophilia
screening prior to prescribing contraceptives. The substantially increased risk of venous
thromboembolism (VTE) with either thrombophilic risk factor(s) or OCs is synergistically
amplified when underlying thrombophilia and OCs are combined.6 Nevertheless, with a
low-baseline VTE risk in young women of < 0.8 per 10,000 women years, even a 30-fold risk
increase in women with factor V Leiden on OCs would translate into a very low overall
event rate, limiting the cost effectiveness of routine screening prior to initiating OCs.6
Furthermore, the psychosocial consequences in adolescents of positive screening for
thrombophilia or denial of access to OCs remain unexplored, but are probably significant.
In the absence of an effective predictive screening strategy, alternative approaches include
counseling for risk-factor avoidance (especially smoking), intermittent thromboprophylaxis
during periods of high thrombosis risk (immobilization, lower extremity injury, surgery),
and minimization of risk through the prescription of less thrombogenic OCs. These
conservative measures would be warranted for this patient. OC preparations containing
low-dose estrogen and a second-generation progestin (levonorgestrel) would be preferred.
Although there are limited data on their use in adolescents and women with bleeding
disorders, combined hormonal contraception in the form of a ring or patch would likely be
equally effective in managing heavy menstrual bleeding, but both the ring and the patch
are associated with an increased risk of VTE compared to OCs.7 An alternative given this
patients family thrombosis history would be a levonorgestrel-containing intrauterine
device (IUD), which has been demonstrated to decrease menstrual blood loss in women
with bleeding disorders,8 but does not increase the risk of VTE.7 Although this patient is
quite young, there are reports of the levonorgestrel IUDs successful use in controlling
menstrual blood flow in adolescents.1,9 Hemostasis prophylaxis prior to IUD insertion
should be considered in adolescents with severe bleeding disorders. Finally, for those
adolescents with a specific bleeding diagnosis who fail gynecologic/hormonal management
of heavy menstrual bleeding, hemostatic therapy at the time of menses would be the next
step. Oral tranexamic acid is FDA-approved for this indicaton in adults, but has been used
in adolescents as well.10
1. James AH. Bleeding disorders in adolescents. Obstet Gynecol Clin North Am. 2009;36:153-162.
2. ACOG Committee Opinion No. 651: menstruation in girls and adolescents: using the menstrual cycle as a
vital sign. Obstet Gynecol. 2015;126:e143-e146.
3. Warner PE, Critchley HO, Lumsden MA, et al. Menorrhagia I: measured blood loss, clinical features,
and outcome in women with heavy periods: a survey with follow-up data. Am J Obstet Gynecol.
2004;190:1216-1223.
4. Munro MG, Critchley HO, Broder MS, et al. FIGO classification system (PALM-COEIN) for causes
of abnormal uterine bleeding in nongravid women of reproductive age. Int J Gynaecol Obstet.
2011;113:3-13.
5. Philipp CS, Faiz A, Dowling NF, et al. Development of a screening tool for identifying women with
menorrhagia for hemostatic evaluation. Am J Obstet Gynecol. 2008;198:163.e1-163.e8.
6. Wu O, Greer IA. Is screening for thrombophilia cost-effective? Curr Opin Hematol. 2007;14:500-503.
7. Lidegaard O, Nielsen LH, Skovlund CW, et al. Venous thrombosis in users of non-oral hormonal
contraception: follow-up study, Denmark 2001-10. BMJ. 2012;344:e2990.
8. Kingman CE, Kadir RA, Lee CA, et al. The use of levonorgestrel-releasing intrauterine system for
treatment of menorrhagia in women with inherited bleeding disorders. BJOG. 2004;111:1425-1428.
9. Silva CD, Geraldes F, Silva IS. Levonorgestrel intrauterine system as a treatment option for
severe menorrhagia in adolescent with type III von Willebrand disease. BMJ Case Rep. 2013;2013.
pii:bcr2013008833.
10. Srivaths LV, Dietrich JE, Yee DL, et al. Oral tranexamic acid versus combined oral contraceptives for
adolescent heavy menstrual bleeding: a pilot study. J Pediatr Adolesc Gynecol. 2015;28:254-257.

Dr. DiMichele and Dr. James indicated no relevant conflicts of interest.

History of treatment of anemia


Family history of a diagnosed bleeding disorder
History of excessive bleeding with tooth extraction, delivery or miscarriage, or surgery
The screening tool alone had a sensitivity of 82 percent for bleeding disorders. Although
the results would not be available at an initial visit, adding a pictorial blood assessment
chart score > 100 increased the sensitivity of the screening tool to 95 percent. The patient
in this case was positive for two of the criteria (7-day periods and anemia). Therefore, an
evaluation for an underlying bleeding disorder would be indicated.

N O N M A L I G N A N T H E M A T O L O G Y 31

The Hematologist Compendium 2010-2015

Treatment of Pregnant Female with Mesenteric Vein Thrombosis and


History of Spontaneous Fetal Losses January/February 2011
UPDATE/COMMENTARY
In 2011, the question was posed concerning treatment of heparin-induced thrombocytopenia
(HIT) during pregnancy. While there are no new treatment options for this patient, a couple
of points are worth making. In terms of treatment, options are actually more limited for the
acute setting, as the intravenous direct thrombin inhibitor lepirudin is no longer available.
More data on the use of fondaparinux in treatment of HIT have been reported, and this
remains a good option for treatment of outpatients and stable inpatients with HIT.U1 The
major caveats remain its renal clearance and long half-life without a reversal agent.
The patient present had a history of both pregnancy loss and mesenteric vein
thrombosis (MVT) during her prior pregnancy. The indication for anticoagulation in
this pregnancy was the history of MVT. Since 2011, more data have been published
showing that anticoagulation does not prevent pregnancy loss.U2,U3 The evidence is
strong in women without documented thrombophilia. In women with thrombophilia,
we await the results of an ongoing clinical trial; however, based on current data, if
there is an effect on pregnancy loss, it is small.
While we have more oral anticoagulant options than in 2011, they remain without
indication in pregnancy and, depending on the drug, animal data have demonstrated

pregnancy loss and fetal harm.U4 Dabigatran crosses the placenta, and given
that the other drugs are small molecules, they also would be expected to cross
the placenta. Excretion in breast milk is unknown. There are accumulating
data reported in women with some exposure during pregnancy,U4 but they are
not sufficient to assess safety, and thus these drugs (dabigatran, rivaroxaban,
apixaban, edoxaban) should not be initiated during pregnancy.
U1. Kang M, Alahmadi M, Sawh S, et al. Fondaparinux for the treatment of suspected heparininduced thrombocytopenia: a propensity score-matched study. Blood. 2015;125:924-929.
U2. Pasquier E, de Saint Martin L, Bohec C, et al. Enoxaparin for prevention of unexplained
recurrent miscarriage: a multicenter randomized double-blind placebo-controlled trial. Blood.
2015;125:2200-2205.
U3. Rodger MA. Recurrent pregnancy loss: drop the heparin needles Blood. 2015;125:2179-2180.
U4. Burnett AE, Mahan CE, Vazquez SR, et al. Guidance for the practical management of the direct
oral anticoagulants (DOACs) in VTE treatment. J Thromb Thrombolysis. 2016;41:206-232.

Dr. Konkles spouse has equity ownership in GlaxoSmithKline and Johnson and
Johnson.

bed thrombosis. It is unknown whether the JAK2 V617F mutation is associated with
thrombosis in the setting of pregnancy.
BARBARA A. KONKLE, MD
Director, Clinical and Translational Research, Bloodworks Northwest; Professor of Medicine/
Hematology, University of Washington, Seattle, WA

The Question
A 34-year-old woman had mesenteric vein thrombosis during her last pregnancy in 2007
and two prior second trimester spontaneous fetal losses. She now presents at 13 weeks
of pregnancy after being started on low-molecular-weight heparin (LMWH) by another
hematologist 14 days prior. Her baseline platelet count was 160,000, but today it is 80,000. A
PF4 antibody assay sent to rule out HIT was positive. Is it reasonable to stop the LMWH and
put her on warfarin, since she has completed the first trimester? Danaparoid is not available,
and fondaparanux is not on the American College of Chest Physicians list for HIT treatment.

My Response
HIT is uncommon during pregnancy. In a retrospective cohort study of 488 heparin-treated
women (244 pregnant), there were no cases of HIT in the 10 pregnant women who became
thrombocytopenic, but 10 cases in the 26 thrombocytopenic non-pregnant women.1 A study
of 31 pregnant women receiving LMWH found that none were positive for anti-heparin/
PF4 antibodies or developed antibodies when followed prospectively throughout their
pregnancies.2 However, case reports and clinical experience document the occurrence of HIT
with and without thrombosis during pregnancy. Although HIT may be less common in patients
treated with LMWH versus unfractionated heparin (UFH), it can occur and must be considered
in pregnant women receiving LMWH who have a > 50 percent decrease in the platelet count.
In choosing an anticoagulant for a pregnant woman with HIT, one has to consider the effects
on both the woman and the fetus. Concerns with anticoagulants that cross the placenta
include teratogenicity and bleeding. Data are limited and mostly reflect predictions based on
drug characteristics and/or animal studies. Danaparoid, the drug with which there is the most
experience in pregnancy, is unavailable in the United States. Fondaparinux, argatroban, and
lepirudin have been reported in treatment of pregnant women with HIT.3,4 All are FDA Class
B, indicating that animal studies have not shown harm in pregnancy, but there are no data
from human studies. Outside the setting of pregnancy, fondaparinux, unlike argatroban and
lepirudin, is not FDA-approved for HIT, but there are reports of its use in this setting.5 Patients
who receive this drug may develop anti-heparin/PF4 antibodies, but it is not clear that these
are associated with the development of HIT.
Fondaparinux is a pentasaccharide that, with antithrombin, inhibits factor Xa. An advantage
of this drug is its long half-life (~17 hrs.) allowing once-daily dosing. Disadvantages include
managing a drug with a long half-life around delivery. Also, the drug has been detected in
cord blood but at concentrations unlikely to cause bleeding,6 and no adverse outcomes
have been reported in infants. The short-acting direct thrombin inhibitors (DTI) argatroban,
lepirudin, and bivalirudin require intravenous administration. Successful short-term use of
argatroban and lepirudin has been reported in pregnant women with HIT.4 Based on their
pharmacokinetic properties, little or no drug would be expected to cross the placenta,
although one study showed minor transplacental passage of lepirudin.6
The patient presented above has a history of MVT in her prior pregnancy. Pregnancy is a
known risk factor for splanchnic bed thrombosis, and, in women with such a history, it is
appropriate to provide at least prophylactic anticoagulation during subsequent pregnancies.
The decision to test for additional conditions should be based on clinical and laboratory
indicators of underlying disease. We would check a platelet count remote from pregnancy,
because essential thrombocythemia is associated with both pregnancy loss and splanchnic

N O N M A L I G N A N T H E M A T O L O G Y 32

When a pregnant woman develops HIT, the heparin should be discontinued; she should
be evaluated for thrombosis by vascular ultrasound studies, and, unless contraindicated,
alternative anticoagulation should be initiated. This patient needs continued anticoagulation
because of the history of mesenteric vein thrombosis and her increased risk of thrombosis in
the setting of HIT. As in non-pregnant patients, warfarin should not be started in the setting
of acute HIT. The patient could be transitioned to warfarin later after the acute HIT resolves.
Teratogenic effects have not been demonstrated when warfarin is ingested after 12 weeks
gestation; however, it does cross the placenta and complications related to fetal hemorrhage
have been reported. Two new oral anticoagulants, dabigatran and rivaroxiban, will not be
useful alternatives, as both demonstrated reproductive toxicity and excretion in breast milk in
animals.7
Given the need for continued outpatient anticoagulation, it is reasonable to switch her to
fondaparinux, recognizing the need to discontinue the drug in anticipation of labor given its
long half-life. In a patient on prophylactic therapy without a recent thrombotic event, it may
be possible to stop the drug three days prior to scheduled induction or cesarean section.
Epidural or spinal anesthesia could be given if an anti-Xa measurement demonstrates drug
clearance. A woman at higher risk for thrombosis could be transitioned to an intravenous
short-acting DTI, but since there will be anticoagulant effects in the mother and possibly the
fetus, it should be discontinued in anticipation of a scheduled delivery to prevent bleeding
complications. The best approach to delivery should be carefully considered in collaboration
with maternal fetal medicine specialists.
After delivery, anticoagulation should be re-initiated with either fondaparinux or a DTI with
transition to warfarin. Fondaparinux is excreted in breast milk in animals, while argatroban,
bivalirudin, and lepirudin are not predicted to be excreted in breast milk. Data from one
lactating woman on therapeutic doses of lepirudin demonstrated no measureable drug in
her breast milk. Thus, if the patient plans to breast feed, lepirudin with transition to warfarin,
which also is not excreted in breast milk, may be the best post-partum option.
While HIT in pregnancy is rare, it does occur and management requires knowledge of disease
and drug effects on the mother and developing fetus, although data to inform these decisions
are limited. Potential risks and benefits of different approaches should be discussed with the
patient and her other health-care providers.
1. Fausett MB, Vogtlander M, Lee RM, et al. Heparin-induced thrombocytopenia is rare in pregnancy. Am
J Obstet Gynecol. 2001;185:148-152.
2. Gerdsen F, Luxembourg B, Langer F, et al. A prospective analysis of heparin-platelet factor 4 antibodies
in pregnant women treated with the low-molecular-weight heparin, dalteparin. Blood Coagul
Fibrinolysis. 2008;19:477-481.
3. Mazzolai L, Hohlfeld P, Spertini F, et al. Fondaparinux is a safe alternative in case of heparin intolerance
during pregnancy. Blood. 2006;108:1569-1570.
4. Young SK, Al-Mondhiry HA, Vaida SJ, et al. Successful use of argatroban during the third trimester of
pregnancy: case report and review of the literature. Pharmacotherapy. 2008;28:1531-1536.
5. Hook KM, Abrams CS. Treatment options in heparin-induced thrombocytopenia. Curr Opin Hematol.
2010;17:424-431.
6. Dempfle CE. Minor transplacental passage of fondaparinux in vivo. N Engl J Med. 2004;350:1914-1915.
7. U.S. Food and Drug and European Medicines Agencies websites: www.pradaxapro.com; www.medicines.
org.uk/emc/medicine/21265/SPC/Xarelto+10+mg+film-coated+tablets/#PREGNANCY

Dr. Konkle consults and receives research funding from Bayer Corporation,
and she has equity ownership in GlaxoSmithKline and Johnson & Johnson.

The Hematologist Compendium 2010-2015

Factor Replacement for Surgery in 48-Year-Old Male with Factor XI


Deficiency September/October 2011
No
update
has
been
provided
U
PD
ATE/C
OM
MEN
T A R Y for this article.

THOMAS L. ORTEL, MD, PhD


Professor of Medicine, Chief of the Division of Hematology in the Department of Medicine, and
Professor of Pathology, Duke University School of Medicine, Durham, NC

(Editors Note: The original question was submitted to Dr. Ortel through Consult
a Colleague. He expanded his answer for print.)

The Question
A 48-year-old man with factor XI deficiency (2% activity) was first diagnosed at the
age of 17 during evaluation for a prolonged aPTT. He underwent extraction of wisdom
teeth without plasma or topical support uneventfully. Periodontal surgery led to oozing
for which he was successfully treated with oral aminocaproic acid. He now needs
arthroscopic surgery on his left shoulder for repair of a torn rotator cuff. Does he need
factor replacement prior to surgery and afterward?

Relatively limited information is available describing therapeutic arthroscopy of


the shoulder in patients with clotting disorders. A single case series describing the
management and outcomes of five patients undergoing a total of six shoulder arthroscopic
procedures included one patient with a mild factor XI deficiency (baseline level of 60 U/dL)
who underwent arthroscopic subacromial decompression.8 This patient was treated with a
combination of intravenous and oral tranexamic acid without any bleeding complications.
Although shoulder arthroscopy is performed in a tissue not considered to have high
fibrinolytic activity, the impact of even a minimal increase in bleeding during a surgical
procedure in an enclosed space can be significant. The severely decreased factor XI
activity level in this patient, and the fact that he had previously required antifibrinolytic
therapy for periodontal surgery, are concerning for an increased bleeding risk.
Consequently, it was recommended that the patient receive fresh frozen plasma to
supplement the factor XI level prior to the procedure. Post-operatively, depending on the
factor XI activity levels and clinical outcome of the procedure, additional fresh frozen
plasma could be administered or antifibrinolytic therapy could be substituted instead.

My Response

1. Rosenthal RL, Dreskin OH, Rosenthal N. New hemophilia-like disease caused by deficiency of a third
plasma thromboplastin factor. Proc Soc Exp Biol Med. 1953;82:171-174.

Inherited deficiency of factor XI was first described in 1953 as a mild to moderate bleeding
disorder.1 Factor XI deficiency exhibits an autosomal inheritance pattern with a variable
clinical penetrance, as described below. It has been described in a wide variety of
population groups but is most common in Ashkenazi Jews. In this particular group, it is
estimated that one in eight individuals are heterozygous and one in 190 are homozygous
for mutations in the factor XI gene.2,3

3. Gomez K, Bolton-Maggs P. Factor XI deficiency. Haemophilia, 2008;14:1183-1189.

Spontaneous bleeding, except for menorrhagia, is uncommon in patients with factor XI


deficiency. Most bleeding episodes occur following surgical procedures or trauma. In
contrast to patients with hemophilia A or B, the correlation between clinical bleeding and
the baseline factor XI clotting activity is poor. For example, some patients with severe
factor XI deficiency (factor XI level < 10-20 IU/dL) exhibit no increase in hemorrhage,
whereas other individuals with levels that are only moderately below the lower limit of the
normal range develop bleeding complications after surgery.3

7. Levy O, Haddo O, Sforza G, et al. Put your extended finger on the bleeder: the use of direct pressure
from the shaver blade to achieve hemostasis. J Arthroscopic Related Surgery. 2011;27:867-869.

2. Seligsohn U. Factor XI deficiency in humans. J Thromb Haemost. 2009;7: Suppl 1:84-87.


4. Salomon O, Steinberg DM, Seligsohn U. Variable bleeding manifestations characterize different types of
surgery in patients with severe factor XI deficiency enabling parsimonious use of replacement therapy.
Haemophilia. 2006;12:490-493.
5. Bolton-Maggs P. Factor XI deficiency resolving the enigma? Hematology 2009. 2009;97-105.
6. Mannucci PM, Bauer KA, Santagostino E, et al. Activation of the coagulation cascade after infusion of a
factor XI concentrate in congenitally deficient patients. Blood. 1994;84:1314-1319.

8. Thomas JMC, Tuddenham EG, Ahrens PM. Therapeutic shoulder arthroscopy in patients with clotting
disorders. Haemophilia. 2008;14:859-861.

Dr. Ortel indicated no relevant conflicts of interest.

Rates of hemorrhagic complication have also been shown to vary depending on the type
of surgery being performed.4 In particular, procedures involving sites with increased local
fibrinolytic activity, such as the oral mucosa, nose, and genitourinary tract, are more
frequently associated with excessive bleeding compared to procedures involving tissues
not expressing fibrinolytic activity, such as bones or muscles. This study included six
patients who underwent trauma surgery and one total knee replacement, but none of the
patients underwent an arthroscopic procedure. The implication from these observations
is that conservative use of replacement therapy in patients with even severe factor XI
deficiency undergoing certain types of surgery is possible.4
Management of bleeding episodes and prevention of bleeding in relation to surgery is not
straightforward and needs to be tailored to the individual patient.5 In many patients with
factor XI deficiency, antifibrinolytic therapy alone may be sufficient to prevent bleeding
in most surgical settings. Factor XI levels can be raised with fresh frozen plasma, but
relatively large volumes may be required. The hemostatic level of factor XI activity to
target is debated, but a level of 30 to 45 IU/dL is probably sufficient in patients with severe
deficiency.5 Factor XI concentrates have been associated with an increased thrombotic
risk in certain patients and are not available in the United States.6
Good hemostasis helps to maintain arthroscopic visual clarity during surgery and is
essential for the successful completion of the procedure. A variety of strategies have been
used to control arthroscopic bleeding, including using electrocautery or a radiofrequency
thermal probe, adding epinephrine to the inflow irrigation solution or increasing the
irrigation fluid inflow pressure, and applying direct pressure via the shaver blade.7

N O N M A L I G N A N T H E M A T O L O G Y 33

The Hematologist Compendium 2010-2015

Immune Thrombocytopenia with Pulmonary Embolism and Deep-Vein


Thrombosis: Recommendations for Bone Marrow Aspirate and Biopsy
November/December 2011
UPDATE/COMMENTARY
In an article written for The Hematologist in 2011, I reviewed the results of several small
series and population-based analyses that assessed the risk for thromboembolism
in patients with immune thrombocytopenia (ITP). I concluded that these studies
provide evidence that the incidence of thromboembolism is increased in patients
with ITP, even in the presence of very low platelet counts. Since that time, there has
been no evidence presented to suggest that that conclusion is not accurate. Indeed,
several additional analyses involving large numbers of patients with ITP support
the conclusion that ITP is indeed associated with a small, but significant risk of
thromboembolism,U1 a risk that many hematologists are now aware of.
In most patients with ITP, the increased risk of thrombosis is small (relative risk < 2)
and not sufficient to influence management.U2 However, more recent studies suggest
that certain subpopulations of patients with ITP may be at a significantly higher risk of
thrombotic complications. These include patients who have undergone splenectomy;

KEITH MCCRAE, MD
Director, Benign Hematology, Department of Hematology and Solid Tumor Oncology; Professor,
Molecular Medicine, Department of Cellular and Molecular Medicine, Cleveland Clinic, Cleveland, OH

(Editors Note: This question was submitted through the Consult-a-Colleague


Program. Dr. McCrae was asked to respond.)

The Question
How often do you see ITP with pulmonary embolism (PE) and deep-vein thrombosis (DVT)?
The patient is a 60-year-old woman who had pneumonia about a month ago and now is
presenting with a platelet count of 30,000/l with DVT and PE. A hypercoagulable diagnostic
panel, including tests for heparin-induced thrombocytopenia (HIT) and antiphospholipid
antibodies, are pending. Would you perform a bone marrow aspirate and biopsy? A
pulmonologist inserted an inferior vena cava filter; was that appropriate?

My Response
ITP is a fascinating disorder with a complex pathogenesis that has only recently begun to be
understood in detail.1,2 While a number of immune abnormalities underlie the development
of ITP, the disease is largely mediated by antibodies reactive with glycoproteins expressed
on platelets and megakaryocytes. These antibodies cause accelerated platelet destruction
in the spleen as well as deficient platelet production secondary to impaired maturation of
megakaryocytes.

several studies, including a large study that identified patients with ITP undergoing
splenectomy through administrative codes,U3 suggest that thrombotic risk is increased
more than 5-fold in the 90 days after surgery, and 2.7-fold thereafter. Patients older than
60 years are also at increased risk of thromboembolism. Finally, though no definitive data
confirm an increased thrombotic risk associated with thrombopoietin receptor agonists,
some studies do suggest increased risk with these agents, and this should be considered
when treating patients with ITP who have other significant thrombotic risk factors.
U1. Ruggeri M, Tosetto A, Palandri F, et al. Thrombotic risk in patients with primary immune
thrombocytopenia is only mildly increased and explained by personal and treatment-related risk
factors. J Thromb Haemost. 2014;12:1266-1273.
U2. Rodeghiero F. Is ITP a thombophilic disorder? Am J Hematol. 2016;91:39-45.
U3. Boyle S, White RH, Brunson A, et al. Splenectomy and the incidence of venous thromboembolism
and sepsis in patients with immune thrombocytopenia. Blood. 2013;121:4782-4790.

antibodies (APLA) appears to be increased in patients with ITP, and several studies suggest
that ITP patients with APLA develop thrombosis more frequently. In a prospective study,
Diz-Kkkaya assessed 82 newly diagnosed patients with ITP, finding 31 (37.8%) positive
for APLA.8 After five years, the cumulative thrombosis-free survival of APLA-positive and
APLA-negative patients was 39 percent and 97.7 percent, respectively (p=0.004). Though
current guidelines do not recommend routine screening of ITP patients for APLA,9,10 this
should be considered in those who develop thrombi. Additional factors contributing to the
development of thrombosis in patients with ITP may include elevated levels of platelet-derived
microparticles11 and complement activation on antibody-coated platelets.12
The management of thrombosis in thrombocytopenic patients with ITP is challenging and not
addressed by current guidelines. Likewise, there are no evidence-based data on which to draw
from for recommendations. Many experts suggest that the risk of anticoagulation is acceptable
at platelet counts above 40,000 to 50,000/l, although the severity of the thrombotic event and
clinical characteristics of the patient should be considered. In the case presented here, the
clot burden is extensive and the platelet count close to where anticoagulation is reasonable.
Risk factors for bleeding in ITP include a prior history of bleeding and age 60 years, the
latter of which is applicable to this patient. I would recommend aggressive treatment of
this patients ITP, initially using corticosteroids, since agents such as IVIg may potentially
predispose to further thrombosis, and I would consider institution of anticoagulation with
intravenous heparin or another short-lived anticoagulant once the platelet count increases to
45,000/l. The use of an IVC filter is a matter of preference; it may be more strongly justified
if there is evidence of progressive thrombosis, or if the thrombocytopenia does not respond
quickly. However, given the association of ITP with thrombosis, I would avoid intravascular
devices if possible, and if an IVC filter is used, I would recommend a removable model.

The most common clinical presentation of ITP is bleeding, the severity of which correlates
with the degree of thrombocytopenia.3 However, many patients with ITP have few bleeding
complications even with very low platelet counts, an observation that may be related to the
presence of young, highly functional platelets, increased levels of plasma microparticles,
or other factors. Surprisingly, recent studies have also demonstrated that patients with ITP
have an increased risk of thrombosis. The data concerning ITP and thrombosis are derived
from several sources. A retrospective study from Aledort et al. suggested an increased risk
of thrombosis in patients with ITP, with 18 thromboembolic events occurring in 186 adults
(the majority after the diagnosis of chronic ITP).4 In another retrospective study reported in
an abstract by Bennett et al., a 6.9 percent cumulative incidence of thromboembolic events in
patients with ITP was seen over a 15-month period. Sarpatwari et al. prospectively analyzed
the incidence of thromboembolic events in patients with ITP using the UK General Practice
Research Database, identifying 1,070 adults >18 years of age with primary ITP that were
matched with 4,280 ITP-free patients.5 Over a median of 47.6 months, the adjusted hazard
ratio for venous, arterial, or combined thromboembolic events in patients with ITP was 1.58
(95% CI, 1.01-2.48), 1.37 (95% CI, 0.94-2.00), and 1.41 (95% CI, 1.04-1.91), respectively. Subgroup
analysis suggested a strikingly increased risk of myocardial infarction in the ITP cohort. Also
suggested was a direct relationship between the severity of thrombocytopenia and the risk
of thrombosis. Another recent study utilizing a matched ITP cohort obtained from the Danish
National Patient Registry observed an incidence rate ratio for venous thromboembolism in
patients with ITP of 2.04 (95% CI, 1.45-2.87).6 Two of the ITP patients with VTE had severe
thrombocytopenia, although one was ultimately diagnosed with lung cancer.

The decision to perform a bone marrow aspirate and biopsy should not be influenced by a
thrombotic event. If the peripheral blood smear shows isolated thrombocytopenia and the
history and physical examination do not suggest other disorders, bone marrow aspirate would
not be required.

Taken together, these studies provide evidence that the incidence of thromboembolism is
increased in patients with ITP, even in the presence of very low platelet counts. While acute
portal vein thrombosis has been reported in up to 15 percent of ITP patients undergoing
splenectomy, and an increased risk of subsequent thrombosis has been associated with
splenectomy for thalassemia,7 the review from Sarpatwari et al. demonstrates a similar
incidence of thrombosis in splenectomized and non-splenectomized ITP patients.

9. Neunert C, Lim W, Crowther M et al. The American Society of Hematology 2011 evidence-based practice
guideline for immune thrombocytopenia. Blood. 2011;117:4190-4207.

The mechanisms underlying the paradoxical development of thrombosis in patients with ITP
have not been defined, though several have been postulated. The incidence of antiphospholipid
N O N M A L I G N A N T H E M A T O L O G Y 34

1. Semple JW, Provan D, Garvey MB, et al. Recent progress in understanding the pathogenesis of immune
thrombocytopenia. Curr Opin Hematol. 2010;17:590-595.
2. Cines DB, Liebman HA. The immune thrombocytopenia syndrome: a disorder of diverse pathogenesis
and clinical presentation. Hematol Oncol Clin North Am. 2009;23:1155-1161.
3. Fogarty PF. Chronic immune thrombocytopenia in adults: epidemiology and clinical presentation.
Hematol Oncol Clin North Am. 2009;23:1213-1221.
4. Aledort LM, Hayward CP, Chen MG, et al. Prospective screening of 205 patients with ITP, including
diagnosis, serological markers, and the relationship between platelet counts, endogenous
thrombopoietin, and circulating antithrombopoietin antibodies. Am J Hematol. 2004;76:205-213.
5. Sarpatwari A, Bennett D, Logie JW et al. Thromboembolic events among adult patients with primary
immune thrombocytopenia in the United Kingdom General Practice Research Database. Haematologica.
2010;95:1167-1175.
6. Severinsen MT, Engebjerg MC, Farkas DK et al. Risk of venous thromboembolism in patients with
primary chronic immune thrombocytopenia: a Danish population-based cohort study. Br J Haematol.
2011;152:360-362.
7. Crary SE, Buchanan GR. Vascular complications after splenectomy for hematologic disorders. Blood.
2009;114:2861-2868.
8. Diz-Kkkaya R, Hacehanefioglu A, Yenerel M et al. Antiphospholipid antibodies and antiphospholipid
syndrome in patients presenting with immune thrombocytopenic purpura: a prospective cohort study.
Blood. 2001;98:1760-1764.

10. Provan D, Stasi R, Newland AC et al. International consensus report on the investigation and
management of primary immune thrombocytopenia. Blood. 2010;115:168-186.
11. Jy W, Horstman LL, Arce M, Ahn YS. Clinical significance of platelet microparticles in autoimmune
thrombocytopenias. J Lab Clin Med. 1992;119:334-345.
12. Peerschke EI, Yin W, Ghebrehiwet B. Complement activation on platelets: implications for vascular
inflammation and thrombosis. Mol Immunol. 2010;47:2170-2175.

Dr. McCrae indicated no relevant conflicts of interest.


The Hematologist Compendium 2010-2015

Heparin-Induced Thrombocytopenia being Treated with Argatroban with


Persistent Thrombocytopenia January/February 2012
UPDATE/COMMENTARY
There have been a number of reports in the literature where a direct oral anticoagulant
has been used in the treatment of heparin-induced thrombocytopenia (HIT).
While dabigatran and apixaban have been employed, the most experience is with
rivaroxaban.U1-3 Dr. Lori-Ann Linkins and colleaguesU3 conducted a multicenter, single
arm, cohort study that included 22 consecutive patients with suspected or confirmed
HIT. Participants with a positive local assay were administered rivaroxaban 15 mg bid
until platelet recovery or 21 days if there was acute thrombosis at study entry; this
was followed by 20 mg daily. Among the 12 patients who proved to be HIT-positive, six
had HIT with thrombosis (HITT) at study entry. Platelet recovery was seen in nine of
10 with thrombocytopenia; new thrombosis occurred in one, and one required limb
amputation despite platelet recovery. This and other case reports suggest that direct
oral anticoagulants such as rivaroxaban are effective in treating HIT.

KENNETH A. BAUER, MD
Professor of Medicine, Harvard Medical School, Beth Israel Deaconess Medical Center, Boston, MA

(Editors Note: This question was submitted through the Consult-a-Colleague


program. Dr. Bauer was asked to respond.)

The Question
How do you manage a patient with HIT being treated with argatroban who has persistent
thrombocytopenia? The patient is a 45-year-old man with laboratory evidence of HIT
(positive serotonin release assay) without thrombosis who has been treated with
argatroban for 10 days. The platelet count was 52,000 per microliter at the time of diagnosis
and increased to 95,000 per microliter during the first five days of therapy. However, over
the past five days, no significant increase in the platelet count has been observed.

Is there a platelet threshold at which you recommend starting warfarin? If so, what
evidence supports that determination? Would your recommendation change if the
patient had HITT?
If the recommended platelet threshold is not reached, is there a point at which you feel
it is safe to start treatment with warfarin? For example, in this patient who has been on
argatroban for 10 days, is it acceptable to start warfarin even though his platelet count is
less than 100,000 per microliter? Would your recommendation change if the patient had
HITT?
Does fondaparinux play a role in management of HIT in this setting?
Is there a role for the new oral anticoagulants (dabigatran and rivaroxaban) in this
setting?

My Response
HIT is a clinical-pathological syndrome that is typically characterized by the onset of
thrombocytopenia (50% fall from baseline) within five to 10 days of drug initiation. The
mean platelet count in patients with this disorder is 60,000 per microliter. Thrombosis
occurs in approximately 50 percent of patients and clusters in the first few days following
onset of thrombocytopenia (termed HITT). It is mediated by antibodies against platelet
factor 4 (PF4)-heparin complexes that activate platelets leading to excessive thrombin
generation. Prompt recognition and appropriate treatment of HIT is required to reduce the
risk of serious thrombotic events and complications including limb loss. Paradoxically,
a major problem with HIT today is its over-diagnosis. Enzyme immunoassays (EIAs)
for antibodies against PF4-heparin complexes are currently widely applied for making
the diagnosis of HIT; while highly sensitive (>99%), only some antibodies have strong
platelet-activating properties in the serotonin-release assay, one of the gold standard
platelet-activation assays used for diagnostic confirmation. As results of serotonin
release assays are rarely available for clinical decision making early in the course of HIT,
it is best not to order ELISA tests in patients with low pretest probability scores for HIT
(4T score: Thrombocytopenia, Timing of platelet count fall, Thrombosis, oTher causes
of thrombocytopenia).1 Also, many labs do not report the optical densities (OD) of
positive EIAs; in such instances, it is useful to request this information, as it can be useful
diagnostically since the strength of a positive EIA (OD) predicts for a positive SRA.2
After stopping heparin administration in patients with HIT, the median time to achieving
a platelet count of >150,000 per microliter is about four days. However, in patients with
more severe HIT, the platelet count can take longer (up to two weeks or more) to recover.
It is important to keep in mind, however, that patients with HIT often have concurrent
medical problems or are on other medications contributing to thrombocytopenia. The
recommendation in the 8th Edition of the American College of Chest Physicians EvidenceBased Clinical Practice Guidelines is as follows3: For patients with strongly suspected or
confirmed HIT, we recommend against the use of vitamin K-antagonist (VKA) therapy until
N O N M A L I G N A N T H E M A T O L O G Y 35

U1. Ng HJ, Than H, Teo EC. First experiences with the use of rivaroxaban in the treatment of
heparin-induced thrombocytopenia. Thromb Res. 2015;135:205-207.
U2. Dhakal P, Pathak R, Giri S, et al. New Oral Anticoagulants for the Management of Heparin
Induced Thrombocytopenia: A Focused Literature Review. Cardiovasc Hematol Agents Med
Chem. 2015;13:87-91.
U3. Linkins LA, Warkentin TE, Pai M, et al. Rivaroxaban for Treatment of Suspected or Confirmed
Heparin-Induced Thrombocytopenia Study. J Thromb Haemost. 2016;14:1206-1210.

Dr. Bauer has served as a consultant to Janssen, BMS, Daiichi Sankyo, and
Instrumentation Laboratory.

after the platelet count has substantially recovered (i.e., usually to at least 150 109/L) over
starting VKA therapy at a lower platelet count (Grade 1B); that VKA therapy be started
only with low, maintenance doses (maximum, 5 mg of warfarin or 6 mg of phenprocoumon)
rather than with higher initial doses (Grade 1B); and that the non-heparin anticoagulant
(e.g., lepirudin, argatroban, danaparoid) be continued until the platelet count has reached a
stable plateau, the INR has reached the intended target, and after a minimum overlap of at
least five days between non-heparin anticoagulation and VKA therapy rather than a shorter
overlap (Grade 1B).
Despite a platelet count that has only risen to 95,000 per microliter, I would transition this
patient with HIT without thrombosis (termed isolated HIT) off a parenteral direct thrombin
inhibitor (i.e., argatroban) after 10 days of therapy. Management options for this patient
include: 1) initiation of warfarin carefully under the cover of continued IV argatroban; 2)
initiation of fondaparinux alone at therapeutic doses (provided the patients creatinine
clearance is greater than 30 mL/min) in lieu of argatroban; or 3) initiation of an oral direct
thrombin or factor Xa inhibitor (either dabigatran etexilate or rivaroxaban) at therapeutic
doses.
The initiation of warfarin causes a rapid decline in protein C levels and a slower decline
in the levels of longer-lived vitamin K-procoagulant factors (especially prothrombin)
augmenting the preexistent hypercoagulable state due to HIT. This mechanism is
contributory to the development of venous limb gangrene, which is estimated to occur in
12 percent of patients with HITT4; its frequency is likely less in patients with isolated HIT.
Fondaparinux, a synthetic pentasaccharide that selectively inhibits factor Xa after binding
to antithrombin, has an attractive profile for the treatment of HIT based on in vitro studies;
it is administered subcutaneously once daily. Small case series indicate generally favorable
outcomes with initial use of this agent in HIT; in a recent retrospective series of 16 patients
with the diagnosis confirmed by serotonin-release assay, nine of whom had thrombosis,
none developed new or recurrent thrombosis.5 However, fondaparinux is not approved
for this indication; it is approved in the United States for the prophylaxis of VTE following
major orthopedic and general surgery and the initial treatment of VTE.
If the continuation of argatroban is the only reason for continued hospitalization, I
would employ fondaparinux alone at this juncture provided the patient could self-inject
the medication and drug acquisition was not a problem (i.e., inability to gain insurance
approval or prohibitive out-of-pocket cost). The total duration of anticoagulant treatment
has not been defined by prospective studies, but I would treat isolated HIT for a minimum
of six weeks given the high risk of thrombosis within the first 30 days after diagnosis. The
use of fondaparinux to complete treatment would obviate the use of warfarin entirely and
the burden of INR monitoring with attendant dose adjustments. Alternatively, warfarin
could be initiated after the platelet count had risen to more than 150,000 per microliter
under the cover of fondaparinux overlap for a minimum of five days until the INR reached
the target range of 2-3. This would also circumvent complexities relating to INR monitoring
during argatroban therapy if this option were chosen.3 For patients with HITT, I would treat
with anticoagulation for three to six months.
It should be pointed out that the efficacy of non-heparin anticoagulants for HIT and their
approval by the U.S. FDA was not based on prospective, randomized controlled clinical
trials. Vitamin K antagonists have been used for the treatment of HIT for many years
and adopted by evidence-based guidelines after initial treatment with a non-heparin
anticoagulant, as they have up until recently been the only class of oral anticoagulants
available. Based on our mechanistic understanding of venous-induced limb gangrene in HIT,
a strong case can be made that we should be moving away from using warfarin in the initial
phase of HITT (and HIT) treatment (up until 30 days after diagnosis) given that alternative
oral anticoagulants that do not lower protein C levels are available.6 These oral agents
selectively target thrombin or factor Xa, have a rapid onset of action, and do not require
coagulation monitoring; however, there is not yet any reported experience with these agents
in this patient population, and they have no specific antidote. Dabigatran and rivaroxaban
have gained FDA approval for stroke prevention in atrial fibrillation7,8 and rivaroxaban is
The Hematologist Compendium 2010-2015

approved for the prophylaxis of VTE following total hip or knee replacement9; they have
shown promising result for the treatment of symptomatic VTE but have not yet been
approved for this indication in the United States. Thus, caution should be exercised if either
dabigatran or rivaroxaban is used in a patient such as this; if chosen, they should be used at
therapeutic doses and limited to adult patients with satisfactory renal function (creatinine
clearance > 30 mL/min). Furthermore, given that HIT can result in serious complications
including limb loss and subsequent litigation against health-care providers, hematologists
choosing to use any of the new anticoagulants (dabigatran, rivaroxaban, or fondaparinux)
should carefully document in the medical record their rationale for choosing the new agent.
Both the infrequent occurrence of HIT confirmed by validated platelet-activation assays
and the clinical heterogeneity of affected patients make it difficult to perform trials of new
agents in HIT. It is therefore unlikely that the new oral anticoagulants will be studied in
controlled trials so as to gain FDA approval for management of this disorder in the near
future. Hopefully, the reporting of well-characterized cohorts of patients with HIT (or HITT)
treated with these agents will lead to favorable outcomes that will improve, as well as
simplify, management of this disorder.

1. Lo GK, Juhl D, Warkentin TE , et al. Evaluation of pretest clinical score (4 Ts) for the diagnosis of
heparin-induced thrombocytopenia in two clinical settings. J Thromb Haemost. 2006;4:759-765.
2. Warkentin TE, Sheppard JI, Moore JC, et al. Quantitative interpretation of optical density measurements
using PF4-dependent enzyme-immunoassays. J Thromb Haemost. 2008;6:1304-1312.
3. Warkentin TE, Greinacher A, Koster A, et al. Treatment and prevention of heparin-induced
thrombocytopenia: American College of Chest Physicians Evidence-Based Clinical Practice Guidelines
(8th Edition). Chest. 2008;133:340S-380S.
4. Warkentin TE, Elavathil LJ, Hayward CP, et al. The pathogenesis of venous limb gangrene associated
with heparin-induced thrombocytopenia. Ann Int Med. 1997;127:804-812.
5. Warkentin TE, Pai M, Sheppard JI, et al. Fondaparinux treatment of acute heparin-induced
thrombocytopenia confirmed by the serotonin-release assay: a 30-month, 16-patient case series. J
Thromb Haemost. 2011;9:2389-2396.
6. Krauel K, Hackbarth C, Frll B, et al. Heparin-induced thrombocytopenia: in vitro studies on the
interaction of dabigatran, rivaroxaban, and low-sulfated heparin, with platelet factor 4 and anti-PF4/
heparin antibodies. Blood. 2012;119:1248-1255.
7. Connolly SJ, Ezekowitz MD, Yusuf S, et al. Dabigatran versus warfarin in patients with atrial fibrillation.
N Eng J Med. 2009;361:1139-1151.
8. Patel MR, Mahaffey KW, Garg J, et al. Rivaroxaban versus warfarin in nonvalvular atrial fibrillation. N
Engl J Med. 2011;365:883-891.
9. Turpie AG, Lassen MR, Eriksson BI, et al. Rivaroxaban for the prevention of venous thromboembolism
after hip or knee arthroplasty. Pooled analysis of four studies. Thromb Haemost. 2011;105:444-453.

Dr. Bauer has served as a consultant to GSK, Bayer Healthcare, and Johnson & Johnson.

The Hematologist Video Series


A new video series for The Hematologist titled Conversations with Innovators
is available on YouTube; it features three videos by Dr. Omar Abdel-Wahab from
Memorial Sloan Kettering Cancer Center, Dr. Neal Young from the National
Heart, Lung, and Blood Institute at the National Institutes of Health (NIH), and
Dr. Ann Mullally from the Brigham and Womens Hospital and the Dana-Farber
Cancer Institute. Dr. Abdel-Wahab discusses his labs efforts to understand
the biology of mutations in components of the spliceosome with the goal of
using them as therapeutic targets in the treatment of leukemias. Dr. Young
covers his labs investigation of an improved treatment protocol for aplastic
anemia involving immunosuppressive therapy in combination with eltrombopag.
In the third video, Dr. Mullally talks about her labs study of mutant calreticulin
in patients with myeloproliferative neoplasms. To watch all three videos and for
more to come soon, visit www.youtube.com/ASHwebmaster under the playlist
titled The Hematologist: Conversations with Innovators.

N O N M A L I G N A N T H E M A T O L O G Y 36

The Hematologist Compendium 2010-2015

Approach to the Diagnosis and Management of the Anemia of Chronic


Inflammation March/April 2013
UPDATE/COMMENTARY
Since the publication of this Ask the Hematologist article, the role of inflammation in
reduced renal erythropoietin (EPO) production has been better defined. With inflammation, renal EPO-producing fibroblasts have increased transforming growth factor-1
(TGF1) and NFB expressions that mediate their transformation to myofibroblasts,
which no longer produce EPO.U1 The potential restoration of EPO production in these
cells by inhibition of specific hypoxic-inducible factor (HIF) prolylhydroxylases suggests
a potential treatment for some patients with anemia of chronic inflammation (ACI).U2
In a study of patients with unexplained anemia of the elderly (UAE) and age-matched
normal controls, increases in serum ferritin and decreases in serum iron as well as
increases in neopterin, a product of activated macrophages, correlated with the degree
of anemia.U3 Compared to controls, the UAE group had decreased renal function and
similar EPO levels despite decreased hemoglobin values, suggesting that they had the
restricted iron flux and decreased EPO found in ACI.

diagnostic utility in those patients in whom the differentiation of these two disease
states is not clear from the standard clinical evaluation. Lastly, targeting hepcidin
has further emerged as a potential treatment in ACI, as direct pharmacological
inhibition of hepcidin via an antihepcidin oligoribonucleotide has shown promising
results in IL-6induced anemia in a primate modelU5and similar activity for a limited
period in lipopolysaccharide-treated human volunteers.U6
U1. Souma T, Yamazaki S, Moriguchi T, et al. Plasticity of renal erythropoietin-producing cells
governs fibrosis. J Am Soc Nephrol. 2013;24:1599-1616.
U2. Souma T, Nezu M, Nakano D, et al. Erythropoietin synthesis in renal myofibroblasts is restored
by activation of hypoxia signaling. J Am Soc Nephrol. 2016;27:428-438.
U3. Artz AS, Xue QL, Wickrema A, et al. Unexplained anaemia in the elderly is characterised by
features of low grade inflammation. Br J Haematol. 2014;167:286-289.
U4. van Santen S, de Mast Q, Oosting JD, et al. Hematologic parameters predicting a response to
oral iron therapy in chronic inflammation. Haematologica. 2014;99:e171-e173.

In terms of differentiating ACI from combined ACI and iron deficiency, a trial of
oral iron in patients with rheumatological diseases demonstrated that increased
reticulocytes and increased reticulocyte hemoglobin content after one week
predicted subsequent responses in hemoglobin.U4 This one-week response may have

U5. Schwoebel F, van Eijk LT, Zboralski D, et al. The effects of the anti-hepcidin Spiegelmer NOX-H94
on inflammation-induced anemia in cynomolgus monkeys. Blood. 2013;121:2311-2315.
U6. van Eijk LT, John AS, Schwoebel F, et al. Effect of the antihepcidin Spiegelmer lexaptepid on
inflammation-induced decrease in serum iron in humans. Blood. 2014;124:2643-2646.

cytokines suppress EPO production, decreasing plasma EPO concentration and increasing
apoptosis of erythroid cells in the EPO-dependent stages of differentiation (Figure).4 Finally,
the inflammatory cytokines IL-6 and members of the bone morphogenetic protein (BMP)
family diminish serum iron concentration by inducing transcription of hepcidin, the master
regulator of iron homeostasis.3 Hepcidin exerts its effects primarily through interaction with
the cellular iron exporter, ferroportin, expressed on the basolateral surface of enterocytes
and on reticuloendothelial cells. Binding to hepcidin induces endocytosis and degradation
of ferroportin thereby restricting gastrointestinal iron absorption and impairing macrophage
export and recycling of iron from phagocytosed senescent erythrocytes. The trapping of iron
in reticuloendothelial cells accounts for the characteristic iron-laden macrophages observed
in ACI bone marrow aspirates stained with Prussian blue. Because iron absorption and iron
recycling are impaired, plasma iron concentration is low, and transferrin saturation is often
subnormal (as in our case), resulting in a functional iron deficiency state with consequent
suboptimal delivery of iron to maturing erythroblasts. Interpretation of transferrin saturation
must also take into account the fact that transferrin is a negative acute phase reactant, and as
such, the serum concentration is often subnormal or at the lower end of the normal range in
patients with ACI (as illustrated in our case).

P. BRENT FERRELL, MD, 1 AND MARK J. KOURY, MD 2


1. Instructor of Medicine, Division of Hematology/Oncology, Department of
Medicine, Vanderbilt University Medical Center, Nashville, TN
2. Professor of Medicine, Emeritus, Vanderbilt University School of Medicine,
Nashville, TN

Patient History
A 64-year-old man had a history of two years of weight loss and mild normocytic anemia,
but over the preceding two months, worsening polyarthritis, fatigue, and anemia (Hgb
8.6 g/dL, Hct 25%, MCV 91 fL) were noted. The reticulocyte count was 1.4 percent with a
normal WBC and normal platelet count. He had normal serum cobalamin, methylmalonate,
LDH, RBC folate, and negative hepatitis serologies. Laboratory tests were consistent with
normal renal, thyroid, and liver function. Abnormal laboratory tests included an erythrocyte
sedimentation rate of 87 mm/h, serum C-reactive protein (CRP) of 145 mg/L, haptoglobin of
447 mg/dL, and serum albumin of 2.5 g/dL. Iron studies showed a serum iron of 23 ug/dL,
TIBC of 209 ug/dL, transferrin saturation of 11 percent, and serum ferritin of 299 ng/mL. He
was transfused with two units of packed erythrocytes, begun on low-dose prednisone, and
referred for evaluation of anemia.

Diagnostic considerations
In most instances, ACI is normocytic, normochromic, but approximately one-third of cases
fall into the microcytic, hypochromic morphological classification. In the latter cases,
functional iron deficiency as a consequence of excess hepcidin production dominates the
pathophysiology, and review of the peripheral blood film reveals features similar to other
processes, such as iron deficiency or thalassemia, that affect production of heme or globin.
ACI is typically a mild-to-moderate hypoproliferative process manifested as grade I or grade
II anemia. If more severe, (grade III or worse, hemoglobin < 8.0 g/dL), the anemia is likely
multifactorial with other processes, such as concurrent gastrointestinal bleeding contributing
to the etiology. Blood loss and inflammation may coexist, especially in patients with renal
failure on hemodialysis, in patients with gastrointestinal malignancy or inflammation, or
in patients with arthritis treated with corticosteroids or non-steroidal anti-inflammation
drugs. In those cases, determining the relative contributions of absolute iron deficiency and
functional iron deficiency can be challenging without performing a bone marrow analysis.

The Question
What is your approach to the diagnosis and management of the ACI?

Our Response
Overview
The traditional disease categories associated with ACI are malignancy, infection, and
connective tissue disorders. When excess cytokine production is a pathologic manifestation,
ACI may be associated with such processes as severe heart
failure and poorly controlled diabetes that fall outside of
Figure
the customary listing of chronic inflammatory diseases. ACI
is often subtle and insidious in onset, presenting as a mild,
persistent anemia that may worsen over time. Causes of the
underlying inflammation such as rheumatoid arthritis may
be obvious, or the basis of the inflammation may not be
immediately apparent as in cases of occult malignancy or
chronic infection.

Pathophysiology
Multiple mechanisms are responsible for the development of
ACI.1-3 Although erythrocytes in ACI have slightly shortened
survivals, the following three mechanisms, mediated by
inflammatory cytokines, exert major effects in sequential
periods of erythropoiesis (Figure): 1) inhibited survival and
differentiation of erythroid progenitor cells; 2) suppressed
EPO production; and 3) hepcidin-mediated sequestration
of reticuloendothelial iron. Chronic inflammatory states
increase production of cytokines such as IL-1, TNF-,
and interferon- that directly inhibit the survival and
differentiation of erythroid progenitor cells.1,2 Inflammatory

Inhibited Survival
and Differentiation

Supressed
EPO Production

Increased RES
Iron Sequestration

Stage of Erythroid Differentiation

HSC

BFU-E

CFU-E

Baso EB

Poly EB

Ortho EB

RET

RBC

l EPO dependence l l Hemoglobin synthesis l

KEY
BFU-E: Burst-forming unit-erythroid
CFU-E: Colony-forming unit-erythroid
Pro EB: Proerythroblast
Baso EB: Basophilic erythroblast
Poly EB: Polychromatophilic erythroblast
Ortho EB: Orthochromatic erythroblast
RES: Reticuloendothelial system
RET: Reticulocyte

N O N M A L I G N A N T H E M A T O L O G Y 37

Pro EB

Mechanisms of inflammatory cytokine inhibition of erythropoiesis in ACI.

Three mechanisms (shown in red) that are mediated by inflammatory cytokines are
key elements in the pathophysiology of ACI and represent potential therapeutic
targets. The main period of erythropoiesis in which each of the respective
mechanisms has its major effect is indicated by arrows. The stages of erythropoiesis
begin with the hematopoietic stem cell (HSC) and extend through the mature
erythrocyte (RBC). The periods of erythropoietin (EPO) dependency and hemoglobin
synthesis are shown below those specific stages of differentiation.

The Hematologist Compendium 2010-2015

Serum ferritin concentration is less than 20 ng/mL in uncomplicated iron deficiency anemia.
Although as an acute-phase protein, the ferritin concentration can be driven into the normal
range by the underlying inflammatory process, a ferritin concentration greater than 150 ng/
mL is rare in ACI patients who have concomitant absolute iron deficiency. As noted above,
low serum iron is characteristic of ACI, and low serum transferrin concentration and low
transferrin saturation are also observed routinely in ACI.

The patient described in the introduction was diagnosed with seronegative rheumatoid
arthritis, and two months after beginning therapy with prednisone and methotrexate, his
arthritis symptoms had improved and his Hgb was 12.5 g/dL.
1. Weiss G, Schett G. Anaemia in inflammatory rheumatic diseases. Nat Rev Rheumatol. 2013;9:205-215.
2. Davis SL, Littlewood TJ. The investigation and treatment of secondary anaemia. Blood Rev. 2012;26:65-71.
3. Ganz T, Nemeth E. Hepcidin and iron homeostasis. Biochim Biophys Acta. 2012;1823:1434-1443.

Transferrin receptor (TfR) expression is regulated post-transcriptionally by intracellular


iron concentration through the iron regulatory element (IRE)/IRE binding protein (IREBP)
system. When intracellular iron concentration is low, the IRE/IREBP system stabilizes TfR
mRNA, thereby increasing translation and protein expression. The effect of intracellular
iron concentration on TfR production led to development of a clinical test of iron status.
In this case, the concentration of TfR in plasma (soluble TfR or sTfR) serves as a surrogate
marker of iron status (i.e., the concentration of sTfR is elevated in absolute iron deficiency).
Subsequent studies suggested that the sensitivity of the assay in distinguishing absolute iron
deficient states from inflammatory processes that affect iron metabolism can be improved
by calculating the sTfR index by dividing the sTfR concentration by the log of the serum
ferritin concentration. Another proposed method for identifying iron deficiency is to measure
reticulocyte hemoglobin concentration (CHr) by flow cytometry. As the most recently
produced 1 percent of erythrocytes in the blood, the reticulocytes are the subpopulation most
affected by iron deficiency at the time the blood sample is obtained, and decreased CHr is a
sensitive indicator of iron-restricted erythropoiesis.5 Combining CHr with sTfR index has been
used to improve the identification of absolute iron deficiency in patients with inflammation.5
While these studies may have value is some particularly problematic cases, their clinical utility
is relatively modest. This interpretation is based on the fact that functional iron deficiency
plays an important role in the pathophysiology of ACI, and patients with functional iron
deficiency, without absolute iron deficiency, may benefit from supplemental iron. Therefore,
the practical value of distinguishing functional iron deficiency from absolute iron deficiency is
arguable because iron supplementation can be therapeutic in either case.

4. Miller CB, Jones RJ, Piantadosi S, et al. Decreased erythropoietin response in patients with the anemia of
cancer. N Engl J Med. 1990;322:1689-1692.
5. Thomas C, Thomas L. Biochemical markers and hematologic indices in the diagnosis of functional iron
deficiency. Clin Chem. 2002;48:1066-1076.
6. Pincus T, Olsen NJ, Russell IJ, et al. Multicenter study of recombinant human erythropoietin in
correction of anemia in rheumatoid arthritis. Am J Med.1990;89:161-168.
7. Papadaki HA, Kritikos HD, Valatas V, et al. Anemia of chronic disease in rheumatoid arthritis is
associated with increased apoptosis of bone marrow erythroid cells: improvement following anti-tumor
necrosis factor- antibody therapy. Blood. 2002;100:474-482.
8. Sasu BJ, Cooke KS, Arvedson TL et al. Antihepcidin antibody treatment modulates iron metabolism and
is effective in a mouse model of inflammation-induced anemia. Blood. 2010;115:3616-3624.
9. Steinbicker AU, Sachidanandan C, Vonner AJ, et al. Inhibition of bone morphogenetic protein signaling
attenuates anemia associated with inflammation. Blood. 2011;117:4915-4923.
10. Theurl I, Schroll A, Sonnweber T, et al. Pharmacologic inhibition of hepcidin expression reverses anemia
of chronic inflammation in rats. Blood. 2011;118:4977-4984.

Dr. Ferrell indicated no relevant conflicts of interest. Dr. Koury is a consultant with Keryx
Biopharmaceuticals, Inc. and the Pharmaceutical Division of Japan Tobacco, Inc.

Treatment
Although many patients have mild anemia that does not require treatment, establishing
a diagnosis of ACI is important as doing so implies an ongoing inflammatory process, the
etiology of which should be investigated. Furthermore, effective treatment of the underlying
disease results in improvement or resolution of the anemia. In some instances, however,
the underlying disease may be resistant to therapy (e.g., poorly responsive malignancy or
refractory connective tissue disease). In such cases, red cell transfusion support and therapy
that targets the pathophysiologic mechanisms that underlie ACI (Figure) are the mainstays
of management. ACI may respond to recombinant human EPO (rhEPO),6 but the response
may be blunted by concomitant functional iron deficiency. Iron supplementation, with a goal
transferrin saturation of > 20 percent, may increase the effectiveness of rhEPO, and some
patients respond to iron supplementation in the absence of rhEPO support. Infused iron rather
than oral iron supplementation is often needed as hepcidin restricts gastrointestinal iron
absorption. The desired response (increase in transferrin saturation) to iron supplementation,
however, is relatively short lived, and repeated iron infusion puts patients at risk for iatrogenic
hemochromatosis1,2 because the high hepcidin prevents the recycling of this iron. AntiTNF- agents have been shown to increase hemoglobin concentration in patients with ACI
independent of an effect on EPO concentration.7 Looking forward, suppression of hepcidin
activity has been examined in animal models. In a mouse model of ACI, antibodies to hepcidin
also required the concurrent administration of EPO to reverse the anemia,8 whereas inhibition
of hepcidin transcription, by blocking BMP receptor activation or downstream signaling,
prevented and reversed ACI without the need for exogenous EPO administration.9,10 As
more therapeutic options become available, well-designed clinical studies will be needed to
determine the optimal pharmacologic approach to the management of ACI.

Watch ASH
Webcasts and
Webinars on ASH
On Demand
View a wide range of online ASH
educational materials including
webcasts of ASH live meetings
and informative webinars. Visit
ashondemand.org for access.

N O N M A L I G N A N T H E M A T O L O G Y 38

The Hematologist Compendium 2010-2015

Approach to the Diagnosis and Management of Transfusion-Related


Acute Lung Injury July August 2013
The authors have indicated that there has been no update regarding the content of this article since the original publication date in 2013.

HUY P. PHAM, MD, 1 AND STEVEN L. SPITALNIK, MD 2


1. Assistant Professor of Pathology, UAB Medicine, Baptist Health,
Birmingham, AL
2. Professor of Pathology and Cell Biology; Vice Chairman, Laboratory
Medicine, College of Physicians and Surgeons, Columbia University, New York,

The Question
What is your approach to the diagnosis and management of transfusion-related acute lung
injury (TRALI)?

Our Response
Epidemiology
The blood supply in the United States is safe. Although non-life-threatening adverse events
such as allergic and febrile transfusion reactions are encountered regularly by clinicians,
transfusion-related fatalities are rare. From 2007 through 2011, 212 fatalities following blood
collection and transfusion were reported to the FDA.1 Non-infectious complications pose the
greatest mortality risk to the transfused patient with TRALI accounting for 43 percent of deaths
and hemolytic transfusion reactions due to ABO (10%) and non-ABO (13%) incompatibility
accounting for 23 percent.1 In comparison, 11 percent of transfusion-related deaths were due
to microbial infections. Although usually associated with infusion of blood products containing
high volumes of plasma (e.g., fresh-frozen plasma and platelets), TRALI has also been linked
to red blood cell transfusions. Estimates of the incidence of TRALI may be influenced by
factors such as transfusion policy. For example, the incidence fell from 2.57 to 0.81 per 10,000
transfusions concurrent with reduction in the use of plasma from female donors.2

Clinical Presentation and Diagnosis


The diagnosis of TRALI is made on clinical grounds, and no single laboratory or radiologic
test definitively identifies or excludes this entity. We currently use the criteria and
case definitions proposed by a Canadian Consensus Conference (Table).3 Thus, TRALI
is an acute event presenting during a transfusion or within six hours of its completion.
Characteristic signs and symptoms include fever, chills, dypsnea, hypoxemia, hypotension
(or possible transient hypertension), and the new onset of bilateral non-cardiogenic
pulmonary edema (e.g., chest x-ray showing bilateral alveolar and interstitial infiltrates
in the absence of cardiomegaly). TRALI is often associated with transient leukopenia or
neutropenia. A diagnosis of Possible TRALI is made based on the same criteria as TRALI
except that an alternative risk factor for acute lung injury is present concurrently (Table).
The differential diagnosis of TRALI includes the following: transfusion-associated
circulatory overload (TACO), anaphylaxis, and sepsis.4 Distinguishing TACO from
TRALI may be challenging because some of the signs and symptoms of the two entities
overlap; in addition, the two processes can occur concurrently in a given patient. The key
difference between these two conditions is the pathophysiologic origin of the pulmonary
edema (i.e., cardiogenic in the case of TACO and non-cardiogenic in the case of TRALI).
Thus, clinical improvement after treatment with a diuretic and/or an inotropic agent
is characteristic of TACO, but not TRALI. Other findings suggestive of TACO include
persistent hypertension, a post-transfusion brain natriuretic peptide (BNP) level of at least
100 pg/mL, and a post-transfusion:pre-transfusion BNP ratio of >1.5.5 Although anaphylaxis
can present with hypotension, cyanosis, and hypoxia due to bronchospasm and laryngeal
edema, the absence of fever and pulmonary edema distinguish this process from TRALI.
Although transfusion-induced sepsis, particularly after a platelet transfusion, typically
presents with pyrexia and hypotension, respiratory distress is an infrequent complication.
Other entities, such as myocardial infarction, pulmonary embolism, and other causes
of acute lung injury share clinical features with TRALI and should be considered in the
differential diagnosis.
The diagnosis of TRALI is particularly challenging in complex inpatients such as those
encountered in the intensive care unit setting, given that such patients often have multiple
medical problems and may exhibit some symptoms of TRALI even before transfusion.

of the donor product could induce the second hit. Together, these processes induce
capillary endothelial damage, resulting in vascular permeability and pulmonary edema.6-8
Based on this proposed mechanism, one might hypothesize that the incidence of TRALI
would be higher for plasma-rich transfusion products and that the incidence would be
lower when plasma derived from female donors (who have a higher prevalence of antiHLA antibodies) is restricted; available data support these two hypotheses.2 In addition,
recent studies indicate that circulating platelets are involved in TRALI pathophysiology,
suggesting that anti-platelet therapy may be beneficial clinically.7,9

Management
The first step in managing any suspected transfusion reaction is to stop the transfusion.
Once the patient is stabilized, the episode should be reported to the transfusion medicine
service so that a transfusion reaction evaluation can begin. Because hemolytic transfusion
reactions are associated with significant morbidity and mortality, the transfusion medicine
service will first perform a clerical check to ensure that the correct unit was transfused
into the correct patient. Next, the ABO type of the patient and the transfused unit will
be confirmed, the post-transfusion blood sample will be inspected for visible evidence
of hemolysis, and an indirect and direct anti-globulin test will be performed on the posttransfusion sample to determine if circulating and/or red blood cell-bound antibodies are
present. Additional laboratory tests to investigate for hemolysis, including a complete
blood count, urinalysis, and plasma concentration of bilirubin, lactate dehydrogenase,
and haptoglobin, are often needed. To diagnose TRALI, physical exam, chest x-ray, and
arterial blood gas studies are recommended. In distinguishing TRALI from TACO, an
echocardiogram may be useful in determining whether the observed pulmonary edema
is of cardiogenic origin. Other causes of adverse events that share clinical features with
TRALI (e.g., sepsis, myocardial infarction, and pulmonary embolus) should be promptly
investigated.
Current management of TRALI consists of respiratory and circulatory support based on
clinical severity. Oxygen supplementation is required in almost all patients; in severe
cases, mechanical ventilation may be necessary. Hypotensive episodes can be treated
with pressors. Corticosteroid treatment has not improved outcome.10 In theory, the noncardiogenic pulmonary edema of TRALI should not respond to diuresis. Most patients
improve within two to three days, but those who do not improve over this period typically
have a protracted clinical course or a fatal outcome.8
Patients who have experienced an episode of TRALI are not at greater risk for a second
episode, assuming that the initial event was a consequence of infusion of donor antibodies
that were present in the transfusion product and that subsequent blood products do not

Table 1: Canadian Consensus Conference on the Definition of TRALI


TRALI:
1) Acute lung injury

a. Acute onset

b. Hypoxemia

i. Research setting: PaO2/FiO2 < 300 and/or SpO2 < 90% on room air

ii. Non-research setting: PaO2/FiO2 < 300 and/or SpO2 < 90% on room
air and/or other clinical symptoms of hypoxia

c. Bilateral infiltrates on frontal chest x-ray

d. No evidence of left atrial hypertension (i.e., no circulatory overload)

2) No pre-existing acute lung injury before transfusion


3) During or within 6 hours of transfusion
4) No temporal relationship to an alternative risk factor for acute lung injury
Possible TRALI:

Pathophysiology

1) Acute lung injury

Although the exact mechanism of TRALI is uncertain, a two-hit process has been
proposed.6-8 According to this hypothesis, the first hit is induced by an underlying
condition, such as trauma or sepsis, which primes granulocytes and/or activates
endothelial cells, thereby causing neutrophils to become sequestered in the pulmonary
vasculature. The second hit results from passive infusion of donor antibodies in the blood
product that recognize either human leukocyte antigens (HLA) on recipient endothelial
cells or human neutrophil antigens (HNA) on recipient neutrophils. Alternatively (or in
addition), infusion of biologic response modifiers (e.g., CD40 ligand) in the plasma portion

2) No pre-existing acute lung injury prior to transfusion

N O N M A L I G N A N T H E M A T O L O G Y 39

3) During or within 6 hours of transfusion


4) A clear temporal relationship to an alternative risk factor for acute lung injury

Kleinman S et al. Transfusion. 2004;44:1774-1789.

The Hematologist Compendium 2010-2015

come from the initial donor. However, notification of the transfusion medicine service
about a possible TRALI episode has important implications for the donor and the safety
of the blood supply. If TRALI is suggested by both clinical presentation and laboratory
results (e.g., finding anti-HLA and/or anti-HNA antibodies in the transfused blood product
that match the corresponding antigens in the patient), the donor must be evaluated for
consideration of their continued eligibility to donate. Indeed, many centers choose to
exclude such donors permanently. That the product from a particular donor caused an
episode of TRALI may be difficult to prove unequivocally, however, because patients often
receive products from multiple donors.

Prevention
Approaches to reducing the risk of TRALI have included avoiding the use of plasma from
female donors, using plasma derived only from males or from never-pregnant females,
and testing female donors for anti-HLA antibodies.1,2 Although these measures reduce
the incidence, they do not completely eliminate risk because TRALI can be induced
by other blood products (e.g., red blood cells, platelet concentrates, cryoprecipitate).
In addition to avoiding the use of high-risk blood products, conservative transfusion
strategies and interventions that address the first hit could also help reduce TRALI
incidence. Nonetheless, TRALI continues to be the most common cause of transfusionrelated mortality, making rapid recognition and institution of appropriate supportive care
imperative.1,7

1. Fatalities reported to FDA following blood collection and transfusion: annual summary for
fiscal year 2011. www.fda.gov/BiologicsBloodVaccines/SafetyAvailability/ReportaProblem/
TransfusionDonationFatalities/ucm302847.htm. 2012. Accessed March 31, 2013.
2. Toy P, Gajic O, Bacchetti P, et al. Transfusion-related acute lung injury: incidence and risk factors.
Blood. 2012;119:1757-1767.
3. Kleinman S, Caulfield T, Chan P, et al. Toward an understanding of transfusion-related acute lung injury:
statement of a consensus panel. Transfusion. 2004;44:1774-1789.
4. Mazzei CA, Popovsky MA, Kopko PM. Noninfectious complications of blood transfusion. In: Roback JD,
Grossman BJ, Harris T, et al. (Eds). Technical Manual, 17th ed. AABB. 2011:272-762.
5. Zhou L, Giacherio D, Cooling L, et al. Use of B-natriuretic peptide as a diagnostic marker in the
differential diagnosis of transfusion-associated circulatory overload. Transfusion. 2005;45:1056-1063.
6. Silliman CC, Boshkov LK, Mehdizadehkashi Z, et al. Transfusion-related acute lung injury: epidemiology
and a prospective analysis of etiologic factors. Blood. 2003;101:454-462.
7. Gilliss BM, Looney MR. Experimental models of transfusion-related acute lung injury. Transfus Med Rev.
2011;25:1-11.
8. Popovsky MA, Moore SB. Diagnostic and pathogenetic considerations in transfusion-related acute lung
injury. Transfusion. 1985;25:573-577.
9. Caudrillier A, Kessenbrock K, Gilliss BM, et al. Platelets induce neutrophil extracellular traps in
transfusion-related acute lung injury. J Clin Invest. 2012;122:2661-2671.
10. Steinberg KP, Hudson LD, Goodman RB, et al. Efficacy and safety of corticosteroids for persistent acute
respiratory distress syndrome. N Engl J Med. 2006;354:1671-1684.

Dr. Pham and Dr. Spitalnik indicated no relevant conflicts of interest.

Make Use of ASHs


Consult-a-Colleague Service
This service for ASH members helps facilitate the
exchange of information between hematologists
and their peers. Members can seek consultation on
clinical cases relating to categories including anemias,
hematopoietic cell transplantation, hemoglobinopathies,
hemostasis/thrombosis, lymphomas, lymphoproliferative
disorders, leukemias, multiple myeloma & Waldenstrm
Macroglobulinemia, myeloproliferative disorders,
myelodysplastic syndromes, and thrombocytopenias.
Email your inquiries to ashconsult@hematology.org, or
visit www.hematology.org/Clinicians/Consult.aspx for
more information.

N O N M A L I G N A N T H E M A T O L O G Y 40

The Hematologist Compendium 2010-2015

Criteria for Selecting Patients with Sickle Cell Anemia for Allogeneic
Hematopoietic Stem Cell Transplantation September/October 2013
The author has indicated that there has been no update regarding the content of this article since the original publication date in 2013.

CHARLES T. QUINN, MD, MS


Associate Professor of Pediatrics and Medical Director, Pediatric Sickle Cell Program; Medical
Director, Erythrocyte Diagnostic Laboratory, Cincinnati Childrens Hospital Medical Center,
Cincinnati, OH

The Question
What criteria do you use for selecting patients with sickle cell anemia (SCA) for allogeneic
hematopoietic stem cell transplantation (HSCT)?

My Response
About 40 years ago, only half of the children born with SCA in the United States were
expected to live long enough to reach adulthood. Now, however, as a result of the high
quality of care available in comprehensive sickle cell centers, disease-related mortality during
childhood has been reduced to 1 to 2 percent.1,2 This remarkable improvement in survival
stems mainly from interventions aimed at preventing early deaths from infection and splenic
sequestration, as well as the greater use of disease-modifying therapies (e.g., hydroxyurea
and chronic transfusions). No current U.S. statistics are available, but in a study published in
2001, the median lifespan for individuals with SCA in a Jamaican cohort was estimated to be
more than 50 years.3 Not reflected in these improving survival data, however, is the burden
of SCA-related chronic organ injury acquired in early childhood that becomes especially
manifest in young adulthood, adversely affecting the quality and duration of the remaining
life of the patient. The burden of morbidity and mortality in SCA has now simply shifted to
adults, and the period after transition to adult medical care is associated with a particularly
high risk of death, especially from acute chest syndrome and multi-organ failure syndrome.1
HSCT is the only available cure for SCA, and more than 500 transplants for SCA have been
reported to international registries. Although usually successful and curative, the widespread
use of HSCT is still limited by the lack of sufficient suitable donors and concerns about regimenrelated morbidity and mortality. HSCT is safest when an HLA-matched sibling donor is available,
but only about 10 percent of transplant candidates will actually have such a donor. Current
estimates place regimen-related mortality for myeloablative transplantation using an HLAmatched sibling donor at about 5 percent, with a concomitant 9 percent risk of graft rejection
and a 15 percent risk of chronic graft-versus-host disease (GVHD) (Table).4,5 There are additional
late effects of transplantation not tabulated here, including infertility, endocrinopathies,
premature cardiovascular disease, and possibly cancer. One can argue that SCA-related
mortality during childhood (in patients cared for in comprehensive centers in developed
nations) is now lower than the regimen-related mortality of HSCT, but this comparison does not
integrate the lifelong risk of progressive morbidity, impaired quality of life, and early mortality
due to SCA. Of course, there is no randomized comparison of transplantation versus no
transplantation that accurately quantifies the relative lifetime risks of morbidity and mortality
from SCA and HSCT, but life after a successful transplant the event-free survival (EFS) of a
full-sibling-donor myeloablative HSCT is approaching 90 percent (Table) is arguably better
than a life with SCA. An important caveat, however, is that the definition of EFS differs by study
and often does not include chronic GVHD, which may be worse than SCA, so EFS rates that are
meaningful to patients are usually lower than reported (70-80% vs. 90%, respectively).

chronic transfusions. If there is continued interest in pursuing HSCT, then I arrange formal
consultation and counseling with the HSCT team. This multi-step process does not lead to
HSCT for most patients, but they are at least aware and knowledgeable about the option, and I
support them with decision-making.
The second scenario is a direct inquiry about HSCT by a patient or parent, oftentimes as a
result of something heard or read about on the Internet. Recent immigrants from Africa also
tend to ask commonly about transplantation.
The third scenario occurs when a patient suffers a particularly serious complication, such
as overt stroke. In these instances, I follow the same general process outlined above in
counseling patients and their families about HSCT.
In essence, I consider every patient with SCA (here I include both HbSS and sickle-0thalassemia) who has an HLA-matched sibling donor to be a potential candidate for
myeloablative HSCT. Honest discussions about the risks, benefits, and alternatives are
needed, regardless of the severity of a patients disease, and the decision to pursue HSCT
is a lengthy and patient- and family-centered process. Once a decision to perform HSCT
is made, I recommend that it be done during early childhood. My opinions and practices
are based upon the assumption that my patients HSCT will be performed in a center with
substantial experience in transplantation for SCA and, ideally, as part of a clinical research
study. The use of alternative donors (unrelated, mismatched, or haploidentical) and stem
cell sources (unrelated umbilical cord blood) for children carries a much higher risk of
transplant-related morbidity, mortality, and graft rejection and should be performed only as
part of a carefully designed investigational study.
Adults with SCA tolerate myeloablative conditioning regimens poorly, likely because of the
cumulative burden of chronic organ injury acquired during childhood. However, there is
very promising early experience with a non-myeloablative conditioning regimen (including
total-body irradiation, alemtuzumab, and sirolimus) in severely affected adults with SCA that
can produce stable, mixed donor-recipient chimerism that effectively cures the disease.6
These outcomes have been achieved without chronic GVHD or mortality and with a low
rate of graft rejection. The mixed chimerism appears to persist after discontinuation of
immunosuppression.7 In the future, adults may no longer be summarily excluded from HSCT.
Until then, all transplants in adults should be performed as part of a clinical research study
with the indications being dictated by the eligibility criteria of the particular study.
While the experience with adult recipients and alternative donors matures, research needs
to focus on reducing transplant-associated morbidity, especially sterility, in young patients
undergoing reduced-intensity conditioning regimens. Ongoing studies will continue to expand
access to HSCT for both children and adults, and the next Ask the Hematologist question on
this topic might also include the option of gene therapy.
1. Quinn CT, Rogers ZR, McCavit TL, et al. Improved survival of children and adolescents with sickle cell
disease. Blood. 2010;115:3447-3452.
2. Telfer P, Coen P, Chakravorty S, et al. Clinical outcomes in children with sickle cell disease living in
England: a neonatal cohort in East London. Haematologica. 2007;92:905-912.
3. Wierenga KJ, Hambleton IR, Lewis NA, et al. Survival estimates for patients with homozygous sickle-cell
disease in Jamaica: a clinic-based population study. Lancet. 2001;357:680-683.
4. Locatelli F, Pagliara D. Allogeneic hematopoietic stem cell transplantation in children with sickle cell
disease. Pediatr Blood Cancer. 2012;59:372-376.
5. Lucarelli G, Isgr A, Sodani P, et al. Hematopoietic stem cell transplantation in thalassemia and sickle

cell anemia. Cold Spring Harb Perspect Med. 2012;2:a011825.


Expert panels and professional societies provide differing sets of clinical indications for
HSCT as a treatment option for SCA, but common recommendations include recurrent
6. Hsieh MM, Kang EM, Fitzhugh CD, et al. Allogeneic hematopoietic stem-cell transplantation for sickle
cell disease. N Engl J Med. 2009;361:2309-2317.
vaso-occlusive complications (acute chest syndrome and painful events), usually despite
7. Hsieh MM, Fitzhugh CD, Tisdale JF. Allogeneic hematopoietic stem cell transplantation for sickle cell
hydroxyurea therapy, and overt cerebrovascular disease. Other indications have been
disease: the time is now. Blood. 2011;118:1197-1207.
proposed and used, such as abnormal transcranial Doppler velocities and silent cerebral
infarction, despite much less agreement among experts and very limited data. Some
hematologists now also advocate that a diagnosis of SCA itself is an indication for HSCT,
Dr. Quinn indicated no relevant conflicts of interest.
justified by the knowledge that complications of SCA are often unpredictable, and most
individuals with SCA will have progressively severe morbidity and early mortality. In my
clinical practice, I do not compare a fixed list of indications
for HSCT with an ongoing tabulation of the number and type
of SCA-related complications experienced by the patient.
Table. International Studies of Myeloablative HSCT for SCA Using Matched-Sibling Donors
Rather, I initiate (or continue) the discussion of HSCT in
three main clinical scenarios.
Vermylen
Walters
Locatelli
Bernaudin
Panepinto
Lucarelli
Weighted

The first usual scenario is planned. I ensure that all patients


with SCA and their parents know about the possibility of
cure by HSCT. I include discussion of the option of HSCT
as part of the comprehensive, ongoing education about
SCA starting with the very first visit after diagnosis (usually
the result of newborn screening). I also recommend tissue
typing of all unaffected full siblings, including those with
sickle trait, to know if there is an HLA-matched donor. If
there is a matched-sibling donor, I offer dedicated clinic
visits for frank discussions about the risks and benefits
of myeloablative HSCT compared with life with SCA, as
well as the risks and benefits of hydroxyurea therapy and

OS

1998
N=50

2000
N=50

2003
N=11

2007
N=87

2007
N=67

2011
N=11

Average
N=276

93

94

100

92

97

90

94

RRM

10

Rejection

10

10

13

cGVHD

20

12

13

22

36*

17

EFS

82

90

86

85

90

85

84

Data compiled from tables in Locatelli et al. and Lucarelli et al.


Values are shown as the percentage of the N value for the corresponding study.
Abbreviations: OS, overall survival; RRM, regimen-related mortality; cGVHD, chronic graft-versus-host disease; EFS, event-free survival.
*Four of 11 patients had mild, chronic GVHD of the skin that resolved completely after steroid therapy.

N O N M A L I G N A N T H E M A T O L O G Y 41

The Hematologist Compendium 2010-2015

Diagnosis and Management of Platelet Alloimmunization November/December 2013


UPDATE/COMMENTARY
Platelet refractoriness remains a clinical challenge associated with an increased
risk of bleeding, prolonged hospital stays, and decreased survival. Poor one-hour
post-transfusion increments typically represent immune platelet refractoriness,
whereas an adequate one-hour increment followed by a suboptimal 18- to 24-hour
count suggests peripheral consumption or sequestration. In the absence of a direct
prospective comparison of the three strategies commonly used to support patients
who have platelet alloimmunization human leukocyte antigen (HLA) matching,
HLA antibody avoidance, and platelet cross-matching two recent reviews have
concluded there is no definitive evidence that any of these strategies improves
hemorrhage control or mortality.U1,8 The provision of HLA-matched platelets can
be very efficient in centralized transfusion systems as it has been in the United
Kingdom.U2
Apheresis platelets stored in additive solution, which contain 2/3 less donor plasma,
have become available in the United States, with a longer track record in Europe.
They are associated with lower platelet increments at one hour, but are less likely to
cause allergic transfusion reactions or hemolysis, compared with standard platelets
suspended in 100 percent donor plasma.U3 Platelets that have undergone psoralenUVA treatment to inactivate bacteria (and other infectious agents) are also known
to produce lower post-transfusion increments. These considerations need to be
taken into account when evaluating patients who seem to be refractory to platelet
transfusion.

A recent study in an animal model suggests that immune-mediated clearance of MHC


mismatched platelets can occur in the absence of anti-platelet alloantibodies, in mice
that are deficient in B cells. Allo-reactive CD8+ T cells appear to mediate antibodyindependent platelet clearance.U4,U5 If this finding is corroborated in humans,
immune platelet refractoriness may actually account for some refractory cases now
presumed to be non-immune. A putative T-cellmediated mechanism would also
favor HLA matching over cross-matching as a management strategy.
U1. Vassallo RR, Fung M, Rebulla P, et al. Utility of cross-matched platelet transfusions in patients
with hypoproliferative thrombocytopenia: a systematic review. Transfusion. 2014;54:1180-1191.
U2. Stanworth SJ, Navarrete C, Estcourt L, et al. Platelet refractoriness practical approaches and
ongoing dilemmas in patient management. Br J Haematol. 2015;171:297-305.
U3. Tobian AA, Fuller AK, Uglik K, et al. The impact of platelet additive solution apheresis
platelets on allergic transfusion reactions and corrected count increment (CME). Transfusion.
2014;54:1523-1529.
U4. Arthur CM, Patel SR, Sullivan HC, et al. CD8+ T cells mediate antibody-independent platelet
clearance in mice. Blood. 2016;127:1823-1827.
U5. Manis JP, Silberstein LE. Platelet refractoriness: its not the B-all and end-all. Blood.
2016;127:1740-1741.

HLA alloimmunization is prevented by the transfusion of leukoreduced red blood cells and platelets.
KAREN QUILLEN, MD, MPH
Medical Director, Blood Bank, Boston Medical Center; Professor of Medicine and Pathology,
Boston University School of Medicine, Boston, MA

The Question
What is your approach to the diagnosis and management of platelet alloimmunization?

My Response
Platelet refractoriness occurs in 5 to 15 percent of patients who receive chronic platelet
transfusions.1 The patients size and the number of platelets transfused should be factored
into the assessment of refractoriness. For example, one measure, the corrected count
increment (CCI), is computed as follows: CCI = (platelet increment after transfusion/l)
x (body surface area in m2) (platelet dose x 1011). For the purposes of this calculation,
assume that each single-donor apheresis unit contains 3 x 1011 platelets or that each wholeblood-derived platelet concentrate contains 5.5 x 1010 platelets. Using this formula, if the
platelet count increased by 20,000/L in a patient who had a body surface area of 2.0 m2 and
who received one apheresis unit of platelets, the CCI is 20,000/L x 2 by 3 = 13,333/L.
Refractoriness is defined as a CCI value below 2,500 platelets/L at 18 to 24 hours posttransfusion or a value below 4,500 platelets/L at 10 to 60 minutes post-transfusion2 on two
to three consecutive platelet transfusion episodes. A less rigorous approach is to assume
that, for an average size, non-refractory adult, the platelet count increment will be at least
10,000/l to 20,000/L one hour after the transfusion of either one unit of apheresis platelets
or an equivalent dose of pooled platelets (5-6 combined random-donor platelet concentrates
are equivalent to 1 apheresis unit), recognizing that the number of platelets per component
unit may vary widely.

Non-immune causes of platelet refractoriness predominate.


Non-immune factors are present in the majority (72-88%) of transfusion-refractory patients,
and immune causes are present in a minority (25-39%).2 Non-immune factors, most of which
are prevalent in a high proportion of hematology-oncology patients requiring prolonged
platelet transfusion support, include splenomegaly, fever, infection, DIC, bleeding, and drugs
such as vancomycin and amphotericin, are associated with platelet refractoriness.2,3 The
mechanism for refractoriness associated with various drugs is partially mediated by drugdependent platelet antibodies, although specific testing for such is not widely available and
unlikely to be of practical help.

Antibodies to HLA antigens account for the overwhelming majority of cases of immune
platelet refractoriness, with antibodies to platelet-specific antigens being much less
common. HLA alloimmunization may occur in response to prior pregnancies or to
transfusions, although only a subset of alloimmunized individuals demonstrates immune
platelet refractoriness. The TRAP study4 showed that filtration-removal of leukocytes and
ultraviolet B irradiation to inactivate leukocytes were equally effective in preventing
the development of platelet transfusion refractoriness, which occurred in 16 percent of
control patients, compared with 7 to 10 percent of patients who received leuko-reduced or
irradiated platelets. On the other hand, HLA antibodies developed in 45 percent of control
patients compared with 17 to 21 percent of intervention patients. Outcomes were equivalent
for filtered apheresis platelets and for filtered pooled platelets. In Canada, universal
prestorage leuko-reduction of platelets has lessened the incidence of alloimmune platelet
refractoriness from 14 to 4 percent.5 Almost all apheresis platelet units and more than 80
percent of packed red blood cell units in the United States are leuko-reduced by filtration
either at the time of collection or immediately prior to storage.

HLA typing and antibody testing are complementary approaches.


Platelets express only HLA Class I antigens. For patients who are candidates for allogeneic
stem cell transplantation, HLA typing results may already be known. Most HLA laboratories
have adopted high-throughput molecular methods for genotyping HLA Class I and II
antigens. A sequence-specific oligonucleotide probe method requires only small amounts
of DNA and therefore can be performed on samples from neutropenic patients. Lowresolution HLA-A and B typing (Class I antigens) is adequate for the management of platelet
alloimmunization, while high-resolution typing, including sequencing, is reserved for HLA
Class II typing to select stem cell donors.
HLA antibody detection can be performed using a variety of methods.3 Multiplex flow
cytometric bead assays are more sensitive than traditional ELISA.6 In the former assay,
patient serum or plasma is incubated with color-coded microbeads that are coated with
HLA antigens. Flurochrome-labeled anti-human IgG is added, and a flow analyzer is used
to determine the color code of the reactive beads with a computer algorithm determining
the specific antigens to which the antibody is reactive. The panel-reactive antibody
(PRA) represents the percent of HLA targets to which the patient has made antibodies.
PRA can be determined by using the traditional lymphocytotoxic assay, by ELISA, or by
fluorescence-based microbead method. Serial assays are useful in assessing candidates for
organ transplantation, but less so for management of platelet-refractory patients because
a numerical PRA result (the percentage of the population to which the patient has HLA
antibodies) does not provide actionable information to guide platelet selection.

Use of fresh ABO-matched platelets can improve transfusion


response.

Strategies to select platelets for refractory patients include HLAmatching, avoidance of known HLA antibody specificities, and
platelet cross-matching.

Platelets express blood group A and B antigens, but they are often transfused without
ABO matching. Major ABO incompatibility (such as group A platelets transfused into a
group O recipient) can decrease post-transfusion increments. A trial of fresh ABO-matched
platelets can be a valuable temporizing measure while investigation of the basis of platelet
transfusion refractoriness is ongoing.

For the purpose of platelet donor selection, a grading system (with designated categories
A, B1, B2 C, D) is employed. A perfect four-antigen match (2 loci each at HLA-A and HLAB) is grade A. In a B1 match, all of the donors HLA antigens are present in the recipient,
but the donor lacks one of the recipients HLA antigens; in a B2-match, all of the donors
HLA antigens are present in the recipient, but the donor lacks two of the recipients HLA

N O N M A L I G N A N T H E M A T O L O G Y 42

The Hematologist Compendium 2010-2015

antigens; in a C-match, the donor has one HLA antigen that


is not present in the recipient; in a D-match, the donor
has more than one HLA antigen that is not present in the
recipient. High-grade matched donors (grade A, B1, or B2)
are specifically recruited to donate platelets for a particular
patient, but transfusion with grade C or D matched units
is unlikely to produce a clinically meaningful incremental
increase in the platelet count. This grading approach to
matching also allows for categorization of antigens into
cross-reactive groups (CREG). For example, HLA-A1 and
A36 are within the same CREG, so if a patient has the A36
antigen and no available donor platelet is A36 positive,
then an A1 donor platelet typically more prevalent in the
Caucasian population can be used in a grade B match. HLA
Matchmaker is an epitope-based computer algorithm used in
some centers to identify permissible donor platelets that are
more likely to yield adequate platelet increment increases
without being HLA matched.7 Despite the resources invested
in the management of patients who are refractory to platelet
transfusion, a recent review of the literature identified no
studies that were adequately powered to detect an effect
of transfusion of HLA-matched platelets on mortality
or hemorrhage.8 Prospective studies utilizing current
technology and examining clinical outcomes are needed
to evaluate the effectiveness of HLA-matched platelet
transfusion.8

HLA-matched platelets

Advantages

Disadvantages

HLA type may already be on file for


allogeneic SCT candidates

Testing takes up to 1 week to


complete.

One-time test per patient


Some haplotypes are relatively common such as A1 B8 in Caucasians.

Perfect matches are rare.

Grade B matches or HLA Matchmaker increase the number of donor


possibilities.
Avoidance of known HLA-antibody
specificities

Many more donor possibilities compared with HLA matching.

Antibody testing takes several days.


Antibody testing should be repeated
periodically.

Platelet cross-matching

For management of the transfusion-refractory patient, available data argue that selection of
donors with HLA antigens against which the recipient does not have antibodies is a better
strategy than HLA matching. In one observational study involving 29 refractory patients
and a database of more than 7,000 HLA-typed donors, a mean of 39 donors were HLA grade
A or B matched, but a mean of 1,426 donors were identified as permissible by antibody
exclusion.9 Post-transfusion platelet count increments were comparable for the two
strategies. HLA antibody testing should be repeated at periodic intervals because antibody
specificities may evolve over time.
Platelet cross-matching tests the patients serum against samples of available donor
apheresis platelets using a solid-phase adherence assay or an ELISA. A recent study found a
mean CCI of 7,000 at one hour in more than 400 cross-matched platelet doses transfused to
71 refractory patients.10 Platelet cross-matching can be done within a few hours compared
with several days for HLA testing, but it does involve frequent repeat testing because
the shelf-life of platelets is five days. Automated platforms are invaluable in making this
approach efficient and practical.1
A comparison of some of the advantages and disadvantages of the three strategies for
dealing with refractoriness to platelet transfusion is contained in the table. Choice of
method depends on local resources, and communication between the hematologist and the
transfusion service is critical to ensure that donor selection is appropriate and that valuable
resources are not wasted or used inappropriately.

Testing takes several hours

Testing is repeated for each platelet


search, labor-intensive unless
automated.

Applicable to apheresis or pooled


whole-blood-derived platelets

Commercial kits have been temporarily unavailable in the United States.

Anti-fibrinolytic agents can be a useful adjunct for mucosal


bleeding.
Other approaches to ameliorating the consequences of alloimmune platelet refractoriness
include infusion of IVIgG, citric acid treatment of platelets to remove Class I HLA epitopes,
and infusion of recombinant activated FVII in actively bleeding patients. Despite anecdotal
reports of success, none of these approaches has been validated for clinical use. Use of
family members as platelet donors for patients who are potential allogeneic SCT candidates
is controversial, based on a theoretical concern for inducing alloimmunization that may
jeopardize engraftment. Anti-fibrinolytic agents such as epsilon-aminocaproic acid and
tranexamic acid, however, can be useful in platelet-refractory patients with oral mucosal
bleeding.
1. Rebulla P. A mini-review on platelet refractoriness. Haematologica. 2005;90:247-253.
2. Stroncek DF, Rebulla P. Platelet transfusions. Lancet. 2007;370:427-438.
3. Hod E, Schwartz J. Platelet transfusion refractoriness. Br J Haematol. 2008;142:348-360.
4. The Trial to Reduce Alloimmunization to Platelets Study Group. Leukocyte reduction and ultraviolet B
irradiation of platelets to prevent alloimmunization and refractoriness to platelet transfusions. N Engl J
Med. 1997;337:1861-1870.
5. Seftel MD, Growe GH, Petraszko T, et al. Universal prestorage leukoreduction in Canada decreases
platelet alloimmunization and refractoriness. Blood. 2004;103:333-339.
6. Fadeyi E, Adams S, Peterson B, et al. Analysis of a high-throughput HLA antibody screening assay for use
with platelet donors. Transfusion. 2008;48:1174-1179.
7. Nambiar A, Duquesnoy RJ, Adams S, et al. HLAMatchmaker-driven analysis of responses to HLA-typed
platelet transfusions in alloimmunized thrombocytopenic patients. Blood. 2006;107:1680-1687.
8. Pavenski K, Rebulla P, Duquesnoy R, et al. Efficacy of HLA-matched platelet transfusions for patients with
hypoproliferative thrombocytopenia: a systematic review. Transfusion. 2013;53:2230-2242.
9. Petz LD, Garratty G, Calhoun L, et al. Selecting donors of platelets for refractory patients on the basis of
HLA antibody specificity. Transfusion. 2000;40:1446-1456.
10. Wiita AP, Nambiar A. Longitudinal management with crossmatch-compatible platelets for refractory
patients: alloimmunization, response to transfusion, and clinical outcomes (CME). Transfusion.
2012;52:2146-2154.

WATCH THE EXPERTS

Dr. Quillen indicated no relevant conflicts of interest.

Adam Cuker, MD, MS


Dr. Adam Cuker discusses the latest developments
in monitoring and administering oral anticoagulants.
He presented on this topic at the 2015 ASH
Annual Meeting in a session titled Evolving Issues
in Anticoagulation. Watch the video and the rest
of the ASH Impact Series on ASH On Demand at
www.ashondemand.org/FreeContent/Videos.

Read an article by Dr. Cuker in this Compendium on page 44.

N O N M A L I G N A N T H E M A T O L O G Y 43

The Hematologist Compendium 2010-2015

Recommendation of Light Transmission Aggregometry as Part of the


Evaluation of a Patient with a Suspected Bleeding Disorder March/April 2014
The author has indicated that there has been no update regarding the content of this article since the original publication date of 2014.

Figure
ADAM CUKER, MD, MS
Assistant Professor of Medicine, and of Pathology
and Laboratory Medicine, University of Pennsylvania,
Philadelphia, PA

The Question
When do you recommend light transmission aggregometry as
part of the evaluation of a patient with a suspected bleeding
disorder?

My Response
Light transmission aggregometry (LTA) remains the reference
method for measurement of platelet function in patients with
suspected platelet function disorders (PFDs). Remarkably, the
core principles on which LTA is based have not changed since
it was first described by OBrien and Born more than 50 years
ago.1,2 Platelet-rich plasma is stirred in a cuvette that is placed
between a light source and a photocell. The plasma is cloudy due
to suspension of platelets and allows relatively little light to pass
through. Upon the addition of an agonist, platelets aggregate,
and the sample becomes clearer, permitting greater light
transmission. Transmission of light is detected by the photocell
and recorded as a function of time (Figure). An enhancement
of modern LTA is the capacity to simultaneously monitor
ATP secretion from dense granules using a luciferin-luciferase
reagent.3
Representative normal and abnormal LTA tracings. Figure A shows normal responses to epinephrine 1.0 M (curve 1)
and collagen 2.0 mg/mL (curve 2). At critical concentrations, weak agonists such as epinephrine and ADP induce a biphasic
response. The primary wave of aggregation reflects response to the addition of exogenous agonist. When normal platelets are
activated, they secrete an endogenous pool of agonists. The secondary wave represents aggregation in response to this endogenous pool. Figure B is a tracing from a patient with epistaxis and menorrhagia. The response to ADP 5.0 M (curve 1) is
attenuated. A primary wave of aggregation is followed by deaggregation and an absent secondary wave of aggregation. There
is no response to epinephrine 5.0 M (curve 2). The patients platelets also demonstrated abnormal ATP secretion in response
to ADP and epinephrine. Electron microscopy of the patients platelets revealed an absence of dense granules consistent with
a diagnosis of dense granule storage pool disorder.

When do I recommend LTA?

Rare PFDs such as Glanzmann thrombasthenia (GT) and Bernard


Soulier syndrome (BSS) are usually diagnosed early in life
because of the severity of the bleeding phenotype. In syndromic
PFDs, the presence of associated features (e.g., oculocutaneous
albinism in HermanskyPudlak syndrome) may facilitate diagnosis.
Far more common, however, are mild PFDs without associated
syndromic features. Even with a detailed bleeding history, these disorders may be difficult
to distinguish from normal variation due to the high frequency of mucocutaneous bleeding
symptoms in the general population. In one study of healthy adults, epistaxis, easy
bruising, and prolonged bleeding after tooth extraction were reported in 25 percent, 18
percent, and 18 percent of subjects, respectively; and 47 percent of menstruant women
reported heavy menses.4

patients with a suspected bleeding disorder were enrolled. A normal basic hemostatic
work-up (platelet count, PT, APTT, fibrinogen, and von Willebrand panel) was required
for study entry. The median ISTH-BAT score was significantly greater in patients than in
controls (12 vs. 0). Among the patients who had a suspected bleeding disorder, however,
there was no difference in score between the 41 who were diagnosed with a PFD by LTA
and the 38 with a normal LTA (11 vs. 12).5

A bleeding history may be taken using either a conventional approach or a standardized


bleeding assessment tool (BAT). Published BATs are effective in discriminating patients
with bleeding disorders from healthy controls, but they have not proven effective in
predicting the presence of PFDs among patients referred for a suspected bleeding
disorder. A recent study investigated the utility of the International Society on Thrombosis
and Haemostasis (ISTH)-BAT for this purpose. Twenty-one healthy controls and 79

In my practice, I employ a conventional approach to the bleeding history. I query


patients about spontaneous, mucocutaneous bleeding symptoms (location, frequency,
severity, treatment); bleeding with hemostatic challenges (e.g., surgery, trauma,
childbirth, menstruation); and family history of bleeding. I also ask about prescription
and over-the-counter medications (e.g., anti-platelet agents) and comorbidities (e.g., liver
disease, uremia) that could promote bleeding. I use data from the history to estimate
the pretest probability of a PFD. I recommend LTA when I judge the pretest probability
to be intermediate or high (i.e., >5-10%). My approach
is likely to be highly operator-dependent and result in
Other Features
over-testing (only about half of the patients I refer for
LTA have an identifiable defect). Although BATs hold
promise for ameliorating these deficiencies, I have opted
not to use BATs in my practice until they are shown to
improve diagnostic accuracy compared with conventional
approaches to assessment of bleeding.
Macrothrombocytopenia

Table. Characteristic LTA Patterns of Selected Platelet Function Disorders


Disorder

Aggregation Response
Primary
ADP

Secondary
ADP

Epinephrine

AA

Collagen

Ristocetin

Inherited disorders
GT

Absent

Absent

Absent

Absent

Absent

Normal

BSS

Normal

Normal

Normal

Normal

Normal

Absent

Dense granule SPD

Normal

Decreased
or Absent

Variable

Normal

Normal

Normal

Deficiency of dense granules seen


with platelet electron microscopy. May
be syndromic or occur in isolation.

Acquired disorders
Aspirin

Normal

Decreased
or Absent

Decreased or
Absent

Absent

Decreased
or Absent

Normal

P2Y12 inhibitors (e.g.,


clopidogrel)

Decreased or
Absent

Absent

Normal

Normal

Normal

Normal

Glycoprotein IIb/IIIa
inhibitors

Absent

Absent

Absent

Absent

Absent

Normal

Uremia

Normal

Decreased

Variable

Decreased

Normal or
Decreased

Normal

Inherited aspirin-like defects in AA


metabolism show a similar pattern

GT=Glanzmann thrombasthenia; BSS=BernardSoulier syndrome; SPD=storage pool disorder; ADP=adenosine diphosphate; AA=arachidonic acid

N O N M A L I G N A N T H E M A T O L O G Y 44

How is LTA interpreted?


A panel of seven platelet agonists ADP, epinephrine,
collagen, thrombin receptor-activating peptide, the
thromboxane A2 mimetic U46619, arachidonic acid, and
ristocetin is recommended by an international consensus
panel.6 Aggregation tracings (Figure) are evaluated
with respect to a number of parameters including
shape change; length of lag phase; slope of aggregation;
presence of a secondary wave of aggregation produced
by weak agonists, such as epinephrine; maximal percent
aggregation; and presence of deaggregation.
The Hematologist Compendium 2010-2015

The pattern of aggregation and/or secretion defects observed with the agonist panel
facilitates identification of a variety of hereditary and acquired PFDs (Table) and also
helps to localize lesions to specific pathways. Classic examples include GT (caused by
mutations in integrin IIb3) and BSS (caused by mutations in the platelet glycoprotein
Ib-IX-V complex), both of which have characteristic LTA signatures (absent aggregation to
all agonists except ristocetin and absent ristocetin-induced agglutination, respectively).
In practice, LTA abnormalities often do not conform to one of the textbook patterns
shown in the Table. Many of these non-specific abnormalities are of uncertain clinical
significance and molecular pathogenesis. However, critical new insights are emerging from
genotypephenotype correlation analyses such as the United Kingdom Genotyping and
Platelet Phenotyping (GAPP) study.7 GAPP is a multicenter study of patients with clinically
suspected inherited PFDs. Results from the first 111 patients were recently reported.8 A
defect by LTA was detected in 64 patients (58%). Most defects (84%) involved Gi (i.e., ADP
and epinephrine) receptor signaling, thromboxane A2 receptor signaling, or dense granule
secretion. Targeted genotyping subsequently identified novel mutations in the P2Y12 ADP
receptor, the thromboxane A2 receptor, and FLI1 and RUNX1, genes encoding transcription
factors that participate in megakaryopoiesis and dense granule secretion.8,9
Several limitations must be borne in mind in the interpretation of LTAs. First, some
LTA abnormalities are observed in a small percentage of healthy volunteers. Clinical
correlation is required, but not always conclusive, in distinguishing disease from
normal variation. LTA cannot assess platelet function under flow conditions and may be
influenced by a variety of preanalytic, analytic, and biologic variables.

Which variables can confound LTA?


Many drugs affect platelet function in vitro. Drugs with reversible effects on platelets (e.g.,
NSAIDs) should be held for at least three days and drugs with irreversible effects (e.g.,
aspirin, clopidogrel) for at least 10 days prior to testing. An international consensus panel
recommends a short rest period and avoidance of smoking and caffeine prior to blood
collection to mitigate the effects of epinephrine release.6 Because chylomicrons in plasma
can interfere with LTA, patients should be counseled to avoid fatty meals shortly before
testing. Laboratories must adhere to meticulous specifications for sample collection and
processing to avoid in vitro platelet activation. The platelet count in platelet-rich plasma
should be measured. Results may be inaccurate when the platelet count is < 150 109/L.
Studies should be completed within four hours after blood collection.6

Conclusion
As they have for decades, a detailed bleeding history and LTA form the backbone of the
diagnostic evaluation of patients with suspected PFDs. These approaches are not without
shortcomings. The bleeding history has relatively poor specificity because of the frequency
of mucocutaneous bleeding symptoms in the general population. LTA is labor-intensive,
requires considerable expertise and a fresh blood sample, is not well-standardized,
and shows overlap between normal variation and pathology across certain parameters.
Continued development of BATs and improved methods for measuring platelet function
are needed to overcome these limitations. Genotypephenotype correlation studies are
elucidating novel defects, are providing new insights into platelet function in health and
disease, and may ultimately pave the way for more exacting approaches to diagnosis.
1. OBrien JR. The adhesiveness of native platelets and its prevention. J Clin Pathol. 1961;14:140-149.
2. Born GV. Aggregation of blood platelets by adenosine diphosphate and its reversal. Nature.
1962;194:927-929.
3. Miller JL. Platelet function testing: an improved approach utilizing lumi-aggregation and an interactive
computer system. Am J Clin Pathol. 1984;81:471-476.
4. Mauer AC, Khazanov NA, Levenkova N, et al. Impact of sex, age, race, ethnicity and aspirin use on
bleeding symptoms in healthy adults. J Thromb Haemost. 2011;9:100-108.
5. Lowe GC, Lordkipanidz M, Watson SP; UK GAPP study group. Utility of the ISTH bleeding assessment
tool in predicting platelet defects in participants with suspected inherited platelet function disorders. J
Thromb Haemost. 2013;11:1663-1668.
6. Cattaneo M, Cerletti C, Harrison P, et al. Recommendations for the standardization of light transmission
aggregometry: a consensus of the working party from the platelet physiology subcommittee of SSC/ISTH.
J Thromb Haemost. 2013;11:1183-1189.
7. Watson SP, Lowe GC, Lordkipanidz M, Morgan NV; GAPP consortium. Genotyping and phenotyping of
platelet function disorders. J Thromb Haemost. 2013;11:351-363.
8. Dawood BB, Lowe GC, Lordkipanidz M, et al. Evaluation of participants with suspected heritable platelet
function disorders including recommendation and validation of a streamlined agonist panel. Blood.
2012;120:5041-5049.
9. Stockley J, Morgan NV, Bem D, et al. Enrichment of FLI1 and RUNX1 mutations in families with excessive
bleeding and platelet dense granule secretion defects. Blood. 2013;122:4090-4093.

Dr. Cuker indicated no relevant conflicts of interest.

ASH-SAP, Sixth Edition


The sixth edition of the ASH SelfAssessment Program (ASH-SAP) is
available for ordering. This educational
resource will help hematologists and others
working in hematology-related fields to stay
up to date with the latest advances in the
rapidly evolving disciplines of adults and
pediatric hematology, including malignant
and nonmalignant disorders. The program
includes multimedia featuring 3D animation
and audio, a question-and-answer book,
and six tests you can take to earn up to
70 CME/MOC Credits. For pricing, to
order your copy, and for any additional
information, visit www.ash-sap.org.

N O N M A L I G N A N T H E M A T O L O G Y 45

The Hematologist Compendium 2010-2015

Anti-Phospholipid Antibodies and Pregnancy July/August 2014


The authors have indicated that there has been no update regarding the content of this article since the original publication date in 2014.

DEEPA SURYANARAYAN, MD, 1 AND DAVID GARCIA, MD 2


1. Clinical Lecturer, Department of Medicine, McMaster University Medical
Center, University of Calgary, Alberta, Canada
2. Professor of Medicine, University of Washington School of Medicine,
Seattle, WA

The Question
What is your approach to a patient with laboratory evidence of anti-phospholipid
antibodies who is pregnant or considering pregnancy?

Our Response
Antiphospholipid antibody syndrome (APS) is a heterogeneous autoimmune disorder
characterized by arterial and venous thromboembolic events and obstetric complications
(especially recurrent fetal losses) in association with persistent laboratory evidence of
anti-phospholipid antibodies (aPL). The aPL associated with the disease include the lupus
anticoagulant (LA), anti-cardiolipin (-CL), and anti-2 glycoprotein-1 (anti-2 GP-1). APS
and aPL can occur in isolation (i.e., primary) or in association with other disease processes
(i.e., secondary) including connective tissue diseases, most commonly systemic lupus
erythematosus (SLE).

Diagnostic Criteria for APS


The presence of aPL alone does not constitute APS. The international classification criteria
for APS (the Sapporo criteria) were introduced in 1998 and revised in 2006.1 To make
a definitive diagnosis of APS, the presence of at least one clinical feature (thrombosis
or pregnancy morbidity) and one laboratory abnormality must be observed, and the
laboratory abnormality must be present on two or more occasions at least 12 weeks
apart. Laboratory parameters are as follows: IgG and/or IgM CL or anti-2 GP-1 in
moderate to high titre (i.e., a titre > 40 for IgG or for IgM or a titre > the 99th percentile),
or documentation of the LA. Further details about the diagnostic criteria for APS can be
found in the Journal of Thrombosis and Haemostasis.1

Maternal and Fetal Morbidity


The pathogenesis of aPL-related pregnancy morbidity is incompletely understood. For
a pregnant woman, APS (and perhaps the presence of aPL) raises the possibility that
she is at increased risk for thrombotic events and/or obstetric complications such as
recurrent pregnancy losses, preeclampsia, eclampsia, intra-uterine growth restriction,
and Hemolysis, Elevated Liver enzymes, Low Platelets (HELLP) syndrome. In a European
referral center cohort of 1,000 patients with APS (many of whom had a connective
tissue disease), 188 pregnancies were documented over 10 years; among patients in
this group, early pregnancy loss (16.5%), intra-uterine growth restriction (26.3%), and
prematurity (48.2%) were common.2 A systematic review found that lupus anticoagulant
was associated with preeclampsia (OR 2.34), intra-uterine growth restriction (OR 4.65),
and late (after 10 weeks of gestation) fetal loss (OR 4.73); however, numerous case-control
studies were included in the analysis. When the analysis was restricted to cohort studies,
late fetal loss was the only outcome associated with the aPL.3 Catastrophic APS (CAPS), a
rare but devastating form of this syndrome, has been associated with pregnancy and the
puerperium with 0.9 percent of the pregnancies from the aforementioned cohort being
complicated by CAPS.2

Laboratory Diagnosis of aPL


The laboratory identification of aPL can be problematic because of variability in the
sensitivity of assays and reagents, high false-negative and false-positive detection rates,
lack of standardization of assays, and lack of adherence to established guidelines. To
address these issues, guidelines for laboratory detection of aPL were published in
2009 by the Scientific Standardization Subcommittee (SSC) of the International Society
of Thrombosis and Haemostasis (ISTH) and by the British Committee on Standards
in Hematology. The Clinical Laboratory Standards Institute (CLSI) has also prepared
guidelines for laboratory testing for LAC that are to be published soon. Before counseling
a patient about risk or recommending therapy, we suggest that the hematologist ensure
that aPL testing is not only confirmed on more than one occasion but also performed by a
laboratory that follows approved aPL testing guidelines.

Pre-Pregnancy Counseling
Management of a woman with APS should start with pre-pregnancy counseling that
focuses on the involved risks. Discussion should include risks of maternal thrombosis and
fetal loss (both early- and late-term) as well as the increased risk of placenta-mediated
complications such as preeclampsia, HELLP syndrome, preterm delivery, and intra-uterine
growth restriction. Involvement by a multidisciplinary management team that includes

N O N M A L I G N A N T H E M A T O L O G Y 46

a maternal-fetal medicine specialist and a hematologist experienced in the management


of thrombophilic conditions is recommended. APS patients on chronic anticoagulation
should be informed about the potential teratogenic effects of warfarin, and oral
anticoagulation should be switched to a therapeutic dose of low-molecular-weight heparin
(LMWH) either before or very shortly after conception.
For all women with APS, the American College of Chest Physicians (ACCP) evidence-based
guidelines4 recommend commencing low-dose aspirin on confirmation of pregnancy, not
only for its antithrombotic effects but also to reduce the risk of preeclampsia.5 In practice,
it would be reasonable for a woman with obstetric APS to begin aspirin during the time
that she and her partner are trying to conceive, because the risk of ASA is low and the
impact of a delay between conception and ASA use is unknown.

Pregnancy with aPL or APS Clinical Management Scenarios


1. Pregnant women with persistent evidence of aPL who do not fulfill the diagnostic
criteria for APS
There are no robust data to guide management of pregnant women in this cohort. Dr.
Anne Lynch et al. measured aPLs in early pregnancy in 451 low-risk nulliparous women
and found that 24.4 percent had aPLs.8 The rate of fetal loss in this cohort was higher than
those without aPLs (15.8 vs. 6.5%), but the rate of adverse maternal outcomes was similar
in both groups. We usually recommend low-dose ASA because it is easy to take and there
is a reasonably high quality of evidence that it reduces the risk of preeclampsia; however,
the significant uncertainty about the benefit of any antepartum, antithrombotic therapy in
women who do not meet criteria for APS means that decisions should be individualized.
The role of post-partum thromboprophylaxis in this patient group is not established.
2. Women with adverse obstetric outcomes plus aPL, but no history of thrombosis
In three meta-analyses of randomized trials in women with APS, the combination of
ASA with prophylactic heparin significantly reduced pregnancy loss (RR 0.46)7 or firsttrimester loss (OR 0.39)9 and increased live births (RR 2.3).10 These analyses, however,
have several limitations, including a small number of trials from which to gather data,
small sample size within trials, and low quality of trial design. Compared with women
who have had recurrent pregnancy losses, but whose thrombophilia testing is negative,
women with a history of obstetric APS are at increased risk for both arterial and venous
thrombotic events.11 In the NOH-APS observational study among women with prior
fetal loss, those treated with LMWH and low-dose aspirin had lower pregnancy loss but
higher preeclampsia rates than other women. Therefore, the evidence that combined
(LMWH + ASA) therapy improves outcomes is, at best, of modest quality. Nonetheless, we
suggest combination LMWH + ASA for pregnant women with APS and adverse obstetric
outcomes, not only because there is some evidence that this will decrease the likelihood
of pregnancy complications, but also because LMWH + ASA should also reduce the risk
of thrombosis during pregnancy. However, because the quality of evidence to support
combination therapy over LMWH or ASA alone is not compelling, we suggest that the
hematologist consider not only patient preferences, but also the recommendation of a
maternal-fetal medicine specialist before choosing LMWH + ASA over LMWH or ASA alone.
The optimal doses for LMWH and ASA have not been defined, but we suggest 75 to 100 mg
of ASA per day plus prophylactic-dose LMWH (Table). For women with obstetric APS and
no history of thrombosis, thromboprophylaxis during the postpartum period should be
considered, but the net benefit is not well established.
3. Pregnant women with thrombotic APS
If a woman has APS and is anticoagulated for prior thrombosis, we suggest switching
to therapeutic-dose LMWH before six weeks gestation. Full-intensity anticoagulation is
important during the pregnancy and the postpartum period since these women have
a significant, ongoing risk of thrombosis that is likely magnified by pregnancy. If a
woman is on therapeutic-dose LMWH, delivery should be planned, and LMWH should be
discontinued at least 24 hours prior to the procedure. Because there is no high-quality
evidence to define the optimal anticoagulant management of patients who desire regional
anesthesia, communication with maternal-fetal medicine and anesthesiology is especially
important. For a woman with prior thrombosis who has discontinued anticoagulation, we
suggest LMWH be started upon confirmation of pregnancy; whether to use prophylactic/
intermediate or therapeutic-dose LMWH would depend on the perceived risks of
thrombosis and bleeding. An elevated risk of thrombosis persists for up to 12 weeks
following delivery.12 The absolute risk for thrombosis beyond six weeks, however, is small.
Thus, thromboprophylaxis beyond the six-week mark of the postpartum period should be
reserved for those patients with an especially high baseline risk.

Conclusion
Management of pregnant women with APS is challenging. Despite the imperfect data
that are currently available, pregnancy outcomes in women with APS who are managed
in centers that use a multidisciplinary approach, which includes input from clinicians
experienced in thrombophilia management and fetal-maternal medicine, is often favorable.

The Hematologist Compendium 2010-2015

Table. Overview of the Therapeutic Options Used in Antiphospholipid Syndrome Pregnancy


Drug

Use

Evidence

Safety in pregnancy

Safety in breastfeeding

Aspirin
75-100 mg daily from conception
until 36 weeks gestation

Reduction of fetal loss

No randomized controlled trials of aspirin


for preventing VTE

Will cross the placenta;


human data inconsistent,
but risk is likely low

Does enter breast


milk, but at low
doses; should be
safe

Prevention of eclampsia
Antiplatelet effect

Some evidence for aspirin use in improving


pregnancy outcomes, specifically
reduction in rates of preeclampsia6

Warfarin
Discontinue as soon as pregnancy is confirmed

Not recommended in pregnancy unless


LMWH may be less effective (e.g.,
prosthetic heart valves)

Teratogenic between 6-12 weeks of


gestation; switch to LMWH before 6
weeks of gestation

Teratogenic

Not excreted in
breast milk

Unfractionated heparin (UFH)

May be used in event of massive


pulmonary embolism

Most studies using UFH have been superseded by studies using LMWH.

Safe

Not excreted in
breast milk

Some first-trimester analyses


have shown small increased
risk of gastroschisis

If rapid reversal of anticoagulation is


needed during peripartum period or
operative procedures
LMWH

Drug of choice for women on warfarin,


women who have had VTE or arterial
thrombosis during pregnancy, women
with previous pregnancy complications, or
women who require thromboprophylaxis

Evidence for its role in preventing firsttrimester loss remains controversial7

Does not cross placenta

Does enter breast


milk, but of little
concern due to low
bioavailability

Steroids (e.g., prednisone)

Little evidence for benefit in APS; used


in immune thrombocytopenia associated
with APS or SLE

Minimal evidence of therapeutic benefit

Cleft palate reported with


first-trimester use

Low concentrations
in breast milk

Intravenous immunoglobulins

Used in a small number of women with


APS as therapy for concomitant, immunemediated thrombocytopenia

No additional benefit when added to conventional therapy with ASA and LMWH

Crosses placenta after 30


weeks of gestation

Excretion is unknown

Hydroxychloroquine
(HCQ)

Used when women have coexisting SLE

Mild antithrombotic effects

No reports of fetal toxicity

Considered safe
despite excretion in
breast milk

Plasmapheresis

Reserved for treatment of catastrophic


APS

Rarely used in pregnancy

Unknown

Benefit outweighed by its adverse effects


(e.g., preeclampsia, gestational diabetes,
increased risk of preterm deliveries)

Decreased risk of congenital heart block in


women on HCQ in case-controlled studies
Limited to case reports

Adapted with permission from Soh, MC and Nelson-Piercy C. Antiphospholipid syndrome in pregnancy http://informahealthcare.com/doi/abs/10.1586/eog.10.57. Expert Rev Obstet Gynecol. 2010;5:748,749.

Future studies that expand our understanding of this complex disease, that clearly define
obstetric and thrombotic risks, and that identify optimal antithrombotic regimens are
needed.
1. Miyakis S, Lockshin MD, Atsumi T, et al. International consensus statement on an update of the
classification criteria for definite antiphospholipid syndrome (APS). J Thromb Haemost. 2006;4:295-306.
2. Cervera R, Serrano R, Pons-Estel GJ, et al. Morbidity and mortality in the antiphospholipid
syndrome during a 10-year period: a multicentre prospective study of 1000 patients. Ann Rheum Dis.
2015;74:1011-1018.
3. Abou-Nassar K, Carrier M, Ramsay T, et al. The association between antiphospholipid antibodies and
placenta mediated complications: A systematic review and meta-analysis. Thromb Res. 2011;128:77-85.
4. Bates SM, Greer IA, Middeldorp S, et al. VTE, thrombophilia, antithrombotic therapy, and pregnancy:
Antithrombotic Therapy and Prevention of Thrombosis, 9th ed: American College of Chest Physicians
Evidence-Based Clinical Practice Guidelines. Chest. 2012;141:e691S-e736S.

10. Mak A, Cheung MW, Cheak AA, et al. Combination of heparin and aspirin is superior to aspirin alone
in enhancing live births in patients with recurrent pregnancy loss and positive anti-phospholipid
antibodies: a meta-analysis of randomized controlled trials and meta-regression. Rheumatology (Oxford).
2010;49:281-288.
11. Gris J, Bouvier S, Molinari N, et al. Comparative incidence of a first thrombotic event in purely
obstetric antiphospholipid syndrome with pregnancy loss: The NOH-APS observational study. Blood.
2012;119:2624-2632.
12. Kamel H, Navi BB, Sriram N, et al. Risk of a thrombotic event after the 6-week postpartum period. N Engl J
Med. 2014;370:1307-1315.

Dr. Suryanarayan and Dr. Garcia indicated no relevant conflicts of interest.

5. Farquharson RG, Quenby S, Greaves M. Antiphospholipid syndrome in pregnancy: a randomized,


controlled trial of treatment. Obstet Gynecol. 2002;100:408-413
6. Empson M, Lassere M, Craig JC, et al. Recurrent pregnancy loss with antiphospholipid antibody: a
systematic review of therapeutic trials. Obstet Gynecol. 2002;99:135-144.
7. Laskin CA, Spitzer KA, Clark CA, et al. Low molecular weight heparin and aspirin for recurrent
pregnancy loss: results from the randomized, controlled HepASA Trial. J Rheumatol. 2009;36:279-287.
8. Lynch A, Marlar R, Murphy J, et al. Antiphospholipid antibodies in predicting adverse pregnancy
outcome. A prospective study. Ann Intern Med. 1994;120:470-475.
9. Ziakas PD, Pavlou M, Voulgarelis M. Heparin treatment in antiphospholipid syndrome with recurrent
pregnancy loss: a systematic review and meta-analysis. Obstet Gynecol. 2010;115:1256-1262.

N O N M A L I G N A N T H E M A T O L O G Y 47

The Hematologist Compendium 2010-2015

Treatment Approaches to Castleman Disease with the Advent of


Anti-Interleukin-6 Therapy March/April 2015
Due to the recent publication date of this article, an update has not been requested.

FRITS VAN RHEE, MD, PhD, MRCP(UK), FRCPath, 1 AND


DAVID FAJGENBAUM, MD, MSc 2
1. Professor of Medicine, Myeloma Institute for Research and Therapy,
University of Arkansas for Medical Sciences; Co-Founder and Scientific Board
Member Castleman Disease Collaborative Network; Little Rock, AR
2. Adjunct Assistant Professor of Medicine, Department of Hematology/
Oncology, University of Pennsylvania; Co-Founder and Executive Director
Castleman Disease Collaborative Network; Philadelphia, PA

The Question
What are your treatment approaches to Castleman disease with the advent of anti
interleukin-6 therapy?

Our Response
Castleman disease (CD) describes a group of heterogeneous lymphoproliferative disorders
that share common histopathological lymph node changes. CD can present with localized
(unicentric CD or UCD) or generalized lymphadenopathy (multicentric CD or MCD). MCD
should be further divided into human herpesvirus 8 (HHV-8) positive and HHV-8negative
MCD (Table). The latter is also referred to as idiopathic MCD (iMCD). MCD patients can exhibit
a spectrum of clinical features from mild flu-like symptoms to sepsis-like multiorgan failure.1 It
is important to distinguish these three entities since they require entirely different therapeutic
approaches.

UCD
UCD presents with lymphadenopathy confined to one lymph node region, and many patients
are asymptomatic. Symptoms are usually due to compression of vital structures such as
the trachea, blood vessels, or nerves. UCD occurs most frequently in younger females. The
diagnosis is made when excisional lymph node biopsy shows characteristic Castleman
changes. Approximately 90 percent of UCD cases demonstrate the hyaline vascular (HV)
subtype. The other 10 percent of patients with UCD demonstrate mixed cellularity or
plasmacytic (PC) pathology and may have constitutional symptoms such as fever, fatigue,
and weight loss. UCD is not associated with HHV-8 or HIV infection. There is an increased
association with lymphoma, which may have been present all along, or the UCD may
transform into lymphoma. The etiology of UCD is poorly understood, and there is usually
no excess interleukin-6 (IL-6) secretion. Limited studies have demonstrated abnormal
cytogenetics or evidence of monoclonality in stromal cells, but the significance of these
findings is unknown.2 Surgical extirpation is curative in 95 percent of UCD cases.3 Unresectable
cases can be treated with rituximab and steroids, which may induce complete responses
or shrink the mass sufficiently to make it resectable. UCD lymphadenopathy is highly
vascularized, and embolization is a further therapeutic option. Involution of lymphadenopathy
after radiotherapy has also been reported. Difficult cases require a multimodal approach and
are best managed at an experienced center.

HHV-8Positive MCD

studies. Siltuximab, which is a monoclonal antibody to IL-6, has not been studied in HHV-8
positive MCD because it did not bind to viral IL-6 in preclinical studies.

iMCD
HHV-8negative MCD patients present with generalized lymphadenopathy, constitutional symptoms, and can also develop multiorgan failure. The disease can wax and wane, be gradually
progressive, or have severe episodic flares resulting in death.8 In a recent literature review, 22
percent of HHV-8negative MCD patients had died by a median follow-up time of 29 months.9
HHV-8negative MCD is driven by pro-inflammatory hypercytokinemia, most notably of IL-6,
which leads to: anemia; anasarca due to hypoalbuminemia from IL-6mediated liver dysfunction and VEGF-mediated vascular permeability; and systemic inflammation with elevated ESR,
CRP, and fibrinogen.8 Some patients may develop kidney, liver, and bone marrow failure and
succumb to their disease. Patients with HHV-8negative MCD may also have one or more features of POEMS (polyneuropathy, organomegaly, endocrinopathy, M protein, and skin changes)
syndrome, or a concurrent POEMS syndrome.10 Patients with one or more features of POEMS
often have less florid constitutional symptomatology.
The etiology of the pro-inflammatory hypercytokinemia in iMCD has not yet been identified.
The pathologic cell and the responsible intracellular inflammatory pathway responsible for
producing the IL-6 in these patients has also not been elucidated. Hypothesized etiologies
include an unknown virus, a small population of malignant cells, or germline genetic mutations in the immune system.1 No single gene causing iMCD has been identified, but systematic
sequencing has not been performed. The disease may be influenced by ethnicity and genetic
factors such as a polymorphism of the IL-6 receptor.11 Asian patients may demonstrate violaceous skin lesions or interstitial pneumonitis usually not seen in other populations. Currently,
a diagnosis of iMCD can be made when patients have 1) histopathology typical of CD on
excisional lymph node biopsy, 2) multiple regions of enlarged lymph nodes, 3) negative quantitative PCR for HHV-8 in the peripheral blood or negative LANA-1 staining of the lymph node
biopsy, and 4) systematic exclusion of diseases known to demonstrate Castleman-like histopathology (e.g., systemic lupus erythematosus, Epstein-Barr virus, lymphoma, IgG4-associated
lymphadenopathy). Hence, both HHV-8positive and negative MCD are not purely pathologic
diagnoses. Efforts are currently underway to establish international consensus around clinical,
pathologic, and exclusion criteria for the diagnosis of iMCD. Although the PC variant predominates in iMCD, HV and mixed pathology have also been reported.
HHV-8-negative MCD has been historically managed with corticosteroids, rituximab, and/or
chemotherapeutic agents derived from the CHOP regimen. Corticosteroids may temporarily
control symptoms, but patients relapse on tapering. Rituximab has not been systematically
evaluated in iMCD and a limited number of case reports suggest that patients often relapse.
Monoclonal antibodies targeting IL-6 have been recently developed and more rigorously
evaluated. A single-arm study of 28 Japanese patients on tocilizumab demonstrated
a high response rate in terms of symptoms, laboratory parameters, and reduction in
lymphadenopathy.12 Siltuximab was evaluated in a double-blind, placebo-controlled,
randomized study using a control arm of best supportive care including up to 60 mg of
prednisone. The combined durable symptomatic and tumor response was 34 percent, and
50 percent of patients remained on drug for the duration of the study. This study provided
the first placebo-controlled evidence for an iMCD therapy, and siltuximab is the first drug
approved for iMCD by the U.S. Food and Drug Administration and European Medicines
Agency. Both siltuximab and tocilizumab are safe and well-tolerated.13 For patients that do
not respond to antiIL-6 therapy, immunosuppressants, immunomodulators, biologics, and
cytotoxic chemotherapies, including cyclosporine, sirolimus, bortezomib, thalidomide,
anakinra, interferon-, cyclophosphamide, and etoposide, have been reported to have some
success in case reports or small series.

HHV-8positive MCD presents with generalized lymphadenopathy and constitutional symptoms


and can progress to multiorgan failure leading to death. The disease is driven by the excessive
release of viral IL-6, which is encoded by the HHV-8 virus and drives human IL-6, IL-10, and
vascular endothelial growth factor (VEGF) secretion. HHV-8positive MCD was first described
during the AIDS epidemic and is classically thought to be due to lytic replication of HHV-8
in the setting of HIV infection.4 However, there are also patients with HHV-8positive MCD
who are HIV negative. These patients may have another
cause for immunosuppression that impairs their control of
Table. Features of the Different Types of Castleman Disease
HHV-8.5 Approximately 10 to 20 percent of individuals have
been exposed to HHV-8, and the virus remains dormant in
Type of
Type of
Pathology
IL-6Driven
Virologic Status
Treatment
Castleman
Lymphadenopathy
Inflammatory
B lymphocytes. A diagnosis of HHV-8positive MCD can
Disease
Syndrome*
be rendered if the patient has 1) a pathologic diagnosis
of CD made on an excisional lymph node biopsy and 2)
Unicentric
Localized
90 percent hyaline
Typically not
Negative for
Complete excision
actively replicating HHV-8 virus detected in the peripheral
vascular
HHV-8 by QPCR
or negative
blood by molecular testing (quantitative polymerase chain
LANA-1 stain
reaction [PCR]) or a positive stain of the lymph node for the
HHV-8 latency associated nuclear antigen (LANA-1). HIVMulticentric
Generalized
Plasmacytic or
Yes
Positive for HHV-8 Rituximab
positive, HHV-8positive MCD patients may have coexistent
HHV-8Positive
hepatosplenomegaly
plasmablastic
by QPCR
etoposide
Kaposi sarcoma and are prone to develop HIV-associated
May be positive
Optional valgancifor HIV
clovir maintenance
lymphomas. Lymph node pathology shows plasmacytic or
plasmablastic changes, and the plasma cells may exhibit
Multicentric
Generalized
Mostly plasmacytic,
Yes, but variable
Negative for
Siltuximab
light chain restriction. Intranodal microlymphomas have also
HHV-8Negative
hepatosplenomegaly
but can be hyaline
clinical presentaHHV-8 by QPCR
Tocilizimab
been reported. HHV-8positive MCD is effectively treated with
(Idiopathic)
vascular or mixed
tion from mild to
or
cellularity
very severe
Negative LANA-1 Rituximab
rituximab, which depletes the reservoir of HHV-8positive
stain
Chemotherapy in
cells and significantly reduces the risk of lymphoma.6
severe cases
Negative for HIV
More severely afflicted patients may additionally require
etoposide. Some experts recommend maintenance therapy
*Symptoms: fevers, night sweats, anorexia, weight loss, fatigue. Laboratory abnormalities: anemia, thrombocytopenia or thrombocytosis,
with valganciclovir. HIV-positive patients should receive
elevated C-reactive protein, Westergren erythrocyte sedimentation rate, fibrinogen, hypergammaglobulinemia, abnormal renal function,
7
appropriate HAART therapy. The value of tocilizumab, which
increased interleukin-6 (IL-6), vascular endothelial growth factor, interleukin-10.
blocks the human IL-6 receptor is the subject of ongoing
Abbreviations: QPCR, quantitative polymerase chain reaction; LANA-1, latency associated nuclear antigen.
N O N M A L I G N A N T H E M A T O L O G Y 48

The Hematologist Compendium 2010-2015

Choice of Therapy for iMCD


In the authors opinion, patients with iMCD should first be treated with antiIL-6 therapy
approved in that region (siltuximab in North America and the European Union; tocilizumab
in Japan). Patients with few symptoms or laboratory abnormalities suggestive of little excess
IL-6 may not respond well to antiIL-6 blockade and should be considered for rituximab and
steroids. Patients with severe hypercytokinemia and organ failure may not respond sufficiently
to antiIL-6 targeting monoclonal antibodies, and they require combination chemotherapy
or consideration of experimental treatment. Dosing intervals can be spaced out in selected
patients responding to anti-IL-6 therapy. Progressive motor polyneuropathy suggesting
coexistent POEMS does not respond well to rituximab or to IL-6targeted therapy, and these
patients require autologous stem cell transplantation as part of their treatment plan.

1. Fajgenbaum DC, Van Rhee F, Nabel CS. HHV-8-negative, idiopathic multicentric Castleman disease: novel
insights into biology, pathogenesis, and therapy. Blood. 2014;123:2924-2933.
2. Chang KC, Wang YC, Hung LY, et al. Monoclonality and cytogenetic abnormalities in hyaline vascular
Castleman disease. Mod Pathol. 2014;27:823-831.
3. Talat N, Belgaumkar AP, Schulte KM. Surgery in Castlemans disease: a systematic review of 404
published cases. Ann Surg. 2012;255:677-684.
4. Dupin N, Diss TL, Kellam P, et al. HHV-8 is associated with a plasmablastic variant of Castleman disease
that is linked to HHV-8-positive plasmablastic lymphoma. Blood. 2000;95:1406-1412.
5. Dossier A, Meignin V, Fieschi C, et al. Human herpesvirus 8-related Castleman disease in the absence of
HIV infection. Clin Infect Dis. 2013;56:833-842.
6. Gerard L, Michot JM, Burcheri S, et al. Rituximab decreases the risk of lymphoma in patients with hivassociated multicentric Castleman disease. Blood. 2012;119:2228-2233.
7. Bower M. How I treat HIV-associated multicentric Castleman disease. Blood. 2010;116:4415-4421.

The Future

8. van Rhee F, Stone K, Szmania S, et al. Castleman disease in the 21st century: an update on diagnosis,
assessment, and therapy. Clin Adv Hematol Oncol. 2010;8:486-498.

The introduction of rituximab has been a major advance in HHV-8positive MCD, while therapy
with IL-6targeting monoclonal antibodies is an important innovation in iMCD. However, anti
IL-6 therapy is not effective for all patients, and it is not curative, as cessation of treatment
results in relapse. In 2012, we co-founded the Castleman Disease Collaborative Network (CDCN;
www.castlemannetwork.org) to accelerate research and elucidate the pathogenesis of MCD.
In 2.5 years, we have assembled a 23-member Scientific Advisory Board representing seven
countries; built a global community of more than 200 researchers and physicians worldwide;
leveraged the community to establish and execute an international research agenda; and
engaged patients throughout the entire process. We are currently finalizing plans to establish
a global registry/natural history study, which we believe will be crucial for establishing
diagnostic criteria and improving patient care. We also plan to launch viral discovery, serum
proteomics, intracellular inflammatory pathway identification, and sequencing studies. We
invite you to register on our website, attend our annual meeting that occurs during the ASH
annual meeting, conduct research, contribute samples for research, and encourage your
patients to enroll in our registry.

9. Fajgenbaum DC, Liu A, Ruth J, et al. HHV-8-negative, idiopathic multicentric Castleman disease (iMCD):
a description of clinical features and therapeutic options through a systematic literature review [abstr].
Blood. 2014;124:4861a.
10. Dispenzieri A, Armitage JO, Loe MJ, et al. The clinical spectrum of Castlemans disease. Am J Hematol.
2012;87:997-1002.
11. Stone K, Woods E, Szmania SM, et al. Interleukin-6 receptor polymorphism is prevalent in HIV-negative
Castleman disease and is associated with increased soluble interleukin-6 receptor levels. PLoS One.
2013;8:e54610.
12. Nishimoto N, Kanakura Y, Aozasa K, et al. Humanized anti-interleukin-6 receptor antibody treatment of
multicentric Castleman disease. Blood. 2005;106:2627-2632.
13. van Rhee F, Wong RS, Munshi N, et al. Siltuximab for multicentric Castlemans disease: a randomised,
double-blind, placebo-controlled trial. Lancet Oncol. 2014;15:966-974.

Dr. Van Rhee and Dr. Fajgenbaum indicated no relevant conflicts of interest.

ASH Meetings
Visit www.hematology.org/Meetings to learn about upcoming meetings on malignant and
nonmalignant hematology. Find out date, location, and program or content information for
each meeting online.

N O N M A L I G N A N T H E M A T O L O G Y 49

The Hematologist Compendium 2010-2015

Treatment with Direct Oral Anticoagulants May/June 2015


Due to the recent publication date of this article, an update has not been requested.

DAVID GARCIA, MD
Professor of Medicine, University of Washington School of Medicine; Assistant Medical Director
for Antithrombotic Therapy at the University of Washington Medical Center, Seattle, WA

The Question
Given the recent U.S. Food and Drug Administration (FDA) approval of edoxaban, the fourth
direct oral anticoagulant (DOAC) to come to the U.S. market since 2010, what should I know
about this class of medications, and what is their role in patient care? Are there patients for
whom these medications are not (or possibly not) appropriate?

My Response
This is an exciting time for clinicians caring for patients with (or at risk for) thrombosis,
but the amount of clinical trial data, not to mention the dosing peculiarities, relevant to the
DOACs, can be difficult to digest. Here, I will try both to provide take-home points about
safety and efficacy relative to other therapies as well as to address some of the practical
management questions I am frequently asked about DOACs.
One or more of the DOACs has now been approved for three main indications: 1) the
prevention of stroke and systemic embolism in patients with atrial fibrillation (AF), 2) the
acute and extended treatment of venous thromboembolism, and 3) the prevention of venous
thromboembolism (VTE) after total knee or hip arthroplasty (Table).
These medications are small molecules that inhibit their serine protease target by binding
to its active site (thrombin in the case of dabigatran and activated factor X [FXa] in the case
of rivaroxaban, apixaban, and edoxaban). There are some very important pharmacologic
differences with warfarin: All these drugs rely, to varying degrees, on elimination by the
kidneys. The prescriber must consult the package insert of a particular agent to determine
whether his or her patients renal function would be a contraindication to treatment or
mandate a dose adjustment. There are no important dietary interactions for the DOACs.
A possible exception is that the bioavailability of rivaroxaban seems to be increased if
it is taken with a meal; however, the clinical relevance of this is uncertain. Although the
DOACs can interact with other medications, the number of potentially clinically important
interactions is small compared to the corresponding number with warfarin.

My patient taking a DOAC needs to undergo an elective procedure.


How far in advance of the procedure should I stop the medication,
and do I need to provide bridge therapy?
Unlike vitamin K antagonists (VKAs), all of the DOACs have both a rapid onset of action
and a relatively short half-life. DOACs induce maximal anticoagulant effect within two
to three hours of the first dose and, unless renal function is impaired, will essentially
disappear from the plasma 24 to 48 hours after the last dose. Therefore, periprocedural
anticoagulation (bridging) with a parenteral anticoagulant is not necessary for a patient
taking a DOAC unless the patient undergoes a surgery that will preclude swallowing or
gastrointestinal absorption. Patients with renal impairment will need to interrupt the
DOAC longer (see individual prescribing information for more details), but others can
safely undergo most interventions as early as 24 to 48 hours after their last DOAC dose.

My patient has moderate renal impairment. Does that mean I


cannot prescribe a DOAC?
Although all DOACs depend on renal clearance more than warfarin does, renal
insufficiency is not a contraindication in all cases. Indeed, an analysis of the subgroup
of patients with moderate renal impairment (creatinine clearance 30 to 50 mL/min)
in the phase III clinical trials indicates that the DOACs (when dosed according to the
manufacturers recommendations) are at least as safe as warfarin for such individuals.1
The thresholds at which dose-adjustment is required vary depending on the DOAC;
most providers would avoid DOACs altogether in patients with severe renal impairment
(creatinine clearance < 30 mL/min). That being said, the U.S. FDA has approved a dose
adjustment for apixaban in patients on hemodialysis. Apixaban is the DOAC that depends
least on renal clearance. On the other end of the spectrum of renal function, the U.S. FDA
has recommended that edoxaban not be used in AF patients with excellent renal function
(creatinine clearance > 95 mL/min) because of a subgroup analysis suggesting it may be
less effective than warfarin to prevent stroke or systemic embolism in this subgroup.

When will antidotes for some or all of the DOACs be available?


Should I avoid prescribing DOACs until antidotes reach the market?
The lack of experience managing DOAC-associated bleeding is an understandable concern.
However, the absence of a dedicated antidote should probably not be a major consideration
in the decision to use (or not use) this class of medications. Outcome data from randomized
phase III clinical trials that included more than 100,000 patients show that, although no
antidote was available, the likelihood of fatal bleeding was lower among the patients
N O N M A L I G N A N T H E M A T O L O G Y 50

randomized to a DOAC than among the patients randomized to warfarin. Furthermore, taking
a DOAC (as opposed to warfarin) was not associated with a higher likelihood of death among
patients who experienced a major bleed. These seemingly counterintuitive observations
are probably explained by several factors: 1) Despite the availability of vitamin K and
plasma products, warfarin-associated major bleeding has a very high associated mortality;
2) compared with warfarin, the DOACs cause 50 percent fewer intracranial bleeds; and 3)
the short half-life of the DOACs means that in many cases of major bleeding, no antidote
or reversal agent is needed. These observations notwithstanding, antidotes and reversal
agents in late-phase clinical development are likely to reach the U.S. market within the next
two to three years. The DOAC antidotes that are furthest in the development process are
andexanet (a factor Xa decoy molecule that is intended for patients on a FXa inhibitor) and
idarucizumab (an antibody fragment targeting the direct thrombin inhibitor, dabigatran).

What should I do if consulted about a patient with DOACassociated bleeding?


Many patients with DOAC-associated bleeding will be best managed with supportive
care such as red blood cell transfusions and volume support; excellent renal perfusion
will maximize the rate at which the anticoagulant effect dissipates. Unless a patient has
compromised renal function, the plasma DOAC concentration (and corresponding drug
effect) will have decreased by more than 50 percent six to 12 hours after the last dose. In
the small minority of patients with DOAC-associated bleeding who require more aggressive
efforts to counteract the anticoagulant effect immediately, one could consider prothrombin
complex concentrates or recombinant activated factor VII.2 However, the thrombosis risk
associated with these interventions is not trivial, and the rationale for their use is based
entirely on animal and other preclinical models.

Condition

Dabigatran

Rivaroxaban

Apixaban

Edoxaban

Stroke Prevention in
Atrial Fibrillation

*
-

Venous Thromboembolism
Treatment
Acute
Extended
Knee Arthroplasty
Hip Arthroplasty

*Edoxaban and dabigatran are approved for the acute treatment of venous thromboembolism only after
an initial 5-day course of treatment with a parenteral anticoagulant (e.g., low-molecular-weight heparin,
fondaparinux, heparin).

In which patients should DOACs be avoided?


In general, DOACs should be used only for approved indications. Because patients with
mechanical prosthetic heart valves were not included in the phase III studies, and because
dabigatran was inferior to warfarin in the only head-to-head comparison that has been
done in this population,3 the DOACs should not be prescribed to patients with mechanical
prosthetic heart valves. There are also subgroups of patients with (or at risk for) VTE where
routine DOAC use should also be avoided pending further evidence. Because of their highly
prothrombotic tendencies, patients with active cancer, heparin-induced thrombocytopenia,
and bona fide antiphospholipid syndrome are other groups for whom I do not think DOACs
should be a first option. Instead, we should work hard to enroll such patients in clinical trials
that will provide evidence on which we can base future practice.

Conclusion
In summary, the DOACs offer hematologists and well-selected patients four oral alternatives
to warfarin and other VKAs.4,5 Important ongoing trials will tell us more about how best
to manage serious DOAC-related bleeding, whether DOACs can be safely used in patients
with cancer-associated VTE, and what is the safest way to combine DOACs with antiplatelet
agents (e.g., in patients with previous acute coronary syndromes). In the meantime, many
patients for whom these medications are indicated (and for whom cost is not a barrier) will
likely choose them over warfarin because of their convenience and impressive safety profile.
1. Harel Z, Sholzberg M, Shah PS, et al. Comparisons between novel oral anticoagulants and vitamin K
antagonists in patients with CKD. J Am Soc Nephrol. 2014;25:431-442.
2. Siegal DM, Garcia DA, Crowther MA. How I treat target-specific oral anticoagulant-associated bleeding.
Blood. 2014;123:1152-1158.
3. Eikelboom JW, Connolly SJ, Brueckmann M, et al. Dabigatran versus warfarin in patients with
mechanical heart valves. N Engl J Med. 2013;369:1206-1214.
4. Yeh CH, Gross PL, Weitz JI. Evolving use of new oral anticoagulants for treatment of venous
thromboembolism. Blood. 2014;124:1020-1028.
5. Dzeshka MS, Lip GY. Non-vitamin K oral anticoagulants in atrial fibrillation: Where are we now? Trends
Cardiovasc Med. 2014;S1050-1738:00195-00199.

Dr. Garcia indicated no relevant conflicts of interest.

The Hematologist Compendium 2010-2015

2016 The American Society of Hematology


All materials contained in this compendium are protected by copyright laws and may not be used, reproduced,
or otherwise exploited in any manner without the express prior written permission of The Hematologist: ASH News and Reports.
All articles in this compendium were originally published in The Hematologist.

You might also like